分からない問題はここに書いてね384

このエントリーをはてなブックマークに追加
1132人目の素数さん
さあ、今日も1日頑張ろう★☆

前スレ
分からない問題はここに書いてね383
http://uni.2ch.net/test/read.cgi/math/1374882185/
2132人目の素数さん:2013/09/24(火) 12:16:02.67
ここは分からない問題を書くスレです。
分からない問題に答えてもらえるスレではありません。
3132人目の素数さん:2013/09/24(火) 12:42:32.50
n,m,x,a∈N, x=qn+r ,r<n ,0≦x<n×m, f:qn+r → rm+qとするとき
0≦a<n×mとなるaについて {x: ∃n∈N(f^n(a)=x) }が同値類になることは明らかである。
このとき各同値類の位数と同値類の数と任意の2つの数が同じ同値類に入る条件をf^nを計算する以外の方法で求めよ。
4132人目の素数さん:2013/09/24(火) 13:07:58.71
(x-1)(x+1)(x^2+x+1)(x^2-x+1)
答えと解き方教えてください
5132人目の素数さん:2013/09/24(火) 13:23:50.03
最悪手間さえかければ解けるだろ
6132人目の素数さん:2013/09/24(火) 13:44:55.92
>>4
問題文は正しく書きましょう
式をどうしたいの?
7132人目の素数さん:2013/09/24(火) 14:37:21.42
式を展開する問題です 
8132人目の素数さん:2013/09/24(火) 14:45:40.99
バカなんだから正面攻撃で展開しろ
9132人目の素数さん:2013/09/24(火) 14:56:51.97
分かりました バカみたいに地道に計算しますね
10132人目の素数さん:2013/09/24(火) 15:03:19.27
数Aの範囲なのですが、ω^43=何になるのでしょうか
11132人目の素数さん:2013/09/24(火) 15:08:45.91
>>10
ω^2=?
ω^3=?
ω^4=?
順にやっていけば
12132人目の素数さん:2013/09/24(火) 15:34:02.12
>>11
ω^2=-ω-1
ω^3=1
ω^4=ω^3・ω

これをω^6,ω^7,...と続けるんですか??すいません
13132人目の素数さん:2013/09/24(火) 15:36:45.11
あ!(ω^3)^14・ωで
答えはωですか!?
146:2013/09/24(火) 15:37:26.91
>>4
第一項X第三項
第二項X第四項
とすれは少し簡単
15132人目の素数さん:2013/09/24(火) 16:31:58.66
>>13
デスネー。
16132人目の素数さん:2013/09/24(火) 17:54:11.32
以下の積分についてです。
I=A∫(1/t^3)exp(-t)dt - B ∫(1/t)exp(-t)dt [t:0〜∞] A,Bは正の有限値(0≦A,B)
この積分の値を評価するときに、指数積分、オイラーの定数を使って有限値に収束する
という結論は得られるのでしょうか?
17132人目の素数さん:2013/09/24(火) 18:01:31.66
>>16
しないよ、0の近くは?
18132人目の素数さん:2013/09/24(火) 18:13:14.66
>>17
下限値で被積分関数が発散しますよね。
この項に対して
S=Io+I
としてIoが→∞に近づけば、S=0あるいは有限値に持っていく事は可能なのかなー。
19132人目の素数さん:2013/09/24(火) 18:18:42.16
>>18
だめー
20132人目の素数さん:2013/09/24(火) 18:37:34.03
ちょっと拡張してしまいますが、前述の積分に対して
S=∫(exp(-t)/t^3)*{δ(f(t))+A-B*t^2}dt [t:0〜∞]
とあった場合、f(t)がある関数(t:0〜∞の間で-∞〜∞の区間が存在する場合つまりその区間で∫δ=1が成立する場合)の時、f(t)=0を満たす時のt値で(Iの中身)が選択されますか?
21132人目の素数さん:2013/09/24(火) 19:04:07.07
>>20
意味不明
22132人目の素数さん:2013/09/24(火) 19:13:07.34
連投すみません。とても基本的な事なのですが、
∫f(x)δ(x)dx=f(0) [x:-∞〜∞]
とデルタ関数とf(x)の積だと上記のように選択された関数が出てきますが、
∫f(t)δ(t)+g(t)dt [t:−∞〜∞]
の時はf(0)が選択されると和である∫g(t)dtは別の積分になってしまうので、
g(0)は出てこないですよね。
ここでtが共通ならば先の積分の値が確定すると期待しましたがダメそうですね。
23132人目の素数さん:2013/09/24(火) 19:17:49.29
>>22
問題を分けよう
1.
∫[0,1](t^α)dtが収束する条件は?
2.
デルタ関数は超関数である
24132人目の素数さん:2013/09/24(火) 19:23:10.93
L=Q(√2,√3)、×をQ上のテンソル積とします。
このときQ(√2)×LとQ(√3)×LとL+L(直和)がQ上の多元環として同型になるそうです(「代数学のひろがり」P291)が、Q(√2)×LとQ(√3)×Lの同型対応、Q(√2)×LとL+Lの同型対応はどう定義すればよいのでしょうか?
加群としての同型なら次元を比べれば済みますが…
25132人目の素数さん:2013/09/24(火) 22:01:42.95
広義積分の問題、
(2→4) 1 / √{(x-2)(4-x)} dx
の解法を悩んでいます。
ヒントのみでも頂きたいので、
よろしくお願いします。
26132人目の素数さん:2013/09/24(火) 22:08:58.30
t=√{(x-2)/(4-x)} で変数変換すれば積分できるだろ
27132人目の素数さん:2013/09/24(火) 22:30:25.43
= ∫[-1→+1] 1 / √{1-x^2} dx
= ∫[-1→+1] 1 / √{1-(sin(t))^2} d(sin(t))
= ∫[-π/2→+π/2] dt
= π
28132人目の素数さん:2013/09/24(火) 22:44:05.88
>>26
ありがとうございます。
うまくいかないのですが…
t=√{(x-2)/(4-x)} とおくと、
tの範囲は0→無限で良いのでしょうか?
29132人目の素数さん:2013/09/24(火) 23:46:20.68
n→∞ lim {1+(1/n)}^n = e を利 用して n→∞ lim {1-(4/5n)}^2n を求めよ
という問題なのですが、よろしくお願いします。
30132人目の素数さん:2013/09/25(水) 00:44:52.17
31132人目の素数さん:2013/09/25(水) 00:50:05.59
囮なんかには引っ掛かりたくないな
32132人目の素数さん:2013/09/25(水) 01:31:10.87
>>25
どなたかお願いします。
33132人目の素数さん:2013/09/25(水) 01:33:48.83
ナレーションを、文字化してくれ。
早すぎて聞き取れん。
34132人目の素数さん:2013/09/25(水) 01:35:31.11
>>32
被積分関数をマイナス3だけ平行移動したら >>27 になるよ
35132人目の素数さん:2013/09/25(水) 02:00:19.89
>>34
ありがとうございます。
ようやく理解できました、
かなりスッキリしました!

あと一問、解けずに迷っているものがあるので教えて頂けるとありがたいです。
良かったら教えてください。

(0→1) 1 / {(x+1)√(x^2+x+1)} dx
36132人目の素数さん:2013/09/25(水) 03:01:33.97
>>35
なんか面倒なだけであまり教育的な問題じゃない気がするんだけど、その式合ってるの?
www.wolframalpha.com/input/?i=integrate%28+1%2F%28%28x%2B1%29*sqrt%28x*x%2Bx%2B1%29%29%2C+0%2C1%29
右下のStep-by-step solution (要メンバー登録[無料]) で丁寧に式変形まで教えてくれるよ

最後のステップだけ省略されてるけど、そこは
atanh(x) = 1/2 *{log(1+x) - log(1-x)} を使います
37132人目の素数さん:2013/09/25(水) 08:49:57.78
すみません、>>3の質問お願いします。
38132人目の素数さん:2013/09/25(水) 09:06:35.36
>>37
ひと目みて、ごめん
39132人目の素数さん:2013/09/25(水) 09:07:00.70
質問じゃなくてポエムだろ
40狢 ◆yEy4lYsULH68 :2013/09/25(水) 10:11:14.50


■□■□■□■□■□■□■□■□■□■□■□■□■□■□■□■□■□■□■□■□■□■□■□■□■□■□■□■□■□■□■□■□■□
□■□■□■□■□■□■□■□■□■□■□■□■□■□■□■□■□■□■□■□■□■□■□■□■□■□■□■□■□■□■□■□■□■
■□■□■□■□■□■□■□■□■□■□■□■□■□■□■□■□■□■□■□■□■□■□■□■□■□■□■□■□■□■□■□■□■□
□■□■□■□■□■□■□■□■□■□■□■□■□■□■□■□■□■□■□■□■□■□■□■□■□■□■□■□■□■□■□■□■□■
■□■□■□■□■□■□■□■□■□■□■□■□■□■□■□■□■□■□■□■□■□■□■□■□■□■□■□■□■□■□■□■□■□
□■□■□■□■□■□■□■□■□■□■□■□■□■□■□■□■□■□■□■□■□■□■□■□■□■□■□■□■□■□■□■□■□■
■□■□■□■□■□■□■□■□■□■□■□■□■□■□■□■□■□■□■□■□■□■□■□■□■□■□■□■□■□■□■□■□■□
□■□■□■□■□■□■□■□■□■□■□■□■□■□■□■□■□■□■□■□■□■□■□■□■□■□■□■□■□■□■□■□■□■
■□■□■□■□■□■□■□■□■□■□■□■□■□■□■□■□■□■□■□■□■□■□■□■□■□■□■□■□■□■□■□■□■□
□■□■□■□■□■□■□■□■□■□■□■□■□■□■□■□■□■□■□■□■□■□■□■□■□■□■□■□■□■□■□■□■□■
■□■□■□■□■□■□■□■□■□■□■□■□■□■□■□■□■□■□■□■□■□■□■□■□■□■□■□■□■□■□■□■□■□
□■□■□■□■□■□■□■□■□■□■□■□■□■□■□■□■□■□■□■□■□■□■□■□■□■□■□■□■□■□■□■□■□■
4137:2013/09/25(水) 13:34:26.66
>>3は直感的に言って閉じた式は無さそうですか?
可能性が高ければ諦めます。
42132人目の素数さん:2013/09/25(水) 22:01:55.74
>>41
{x: ∃n∈N(f^n(a)=x)}
この集合上にどういった同値関係が置かれているの?
43132人目の素数さん:2013/09/25(水) 22:18:41.31
>>37
> すみません、>>3の質問お願いします。
言葉が無さ杉。
fの定義は曖昧だし、同値類云々をいうなら、まず同値関係を定義しておかなければそれはpoemといわれても仕方が無い
44132人目の素数さん:2013/09/26(木) 01:32:56.52
詩人は馬鹿ではない。
言葉を選べ。
45132人目の素数さん:2013/09/26(木) 01:46:49.13
詩人もピンキリだろ
都合のいいところだけ拾ってくるなよ
46132人目の素数さん:2013/09/26(木) 02:11:46.18
数学者もピンキリだ。
証明が詩と呼べる水準の人は、
歴史的にも多くはない。
47132人目の素数さん:2013/09/26(木) 02:35:28.98
そもそも数学の証明は詩を指向するものではないからな
48132人目の素数さん:2013/09/26(木) 03:05:07.50
詩人でもないのがポエムを書くからダメなんだろ
49132人目の素数さん:2013/09/26(木) 06:43:03.14
美しくないと数学ではない(棒)
50132人目の素数さん:2013/09/26(木) 07:00:52.91
高校生が絶賛するオイラーの公式(笑)
51132人目の素数さん:2013/09/26(木) 07:03:37.38
たとえ歴史的定理であっても、オリジナルの証明を読む機会なんてそうそうないよな
52132人目の素数さん:2013/09/26(木) 07:09:37.34
>>35-36

 {2√(xx+x+1) +(x-1)}{2√(xx+x+1) -(x-1)} = 4(xx+x+1) - (x-1)^2 = 3(x+1)^2,
を利用して
 sinhθ = (x-1)/{(x+1)√3},
 coshθ = 2√(xx+x+1)/{(x+1)√3},
とおける。さらに
 t = tanhθ = (x-1)/{2√(xx+x+1)},
とおくと、
 dt = 3(x+1)/{4(xx+x+1)^(3/2)} dx,
よって
 ∫1/{(x+1)√(xx+x+1)} dx
   = ∫4(xx+x+1)/{3(x+1)^2} dt
   = ∫1/(1-tt) dt
   = (1/2)log|(1+t)/(1-t)|
   = log(|x+1|√3) - log|2√(xx+x+1) -(x-1)|
   = log|2√(xx+x+1) +(x-1)| - log(|x+1|√3),

答え (1/2)log(3).
53>>3:2013/09/26(木) 07:51:00.34
a,bが同値であるというのは
0≦a,b<n+mに対して∃n∈N(f^n(a)=b)が成り立つことを言います
54132人目の素数さん:2013/09/26(木) 08:09:38.08
>>52
最初の質問者ではないのだけど、その式変形を思いつく思考の流れを示してほしいです
Wolfram先生の解法のような何かパターンがあるのでしょうか?
55132人目の素数さん:2013/09/26(木) 08:18:21.66
>>53
それが同値関係の条件を満たしている事は自明ですか?
fの定義は自明ですか? fに逆写像が存在しないと同値関係とは言えませんよね
56132人目の素数さん:2013/09/26(木) 08:23:24.77
>>55
同値関係の条件は満たしてます。
証明は長くなるので略します。
57132人目の素数さん:2013/09/26(木) 08:29:01.03
>>3
説明は長くなるので省略します
583:2013/09/26(木) 08:40:30.99
まあ、分らない問題を解くスレだから答えを期待してるわけじゃないから答えなくてもいいですよwww
この問題に興味が無い人は数学に適さないと思うので・・・
やられたらやりかえす!倍返しだ!
59132人目の素数さん:2013/09/26(木) 08:51:01.23
ばーーか!!!
土下座しろ
60132人目の素数さん:2013/09/26(木) 09:03:44.67
倍返しは撤回しましょう。
やられたらやりかえす!100倍返しだ!!
61132人目の素数さん:2013/09/26(木) 09:15:03.11
やられたらやりかえす!256倍返しだ!!
62狢 ◆yEy4lYsULH68 :2013/09/26(木) 09:23:49.58
>>61
その精神はとても大切や。特にこの馬鹿板ではナ。そやし報復っちゅうんは何年も掛かるワ。

ケケケ狢
63132人目の素数さん:2013/09/26(木) 09:45:40.84
>>61
0は何倍しても0のままだ。
64132人目の素数さん:2013/09/26(木) 09:46:57.33
>>63
つまり「触らぬ神に祟りなし」ということか。
6555:2013/09/26(木) 09:55:26.96
f:qn+r → rm+q
ああ定義分かったわ。
例えば、
n=11, m=23, (nm=253)
f(56) = f(5*11+1) = 1*23+5 = 28
f(28) = f(2*11+6) = 6*23+2 = 140
f(140) = f(12*11+8) = 8*23+12 = 196
f(196) = f(17*11+9) = 9*23+17 = 224
f(224) = f(10*11+2) = 2*23+10 = 56
みたいに循環するのね。

q < (qn+r)/n = mn/n < m ...(※)
rm+q < rm + m = (r+1)*m ≦ nm ...(※※)
だから象はfの定義域内に留まり(※※)、fには逆があって(※)有限なので必ず最初に戻ってこれると。
という事で循環軌道の単位に類別って事と同じですね。

まあ1つだけでも例示したり言葉ちょっと足せば興味持って答えてくれる人いたかも知れんのに
なんでワザワザ敵を作ろうとするかな。
66132人目の素数さん:2013/09/26(木) 10:01:24.42
ここは分からない問題を解くスレでもポエム自慢をするスレでもないが
67狢 ◆yEy4lYsULH68 :2013/09/26(木) 10:08:02.72
>>64
そやけど『もし不幸にして触ってしもうたらもうシマイ』っちゅう事や。そやし痴漢と一緒や。
ほしてや『最後まで一生祟られる』っちゅう結末を迎えるのや。判るわナ。

ケケケ狢
68132人目の素数さん:2013/09/26(木) 11:59:19.26
>>63
馬鹿に使う時間はない
69132人目の素数さん:2013/09/26(木) 12:04:50.78
昔の人は収束のことを収斂と言ってたそうですが
ここにその時代の人いますか?
70132人目の素数さん:2013/09/26(木) 12:50:48.47
Xを任意の整数とし Sを素数かどうか精査する数とする
F(X)=X*{(S+1)(S-1)+(-5+S^2)X−8X^2−4X^3}/(2X+1)^2 が 
[ (S-1)^2/4 ≧F(X)>0 ] [(S-1)/2≧ X>0] の範囲において
格子点を通るときSは素数でない
71132人目の素数さん:2013/09/26(木) 12:52:28.09
またポエムか
72132人目の素数さん:2013/09/26(木) 12:53:16.90
難しそう
73132人目の素数さん:2013/09/26(木) 12:54:35.60
私の前に道はない、私の後に道はできる
74132人目の素数さん:2013/09/26(木) 12:54:39.78
エスパーがな
75132人目の素数さん:2013/09/26(木) 13:03:44.27
vanishing idealの日本語訳は何ですか?
ググったら零化イデアルと出てきたけどannihilatorとは違うし……
消滅イデアル?は聞いたこともないし

一応、定義は体k上の多項式環K=k[X_1,...,X_n]の部分集合Sに対して
I(V):={f\in K;f(x)=0, forall x\in S}
です
積がすべて0になるものの全体がannihilatorで,代入して0になる多項式全体がvanishing idealで似てる?から両方とも零化イデアルでいいのかな……
76132人目の素数さん:2013/09/26(木) 14:04:14.05
I(S) だろ?
はおいといて、単に「Sのイデアル」って呼んでるけどな
Hartshorne P.3 でもこれに相当する英語
77132人目の素数さん:2013/09/26(木) 14:13:36.43
あ、I(S)でした
ありがとうございます
78132人目の素数さん:2013/09/26(木) 14:54:59.80
ああそうか
Syoumetsu IdealだからI(S)なのか
79132人目の素数さん:2013/09/26(木) 16:16:30.72
ヌルポイデアル
80132人目の素数さん:2013/09/26(木) 18:15:49.68
defining ideal
81132人目の素数さん:2013/09/26(木) 20:41:35.01
>>53
> a,bが同値であるというのは
> 0≦a,b<n+mに対して∃n∈N(f^n(a)=b)が成り立つことを言います
こういう書き方をするから嫌われる。nは重要な定数なのだから同じ文字を使ってはいかんだろ。

それはそれとして、面白い問題なので、読んでもらえるような書き方で問題を書いておこう。

n、mを自然数として、Aで集合{x|x∈Z、0≦x<m*n}を表すとする。
Aの各元xに対しx=q*n+r(q,rは整数で0≦r<n)とあらわしたときf(x)=r*m+qで写像f:A→Aを定める。
次の問に答えよ。
(1)fは1対1上への写像であることを示せ。
(2)a,b∈Aに対し、自然数sが存在してf^s(a)=bのときa〜bと関係〜を定めれば
  〜は集合Aの元の上の同値関係であることを示せ。(ここに、f^sはfをs回連続して施すことを表す)
(3)集合Aを関係〜で割った同値類の類の数、ならびに、各類の元の数を求めよ。
(4)a∈Aが与えられたとき、b∈Aがa〜bを満たすための条件をa,m,nを使って書き表せ。
82132人目の素数さん:2013/09/26(木) 21:02:22.79
我々は遠くから来て遠くへ行く
83132人目の素数さん:2013/09/26(木) 21:05:13.51
>>51
先ず以て言語の壁 (ラテン語、フランス語、イタリア語、ドイツ語、オランダ語、ロシア語、中国語...) があって、
そこを超えても往々にしてオリジナルは証明に不備があるからなあ。
84132人目の素数さん:2013/09/26(木) 21:52:26.10
フビニなんて名前からして不備ありそうだもんな
85132人目の素数さん:2013/09/26(木) 22:02:37.38
野分の翌日はとても趣深くておもしろい
86132人目の素数さん:2013/09/27(金) 09:28:36.51
>>81
n・m進法で表現して桁をひっくり返すだけやん
87132人目の素数さん:2013/09/27(金) 10:11:59.78
そんな...、n・m進法だと、0≦x<nm の数値はみんな1桁やん
って違うの?
88132人目の素数さん:2013/09/27(金) 11:13:22.99
(n×m)進法ではなくて、(n,m)進法という意味で書いた
位ごとに単位が違う進法ね。
89132人目の素数さん:2013/09/27(金) 11:43:09.46
(n,m)進法的とやらの定義が曖昧だけど(例.140=12*11+8=11*11+20=? の(11,23)進表記はどうなんのとか...)、
それって f^2(x) = x だとでも言いたいの?
ひっくり返すだけなんでしょ?
>>65 の例を見ればそうじゃないって分かるけど
90132人目の素数さん:2013/09/27(金) 11:46:52.47
訂正: 140=12*11+8=11*11+19
2桁目がうまく納まらないけどいいのか? 無視したら今度は一意に定まらないわけだが
91132人目の素数さん:2013/09/27(金) 15:41:42.70
>>81
f は Z/(mn-1)Z での m倍写像と考えるのがわかりやすい。
この問題は、分数 x/(mn-1) を m進法で小数展開したときの循環節の長さがどうなるかなどと同じだから、
簡単な答えはないだろう。
92132人目の素数さん:2013/09/27(金) 20:11:25.40
f(x)=x (0≦x<m*n)となるx(つまり元が1個の類)を求めるのだけでも結構面白い。
93132人目の素数さん:2013/09/27(金) 20:32:02.60
http://i.imgur.com/qM01eUI.jpg
?で書いたところがグラフでは分かるのですが、計算するとマイナス無限大になるはずなのに無限大になってしまいます。
正しい計算式を教えてください
94132人目の素数さん:2013/09/27(金) 20:42:27.85
>>93
(log3)/x が悪さをしているように思えるのかい?
95132人目の素数さん:2013/09/27(金) 20:50:16.38
(log3)/x は1/xをlimを使って無限大なりました
96132人目の素数さん:2013/09/27(金) 20:54:36.80
Lim x→∞にした時は分かりました
97132人目の素数さん:2013/09/27(金) 20:57:12.23
それが気になるなら無理やり
2log(x)+log(3)=2(log(x)+(1/2)log(3))=2log(1og((√3)x)として
t=(√3)x とおけば 
(2log(x)+log(3))/x=2√3(log(t)/t) だ
98132人目の素数さん:2013/09/27(金) 21:02:32.99
こういった問題の時はグラフからだいたいの極限を求めていいんですか?
それともちゃんと計算したほうがいいんですか?
99132人目の素数さん:2013/09/27(金) 21:04:10.71
マルチに答えるとこれからもマルチするようになるぜ
100132人目の素数さん:2013/09/27(金) 21:07:45.52
両方とも大事って事ですか?
101132人目の素数さん:2013/09/27(金) 22:57:15.92
そうだよ
マルチに答えることも、マルチすることも大事
102132人目の素数さん:2013/09/27(金) 23:02:15.26
マルチは消費生活センターに通報します
103132人目の素数さん:2013/09/27(金) 23:06:18.60
>>98
「グラフから求めただいたいの極限」www
見ることの出来ない無限の果ての話でっせ、そんなもんを答えと認めはる人がいてはると思うてんでっか? 
あかんよ、そんな考えでは
104132人目の素数さん:2013/09/27(金) 23:08:36.86
>>103
ほまんでんな、これだから・・・
105132人目の素数さん:2013/09/28(土) 13:50:14.95
>>24が分かる方はいらっしゃらないでしょうか?
106132人目の素数さん:2013/09/28(土) 15:03:10.92
数学の問題の質問ではないのですが
自分は高校生で数学がしにほど好きなのでもっと深く数学を学びたいと思っています
しかし高校数学は教科書をもらっていたのでどんどん進めたのですが大学の数学をしようと思っても何から始めれば良いのかわかりません
高校数学では整数問題が大好きなのですが何から勉強していけばいいのてしょうか
107132人目の素数さん:2013/09/28(土) 15:40:42.81
>>106
マセマがいいんじゃねー
108132人目の素数さん:2013/09/28(土) 15:44:21.29
>>106
解析や線型代数の教科書から入ればいい。
検索しても沢山出てくる。
この板にもあるけど
【激しく】解析と線型代数の本何がいい?【既出】7
http://uni.2ch.net/test/read.cgi/math/1379351269/
109132人目の素数さん:2013/09/28(土) 16:26:52.95
>>105
定義憶えてれば考えたろうが忘れたからダメ
110132人目の素数さん:2013/09/28(土) 21:56:55.68
>>109
何の定義でしょうか?
111132人目の素数さん:2013/09/29(日) 09:20:29.73
ttp://i.imgur.com/rtmlfnK.jpg

この問題を作ったはいいけど解けません
誰か解答お願いします
112132人目の素数さん:2013/09/29(日) 11:43:04.83
>>111
いやです
113132人目の素数さん:2013/09/29(日) 11:56:47.35
>>111
数学的センスの欠片も無い最底辺のアホが作った自作問題とかやめろよ
114132人目の素数さん:2013/09/29(日) 12:01:47.80
いやまあ解けないならいいんですけどね
115132人目の素数さん:2013/09/29(日) 12:33:35.85
>>114
まいった、次
116132人目の素数さん:2013/09/29(日) 12:38:42.95
ここはレベル低いですね
117132人目の素数さん:2013/09/29(日) 12:40:00.53
解けない問題など見る気も起きん
118132人目の素数さん:2013/09/29(日) 12:49:34.85
ってか、最小値存在しねえんじゃね?
119132人目の素数さん:2013/09/29(日) 13:22:04.86
問題って解けるような方向性とか
なんらかの意味のある方向性とか決めて作るもんだが
馬鹿が作る問題は本当に何も考えずに作っただけ
作りっぱなしで考える価値がない
120132人目の素数さん:2013/09/29(日) 13:23:39.48
>>118
よくわからないんでwolframalpha先生にやってもらった
例えばf4(x)にはあるようだがなんかめんどくさそうな感じだ
http://www.wolframalpha.com/input/?i=x%5Ex%5Ex%5Ex
121120:2013/09/29(日) 13:24:25.98
ちなみに俺は出題者ではない
122132人目の素数さん:2013/09/29(日) 13:38:41.26
新作ポエム発表はポエムスレでやれよ
123132人目の素数さん:2013/09/29(日) 15:55:31.14
>>122
削除対象になってるぞあのスレ
124132人目の素数さん:2013/09/29(日) 15:57:46.20
N=1で調子こんだんだろう
あ、つられちまった
125132人目の素数さん:2013/09/29(日) 18:19:00.81
ある自然数の二乗を、もとの自然数より4だけ大きい数でわると、
商が7で余りが2になるとき、ある自然数を求めなさい。

この問題の二次方程式の立て方がわかりません。
宿題の問題なので至急助けて欲しいです(´・ω・`)
126132人目の素数さん:2013/09/29(日) 18:43:41.26
>>125
一気に全文読むからどこから手を付けていいかわからなくなる
文節単位に区切って小さいところからコツコツと組み立てていく
途中で分からない量とかがでてきたら順番に文字を作って当てはめていく
127132人目の素数さん:2013/09/29(日) 19:15:15.60
>>126
やったけど自然数にならなかったんです。
だからヒントが欲しいのですが...
128132人目の素数さん:2013/09/29(日) 19:18:39.89
分かりました。
因数分解に戸惑ってました。
勉強不足ですみません。
ありがとうございました。
129132人目の素数さん:2013/09/29(日) 19:28:38.01
相加相乗平均って変数が消えないときに使ったらダメなんですか?
130132人目の素数さん:2013/09/29(日) 20:08:09.09
お前は一切使わない方が身のためだ
131132人目の素数さん:2013/09/29(日) 20:36:29.88
>>129
まともな教師とかできる塾講師とかきちんとした人に物理的に対面して
じっくりと学ぶべき
132132人目の素数さん:2013/09/29(日) 22:39:17.50
>>129
使っても良いけど、最大値や最小値を示す用途なんかには使えないから
問題によるとしか言えない。
133132人目の素数さん:2013/09/29(日) 23:05:39.60
次の極限値を求めよ
lim [n→∞]1/n*{(4n)!/(3n)!}^1/n
134132人目の素数さん:2013/09/29(日) 23:08:08.64
いやだ
135132人目の素数さん:2013/09/29(日) 23:27:26.79
分からない問題を書くスレなんだから、変なお願いなんかする奴よりマシでね?
136132人目の素数さん:2013/09/29(日) 23:28:47.20
256/(25e)ってwolframさんが言ってた
137132人目の素数さん:2013/09/29(日) 23:29:35.55
あ、256/(27e)だった
138133:2013/09/29(日) 23:31:29.90
これを授業中に出されたのですが全くわかりません
139132人目の素数さん:2013/09/30(月) 00:04:12.45
>>133
logとって区分求積するだけだろう。
考える所が無い。
140132人目の素数さん:2013/09/30(月) 02:26:31.04
複雑そうな積・累乗・商がでてきたらとりあえず対数とってみることくらいしろよ
141132人目の素数さん:2013/09/30(月) 06:55:37.97
>>133

自然対数をとって     >>140
 (1/n){log(4n)! - log(3n)!} - log(n)
 = (1/n)Σ[k=3n+1,4n] log(k) - log(n)
 = (1/n)Σ[k=3n+1,4n] log(k/n)
区分求積法と比べて    >>139
 → ∫[3,4] log(x)dx   (n→∞)
 = [ x・log(x) - x ](x=3,4)
 = 4log(4) - 3log(3) -1
 = log{(4^4)/(3^3・e)},
よって答は
 (4^4)/(3^3・e),     >>137
142132人目の素数さん:2013/09/30(月) 16:55:07.15
>>141
> = (1/n)Σ[k=3n+1,4n] log(k/n)
ちがう
143132人目の素数さん:2013/09/30(月) 17:27:04.13
>>142
どこが違うんだ?
キミはどうなると思うのか書いてごらん。
144132人目の素数さん:2013/09/30(月) 21:32:03.46
>>143
ツレタw
145132人目の素数さん:2013/09/30(月) 22:35:43.57
どういう内容の釣りなのか意味不明
つか、どうみても簡単な計算を分からなかったのを
必死で隠そうとしてるようにしかwwww
頭悪いと大変だな

142 名前:132人目の素数さん[sage] 投稿日:2013/09/30(月) 16:55:07.15
>>141
> = (1/n)Σ[k=3n+1,4n] log(k/n)
ちがう

144 名前:132人目の素数さん[sage] 投稿日:2013/09/30(月) 21:32:03.46
>>143
ツレタw
146132人目の素数さん:2013/10/01(火) 00:29:38.94
質問です
mを定数として
n個の変数a(1),a(2),…,a(n)
が、それぞれ0以上の値をとる、かつ

a(1) + a(2) + … + a(n) =n*m

が成立しているとします。
この時

a(1)^2 + a(2)^2 + … + a(n)^2

の最小値は、n*(m^2) でしょうか?

もしそうなら、その導き方を教えて下さい。
できれば、高校数学レベルの知識のみを用いた最小値の導き方が知りたいです。
よろしくお願いします。
147132人目の素数さん:2013/10/01(火) 00:31:21.66
>>146
いやです
148132人目の素数さん:2013/10/01(火) 00:45:52.06
>>146
x^2 + y^2 + z^2 = { (x+y+z)^2 + (x-y)^2 + (x-z)^2 + (y-z)^2 } / 3
の一般化を考えよう
149KingMathematician ◆LoZDre77j4i1 :2013/10/01(火) 00:51:46.94
Re:>>8 攻撃とは何事か.
Re:>>24 基底を考えれば良かろう.
Re:>>69 いるかもしれない.
150132人目の素数さん:2013/10/01(火) 01:11:38.07
>>148
なるほど

 a(1)^2 + a(2)^2 + … + a(n)^2
=[{a(1) + a(2) + … + a(n)}^2 + {a(1) - a(2)}^2 + … + {a(n-1) - a(n)}^2]/n

となるので、a(1)=a(2)=…=a(n)=m の時に最小で
最小値 n*(m^2) となるのですね

ありがとうございました!
151KingMathematician ◆LoZDre77j4i1 :2013/10/01(火) 01:43:40.16
Re:>>150 a(1)+a(2)+a(3)+a(4)=4m の時の a(1)^2+a(2)^2+a(3)^2+a(4)^2 の最小値はどうか.
152狢 ◆yEy4lYsULH68 :2013/10/01(火) 01:44:48.69
>>149


■□■□■□■□■□■□■□■□■□■□■□■□■□■□■□■□■□■□■□■□■□■□■□■□■□■□■□■□■□■□■□■□■□
□■□■□■□■□■□■□■□■□■□■□■□■□■□■□■□■□■□■□■□■□■□■□■□■□■□■□■□■□■□■□■□■□■
■□■□■□■□■□■□■□■□■□■□■□■□■□■□■□■□■□■□■□■□■□■□■□■□■□■□■□■□■□■□■□■□■□
□■□■□■□■□■□■□■□■□■□■□■□■□■□■□■□■□■□■□■□■□■□■□■□■□■□■□■□■□■□■□■□■□■
■□■□■□■□■□■□■□■□■□■□■□■□■□■□■□■□■□■□■□■□■□■□■□■□■□■□■□■□■□■□■□■□■□
□■□■□■□■□■□■□■□■□■□■□■□■□■□■□■□■□■□■□■□■□■□■□■□■□■□■□■□■□■□■□■□■□■
■□■□■□■□■□■□■□■□■□■□■□■□■□■□■□■□■□■□■□■□■□■□■□■□■□■□■□■□■□■□■□■□■□
□■□■□■□■□■□■□■□■□■□■□■□■□■□■□■□■□■□■□■□■□■□■□■□■□■□■□■□■□■□■□■□■□■
■□■□■□■□■□■□■□■□■□■□■□■□■□■□■□■□■□■□■□■□■□■□■□■□■□■□■□■□■□■□■□■□■□
□■□■□■□■□■□■□■□■□■□■□■□■□■□■□■□■□■□■□■□■□■□■□■□■□■□■□■□■□■□■□■□■□■
■□■□■□■□■□■□■□■□■□■□■□■□■□■□■□■□■□■□■□■□■□■□■□■□■□■□■□■□■□■□■□■□■□
□■□■□■□■□■□■□■□■□■□■□■□■□■□■□■□■□■□■□■□■□■□■□■□■□■□■□■□■□■□■□■□■□■
153狢 ◆yEy4lYsULH68 :2013/10/01(火) 01:49:10.23
>>151


■□■□■□■□■□■□■□■□■□■□■□■□■□■□■□■□■□■□■□■□■□■□■□■□■□■□■□■□■□■□■□■□■□
□■□■□■□■□■□■□■□■□■□■□■□■□■□■□■□■□■□■□■□■□■□■□■□■□■□■□■□■□■□■□■□■□■
■□■□■□■□■□■□■□■□■□■□■□■□■□■□■□■□■□■□■□■□■□■□■□■□■□■□■□■□■□■□■□■□■□
□■□■□■□■□■□■□■□■□■□■□■□■□■□■□■□■□■□■□■□■□■□■□■□■□■□■□■□■□■□■□■□■□■
■□■□■□■□■□■□■□■□■□■□■□■□■□■□■□■□■□■□■□■□■□■□■□■□■□■□■□■□■□■□■□■□■□
□■□■□■□■□■□■□■□■□■□■□■□■□■□■□■□■□■□■□■□■□■□■□■□■□■□■□■□■□■□■□■□■□■
■□■□■□■□■□■□■□■□■□■□■□■□■□■□■□■□■□■□■□■□■□■□■□■□■□■□■□■□■□■□■□■□■□
□■□■□■□■□■□■□■□■□■□■□■□■□■□■□■□■□■□■□■□■□■□■□■□■□■□■□■□■□■□■□■□■□■
■□■□■□■□■□■□■□■□■□■□■□■□■□■□■□■□■□■□■□■□■□■□■□■□■□■□■□■□■□■□■□■□■□
□■□■□■□■□■□■□■□■□■□■□■□■□■□■□■□■□■□■□■□■□■□■□■□■□■□■□■□■□■□■□■□■□■
■□■□■□■□■□■□■□■□■□■□■□■□■□■□■□■□■□■□■□■□■□■□■□■□■□■□■□■□■□■□■□■□■□
□■□■□■□■□■□■□■□■□■□■□■□■□■□■□■□■□■□■□■□■□■□■□■□■□■□■□■□■□■□■□■□■□■
154132人目の素数さん:2013/10/01(火) 07:22:10.91
(∂f(x(t),t)/∂x) dx-(∂f(x(t),t)/∂t) dt=0
の左辺って俗にいう全微分じゃないですか?
上の式がdx/dt=(∂f(x(t),t)/∂x)/(∂f(x(t),t)/∂t)
と掛けると決まってるんですけど。
分数の/と微分の/は違うからx(t)をtで微分したのとdx/dtが一致するのはいちいち証明がいるのに
みんな当たり前のように同一視してますよね?
杉浦の解析と解析概論を読んだけど分りません。
教えてください。
155132人目の素数さん:2013/10/01(火) 08:47:33.58
>>154
dxの定義をかいてみ
156132人目の素数さん:2013/10/01(火) 08:57:05.00
ちょっと誤解してるだけかと思いますが、最初の式は
全微分: df = (∂f/∂x)dx + (∂f/∂t) dt と
曲線:(x(ξ),t(ξ)) の速度ベクトル: (dx/dξ,dt/dξ)、
それぞれの成分間の関係式(条件)が与えられていると見なすべきでは? 「/dξ」は省略されているのです。
実際のところ ξ=t なので
(dx/dξ)/(dt/dξ) = (dx/dt)/1 = dx/dt これなら何の不思議も無いかと。

全微分自体の意味は、パラメータ曲線:(x(ξ),t(ξ)) が与えられたら
値fの変化速度は
df/dξ = (∂f/∂x)dx/dξ + (∂f/∂t) dt/dξ
になるよねーって事の省略記法ととる事が可能です。(厳密にはfに外微分dを作用させた1次微分形式云々になります)
こちらは条件でも何でもありません。
157132人目の素数さん:2013/10/01(火) 09:00:44.87
dx=dx(t)/dt * dtと置くと
(∂f(x(t),t)/∂x) dx-(∂f(x(t),t)/∂t) dt=0
(∂f(x(t),t)/∂x) (x(t)' *dt)-(∂f(x(t),t)/∂t) dt=0

(∂f(x(t),t)/∂x) (x(t)' *dt)=(∂f(x(t),t)/∂t) dt

((∂f(x(t),t)/∂x)(x(t)' *dt))/(∂f(x(t),t)/∂t)= dt

((∂f(x(t),t)/∂x)(x(t)' ))/(∂f(x(t),t)/∂t)= 1

((∂f(x(t),t)/∂x))/(∂f(x(t),t)/∂t)= 1/x(t)'

∂f(x(t),t)/∂t)/((∂f(x(t),t)/∂x))= x(t)'
ということですか・・・
158132人目の素数さん:2013/10/01(火) 09:04:48.60
>>154
意味不明です
159132人目の素数さん:2013/10/01(火) 09:10:06.67
「○○じゃないですか」「●●よね?」は誤解フラグ
160132人目の素数さん:2013/10/01(火) 09:15:58.90
dy=y'*dxが全微分じゃないですか?
それは割り算としての/によってdy/dx=y'と
掛けると同じことですよね。
すなわち全微分可能というのは割り算としての/と微分としての/が
同じ意味を持つことをあらわしているのではないのですか?
161132人目の素数さん:2013/10/01(火) 09:20:10.56
「これでトドメだっ! (どかーーん)」
「やったか?」← やってない
みたいな?
162132人目の素数さん:2013/10/01(火) 09:37:04.67
>>160
全微分の記法が定着したのは、その種のアナロジーが使えて便利だからとも考えられるけど
別に解釈や定義は1つである必要はないんだよ
163132人目の素数さん:2013/10/01(火) 10:14:48.68
>>160
いい加減、教科書の定義を見ろよ。
164132人目の素数さん:2013/10/01(火) 11:10:34.67
dy=y'*dx
一変数の場合はこれだが・・・・
まあテーラー展開の一次の項だとおもえば分りやすいな。
165132人目の素数さん:2013/10/01(火) 11:17:32.66
>>164
何の定義にもなってないだろう
166132人目の素数さん:2013/10/01(火) 11:21:50.25
いや、なってる。
167132人目の素数さん:2013/10/01(火) 11:25:19.10
いや、なっていない。
168132人目の素数さん:2013/10/01(火) 11:29:32.14
165は多変数のテーラー展開を知らない池沼なんだろうな。
悔しかったら定義の1つでも書いてみれば良いのに
頭が悪すぎてかけないんだろうw
169132人目の素数さん:2013/10/01(火) 11:33:17.99
池沼が雲泥
170132人目の素数さん:2013/10/01(火) 11:34:11.85
例えば解析概論にはちゃんと定義から
dx=凅、dy=凉が出ることが書いてあるんだよな
171132人目の素数さん:2013/10/01(火) 11:36:31.71
池沼あh糾える縄の如し
172132人目の素数さん:2013/10/01(火) 11:36:54.42
定義を読もうとすらしないから>>164のような形式的に並べただけの数式を定義だと言い張り
割り算?ナニソレ?状態に陥る
こういう文盲は何しても駄目だな
173132人目の素数さん:2013/10/01(火) 11:38:11.39
馬鹿だネェ・・・
定義は1つじゃ無いんだよ。
杉浦の定義では>>164になってる。
見てみるが良いよ。
174132人目の素数さん:2013/10/01(火) 11:40:19.71
ちなみに解析概論の定義だとdx=凅は正しいけど
dy=凉は正しくないよ。yは独立変数じゃないからね。
175132人目の素数さん:2013/10/01(火) 11:40:49.70
>>173
何ページ、定義の番号は?
176132人目の素数さん:2013/10/01(火) 11:43:01.16
目次から微分を調べろ、
杉浦は全微分=微分だから見つからないのも無理は無い。
177132人目の素数さん:2013/10/01(火) 11:43:52.84
>>176
いいわけはいいから
178132人目の素数さん:2013/10/01(火) 12:23:17.09
もしかして:微分形式というオチ
179132人目の素数さん:2013/10/01(火) 12:32:08.03
少なくとも>>154は解析概論を読んだと嘘ついてたけど
さすがに解決しただろうな。
180132人目の素数さん:2013/10/01(火) 12:36:23.94
すくなくとも164も杉浦を読んだと嘘をついてたけど
さすがに引っ込みがつかないだろうな。
181132人目の素数さん:2013/10/01(火) 12:49:31.31
ライプニッツの記法で検索したら一番目に見つかったのが
http://detail.chiebukuro.yahoo.co.jp/qa/question_detail/q1259319227
なんですけど・・・
要約すると
全微分がdy=y'*dxであると考える人と、
単純に分数で考える人の2通りあるんだって。
そしてこのすれには
dy=y'*dxでないという
第3の選択肢しかないやつがいるというわけだ・・・
182132人目の素数さん:2013/10/01(火) 12:54:40.90
>>181
は?アホ?
183132人目の素数さん:2013/10/01(火) 12:57:30.90
まさに>>172の状態
184132人目の素数さん:2013/10/01(火) 13:32:47.35
2変数の全微分の定義は
df=(∂df(x,y)/∂x)dx+(∂df(x,y)/∂y)dy
なんだけど、
これを一変数にしてたのがdy=y'*dx
ここまで書かないとわからないのか?
185132人目の素数さん:2013/10/01(火) 13:51:05.22
でどうした
186132人目の素数さん:2013/10/01(火) 14:10:50.74
lim(h→0) (f(x)-f(x+h))/h=f(x)'

f(x)-f(x+h)=f'(x)*h+o(h)
の左辺がdfだからf'(x)*h+o(h)がdfのもう1つの定義と言いたいのか?
(∂f(x(t),t)/∂x) dx-(∂f(x(t),t)/∂t) dt=0

(∂f(x(t),t)/∂x) (x(t)' *h+o(h))-(∂f(x(t),t)/∂t) dt=0

(∂f(x(t),t)/∂x) (x(t)' *h+o(h))=(∂f(x(t),t)/∂t) dt

(∂f(x(t),t)/∂x) (x(t)(h/dt)' +o(h)/dt)=(∂f(x(t),t)/∂t)

(x(t)' (h/dt )+o(h)/dt)=(∂f(x(t),t)/∂t) /(∂f(x(t),t)/∂x)

x(t)' =(∂f(x(t),t)/∂t) /(∂f(x(t),t)/∂x) (h→0)
187132人目の素数さん:2013/10/01(火) 14:28:41.33
>>184
その右辺のdxやdyが未定義であるうちは割り算だとかには辿り着かないだろう
よくわからないdxやdyを割り算が定義できるような量ということにするなど
その2変数の全微分だけ持ってきても何も説明できていない事は明か。
188132人目の素数さん:2013/10/01(火) 14:34:36.04
いやいや、右辺のdx,とdtは再帰的に定義されてるだろ、
dfが定義されているわけだから。
189132人目の素数さん:2013/10/01(火) 14:48:34.50
>>188
dfのdはtypoだとして
dx=(∂x/∂x)dx+(∂x/∂y)dy
=1dx+0dy
=dx
だが右辺のdxの定義はどこに?
190132人目の素数さん:2013/10/01(火) 15:16:23.81
dxは変数だからねぇ、定義といわれても任意の実数としかいいようがないねぇ。
191132人目の素数さん:2013/10/01(火) 15:16:38.34
>再帰的に定義されてるだろ

アイタタタwww
定義されてるどころか消えちゃったね( ´,_ゝ`)プッ
192132人目の素数さん:2013/10/01(火) 15:20:42.43
>>190
じゃあ>>188は真っ赤な嘘で
再帰的には定義されていなくて
それとは全く別に、任意の実数をとる変数をdxとしたのか


滅茶苦茶すぎて話にならんなwww
193132人目の素数さん:2013/10/01(火) 15:23:30.21
再帰的定義の場合は基礎の部分は再帰の部分と定義が違うのが普通だろ。
逆に基礎がなくて再帰の部分だけ何ていう奴見たこと無いんですけど・・・・
194132人目の素数さん:2013/10/01(火) 15:26:31.11
df=(∂df(x,y)/∂x)dx+(∂df(x,y)/∂y)dy
dfというのは定数じゃなくて変数dxとdyによって定まる量だからな、
dfはdxとdyの関数でありdxとdyが0に近づくときの変化率を表す量と考える事ができるんだよ。
195132人目の素数さん:2013/10/01(火) 15:29:45.19
>>193
そもそもdxの定義は何かと聞かれたときに
おまえの言う所の基礎の部分で、任意の実数をとる変数と定義されているということは
2変数全微分の定義とか全く無関係だったわけだな。

その再帰の部分とは無関係に定義されてたんだもんな
やられたねぇ、馬鹿は何やっても馬鹿なんだねぇ
196132人目の素数さん:2013/10/01(火) 15:32:24.99
>>195
それはお前がバカなだけ
>>154ではx(t)って書いてあるからxはtの関数だからdx(t)/dt * dtなわけ、
お前が聞くベキなのはdtの定義だったわけだ。
197132人目の素数さん:2013/10/01(火) 15:32:52.47
>>194
完全に>>188の再帰的定義を否定し
dx,とdtはdfよりも先に定義されていたわけだ
何のために>>188のような嘘をついたんだろうな
198132人目の素数さん:2013/10/01(火) 15:37:06.65
再帰的定義っていうのは基礎があるって言ってるだろ。
ちなみにニ変数の全微分でもdxが変数じゃなくて関数の場合もあるからね。
そういう場合は再帰の定義を使い、関数じゃない場合は基礎の定義をつかうんだよ。
199LAT:2013/10/01(火) 15:44:49.51
200132人目の素数さん:2013/10/01(火) 15:50:12.69
どうもおもわんよ
201132人目の素数さん:2013/10/01(火) 16:01:54.87
>>198
dxは任意の実数としかいいようがない(>>190)と言っているよな。
もしこれが本当なら再帰式を持って来る必要は全くなくて
dy/dxはただの実数の割り算で>>154の疑問はそこで終わりだろうな。
202132人目の素数さん:2013/10/01(火) 16:15:25.67
>>199
そこの回答者さんのおっしゃる通りかと思いました
多分の中線定理の一般的な証明中ではベクトル和(と差)のノルムが現れるでしょう...
|u+v|^2 = |u|^2 + |v|^2 + 2u・v
これもう一般化された三平方定理と言えます
証明の途中でその定理を(直接あるいは間接的に)使うのは循環論法、ナンセンスです。
203132人目の素数さん:2013/10/01(火) 16:19:51.77
>>198
はyがxの関数になってるからただの割り算ではないな。
204132人目の素数さん:2013/10/01(火) 16:54:54.87
1/x=10yの形をx=にする場合どういう過程をたどればいいのでしょうか
205132人目の素数さん:2013/10/01(火) 16:59:42.04
>>204
書き込むスレは書き込む内容によって正しく選びましょう
206132人目の素数さん:2013/10/01(火) 17:20:05.77
>>204
1/xに何をすればxになるか考えろ
この場合分母と分子が逆になればxになる
1÷1/xすると分母が逆になるだろ
反対も同じく1÷10y
x=1/10yになる
割る位置に注意な
掛け算と違い1/x÷1じゃ出来ない
中学位の宿題かな?

>>205
こんなのも分からないならレスするのやめなよ^^
207132人目の素数さん:2013/10/01(火) 17:32:31.39
>>205に軍配
まあ>>206は自演などと騒ぐだろうけど、>>205本人には伝わるからそれでいい
208132人目の素数さん:2013/10/01(火) 17:53:07.90
きもちわる(笑)
>>205に伝わるから(笑)
予防線張ってるのも本当気持ち悪い^^;
軍配とかなにと戦ってんだよタコwwwww
209KingMathematician ◆LoZDre77j4i1 :2013/10/01(火) 18:26:03.56
KingMathematician ◆LoZDre77j4i1 に軍配.

Re:>>202 内積の双線形.
210132人目の素数さん:2013/10/01(火) 18:31:31.27
>>206
ようやく回答できる質問がきてよかったね。
て、ここ質問スレじゃないが。
^^^
211狢 ◆yEy4lYsULH68 :2013/10/01(火) 18:40:52.08
>>209


■□■□■□■□■□■□■□■□■□■□■□■□■□■□■□■□■□■□■□■□■□■□■□■□■□■□■□■□■□■□■□■□■□
□■□■□■□■□■□■□■□■□■□■□■□■□■□■□■□■□■□■□■□■□■□■□■□■□■□■□■□■□■□■□■□■□■
■□■□■□■□■□■□■□■□■□■□■□■□■□■□■□■□■□■□■□■□■□■□■□■□■□■□■□■□■□■□■□■□■□
□■□■□■□■□■□■□■□■□■□■□■□■□■□■□■□■□■□■□■□■□■□■□■□■□■□■□■□■□■□■□■□■□■
■□■□■□■□■□■□■□■□■□■□■□■□■□■□■□■□■□■□■□■□■□■□■□■□■□■□■□■□■□■□■□■□■□
□■□■□■□■□■□■□■□■□■□■□■□■□■□■□■□■□■□■□■□■□■□■□■□■□■□■□■□■□■□■□■□■□■
■□■□■□■□■□■□■□■□■□■□■□■□■□■□■□■□■□■□■□■□■□■□■□■□■□■□■□■□■□■□■□■□■□
□■□■□■□■□■□■□■□■□■□■□■□■□■□■□■□■□■□■□■□■□■□■□■□■□■□■□■□■□■□■□■□■□■
■□■□■□■□■□■□■□■□■□■□■□■□■□■□■□■□■□■□■□■□■□■□■□■□■□■□■□■□■□■□■□■□■□
□■□■□■□■□■□■□■□■□■□■□■□■□■□■□■□■□■□■□■□■□■□■□■□■□■□■□■□■□■□■□■□■□■
■□■□■□■□■□■□■□■□■□■□■□■□■□■□■□■□■□■□■□■□■□■□■□■□■□■□■□■□■□■□■□■□■□
□■□■□■□■□■□■□■□■□■□■□■□■□■□■□■□■□■□■□■□■□■□■□■□■□■□■□■□■□■□■□■□■□■
212132人目の素数さん:2013/10/01(火) 18:47:26.52
そう、あくまでもここは分からない問題を書くスレ
213あのこうちやんは始皇帝だった:2013/10/01(火) 19:09:04.28
>>210
 コイツ、30代の、無職の、ゴミ・クズ・カス・無能・虫けらのクソガキ!

 無職のクソガキども!  大変なコトになるな!

憲法改正だ! 96条を改正してから、9条を改正する。 そして、何条を改正するか?
18条だ! そうして、国家総動員法ができて、オマエたち、無職のクソガキどもは、真っ先に徴兵だ!
オマエたちは、頭デッカチの虚弱児・ひ弱だから、最下等兵! すぐ戦死だ!

アハハハハハハハハハハ!!!!!!!!!!!!!!!!!!!!

 死にゆく、クソガキどもに、大伴家持の詩を贈ってやろう!

海行かば 水浸く屍 山行かば 草むす屍 大君の 辺にこそ死なめ かえりみはせじ!
214132人目の素数さん:2013/10/01(火) 19:18:21.23
女子中学生>>204「私のために争わないで......!」
215狢 ◆yEy4lYsULH68 :2013/10/01(火) 20:14:46.68
>>213


■□■□■□■□■□■□■□■□■□■□■□■□■□■□■□■□■□■□■□■□■□■□■□■□■□■□■□■□■□■□■□■□■□
□■□■□■□■□■□■□■□■□■□■□■□■□■□■□■□■□■□■□■□■□■□■□■□■□■□■□■□■□■□■□■□■□■
■□■□■□■□■□■□■□■□■□■□■□■□■□■□■□■□■□■□■□■□■□■□■□■□■□■□■□■□■□■□■□■□■□
□■□■□■□■□■□■□■□■□■□■□■□■□■□■□■□■□■□■□■□■□■□■□■□■□■□■□■□■□■□■□■□■□■
■□■□■□■□■□■□■□■□■□■□■□■□■□■□■□■□■□■□■□■□■□■□■□■□■□■□■□■□■□■□■□■□■□
□■□■□■□■□■□■□■□■□■□■□■□■□■□■□■□■□■□■□■□■□■□■□■□■□■□■□■□■□■□■□■□■□■
■□■□■□■□■□■□■□■□■□■□■□■□■□■□■□■□■□■□■□■□■□■□■□■□■□■□■□■□■□■□■□■□■□
□■□■□■□■□■□■□■□■□■□■□■□■□■□■□■□■□■□■□■□■□■□■□■□■□■□■□■□■□■□■□■□■□■
■□■□■□■□■□■□■□■□■□■□■□■□■□■□■□■□■□■□■□■□■□■□■□■□■□■□■□■□■□■□■□■□■□
□■□■□■□■□■□■□■□■□■□■□■□■□■□■□■□■□■□■□■□■□■□■□■□■□■□■□■□■□■□■□■□■□■
■□■□■□■□■□■□■□■□■□■□■□■□■□■□■□■□■□■□■□■□■□■□■□■□■□■□■□■□■□■□■□■□■□
□■□■□■□■□■□■□■□■□■□■□■□■□■□■□■□■□■□■□■□■□■□■□■□■□■□■□■□■□■□■□■□■□■
216132人目の素数さん:2013/10/01(火) 21:40:05.40
>>204
( ゚д゚)ポカーン 
217132人目の素数さん:2013/10/01(火) 22:29:44.89
http://i.imgur.com/ltsabat.jpg
図1,2の二つの方法で電流と電圧を測定しその測定値から抵抗値を見積もるときa、b…を正しく埋めよ。なを電流計、電圧計の内部抵抗はそれぞれrA,rVで電池の内部抵抗は無視できる。

1図1のような回路においては、電流計のよみI1と電圧計のよみV1から見積もった抵抗は真の抵抗値をRとするとRとrVを用いてV1/I1=aとあらわせる。したがってV1/I1=RとなるためにはrVとRの関係はbとなっていなければならない(記号>>または<<を使って表せ)

2図2のような回路においては、電流計のよみI2と電圧計のよみV2から見積もった抵抗は真の抵抗値をRとするとRとrVを用いてV2/I2=cとあらわせる。したがってV2/I2=RとなるためにはrAとRの関係はdとなっていなければならない(記号>>または<<を使って表せ)

よろしくお願いします
218132人目の素数さん:2013/10/01(火) 22:30:45.47
>>217
板違い。
219132人目の素数さん:2013/10/01(火) 22:33:04.58
な上にマルチ
220132人目の素数さん:2013/10/01(火) 23:54:36.45
>>186
> lim(h→0) (f(x)-f(x+h))/h=f(x)'
221132人目の素数さん:2013/10/01(火) 23:55:52.64
遅れてきた馬鹿
222132人目の素数さん:2013/10/02(水) 00:26:50.21
俺のことは風間くんってよんでくれ
223132人目の素数さん:2013/10/02(水) 01:33:50.37
>>221
無垢な素人をイジルバカ
224132人目の素数さん:2013/10/02(水) 07:32:32.71
>>223
無垢な素人なら
2chなんて危険な掲示板には書き込まずに10年romってろ
225狢 ◆yEy4lYsULH68 :2013/10/02(水) 07:56:46.04


■□■□■□■□■□■□■□■□■□■□■□■□■□■□■□■□■□■□■□■□■□■□■□■□■□■□■□■□■□■□■□■□■□
□■□■□■□■□■□■□■□■□■□■□■□■□■□■□■□■□■□■□■□■□■□■□■□■□■□■□■□■□■□■□■□■□■
■□■□■□■□■□■□■□■□■□■□■□■□■□■□■□■□■□■□■□■□■□■□■□■□■□■□■□■□■□■□■□■□■□
□■□■□■□■□■□■□■□■□■□■□■□■□■□■□■□■□■□■□■□■□■□■□■□■□■□■□■□■□■□■□■□■□■
■□■□■□■□■□■□■□■□■□■□■□■□■□■□■□■□■□■□■□■□■□■□■□■□■□■□■□■□■□■□■□■□■□
□■□■□■□■□■□■□■□■□■□■□■□■□■□■□■□■□■□■□■□■□■□■□■□■□■□■□■□■□■□■□■□■□■
■□■□■□■□■□■□■□■□■□■□■□■□■□■□■□■□■□■□■□■□■□■□■□■□■□■□■□■□■□■□■□■□■□
□■□■□■□■□■□■□■□■□■□■□■□■□■□■□■□■□■□■□■□■□■□■□■□■□■□■□■□■□■□■□■□■□■
■□■□■□■□■□■□■□■□■□■□■□■□■□■□■□■□■□■□■□■□■□■□■□■□■□■□■□■□■□■□■□■□■□
□■□■□■□■□■□■□■□■□■□■□■□■□■□■□■□■□■□■□■□■□■□■□■□■□■□■□■□■□■□■□■□■□■
■□■□■□■□■□■□■□■□■□■□■□■□■□■□■□■□■□■□■□■□■□■□■□■□■□■□■□■□■□■□■□■□■□
□■□■□■□■□■□■□■□■□■□■□■□■□■□■□■□■□■□■□■□■□■□■□■□■□■□■□■□■□■□■□■□■□■
226132人目の素数さん:2013/10/02(水) 08:13:23.48
S≠φ(Sは空集合では無い) と ∃ψ{ψ(S) ∈ S} は同値でしょうか?
だとしたら、空でない集合族: S_λ (λ∈Λ) について
∀S_λ{S_λ≠φ} ⇒ ∀S_λ{∃ψ_λ{ψ_λ(S_λ) ∈ S_λ}}
ここで選択公理に現れる選択関数: f(λ) ∈ S_λ が
f: λ → ψ_λ(S_λ) と定義できてしまう(まだ選択公理を仮定していないのに...)
と思うのですが
最初の前提から間違いなのか、途中で何か論理飛躍があるのでしょうか?
227132人目の素数さん:2013/10/02(水) 08:26:59.25
論理厨が湧く
228132人目の素数さん:2013/10/02(水) 15:07:22.50
ψって何さ?
ψ(S) と書けることからすると写像らしいけれど、
集合Sがそのまま対象なのか、Sの元が対象なのか分からん。

あと、選択公理云々に関しては
ψ_λが存在するからといって具体的に選べるとは限らん。
それこそが選択公理が自明でないポイント。
229132人目の素数さん:2013/10/02(水) 16:48:20.67
>>228
気にしていなかったのですが、
各ψ_λの定義域は{S_λ} (要素がS_λのみの集合)とします
定義域の濃度が1の選択関数と言ってもいいかと

各 S_λ が空で無い事が ψ_λ の存在を意味するなら
定義域はΛ, 終域は U[λ∈Λ] S_λ として
選択関数: f(λ) = ψ_λ(S_λ)
これは構成できた事にならないのでしょうか?
230132人目の素数さん:2013/10/02(水) 18:35:33.37
λ毎の存在と全λの存在は別の事
常識的な推論ではなく形式化された推論での話
231132人目の素数さん:2013/10/02(水) 18:38:46.18
赤、青、黄のボールがあり、
10%の確率で白のボールが追加され、
1%の確率で黒いボールも追加されるとします。最大で五個になります。

この時の赤いボールを選ぶ確率は何%でしょうか?
見えない状態で、ランダムに選択するものとします。

ずっと考えてて頭がこんがらがってきました。考え方も教えて下さい・・・
232132人目の素数さん:2013/10/02(水) 18:54:43.91
うむ、頭がこんがらがっているな。
233132人目の素数さん:2013/10/02(水) 19:08:59.97
>>231
地道に場合わけしれ
234132人目の素数さん:2013/10/02(水) 19:09:07.88
779: 132人目の素数さん [] 2013/10/02(水) 19:03:42.14
cos(aπ)が有理数になる有理数a,sin(bπ)が有理数になる有理数b,tan(cπ)が有理数になる有理数cをそれぞれ全て求めよ
235132人目の素数さん:2013/10/02(水) 19:09:21.13
赤青黄……90/100
赤青黄白……9/100
赤青黄白黒……1/100
という意味なら、赤いボールを選ぶ確率は
(90/100)*(1/3)+(9/100)*(1/4)+(1/100)*(1/5)

別の意味ならそれに合わせた立式が必要
236132人目の素数さん:2013/10/02(水) 19:10:27.94
>>234
Hint: 2cos(aπ)は代数的整数
237132人目の素数さん:2013/10/02(水) 19:20:37.99
>>234
仮に三種の三角関数のどれか一つでも有理数になるのなら、その値を用いて
いわゆる定規とコンパスだけで正分母角形が描ける

よってa, b, cは分母を素因数分解したときに
奇数因子が全てフェルマー素数のどれかであり
なおかつ同じフェルマー素数が2つ以上存在しない場合に限る

こっから先はまだわからん
238132人目の素数さん:2013/10/02(水) 19:24:02.58
>>236
なるほど
これでaは全て求まるのか

というか、もしかして自明な場合しかない?
239132人目の素数さん:2013/10/02(水) 19:34:52.28
>>232-235
ありがとうございます!
240132人目の素数さん:2013/10/02(水) 20:00:51.20
>>230
λ → ψ_λ(S_λ)
これは全λ(∈Λ)に対して対応が取れている事にならないのでしょうか?
すみませんが、もう少し詳しくお願いします。
241132人目の素数さん:2013/10/02(水) 21:41:28.00
条件付き確率の定義が、
P(A|B)=P(A∩B)/P(B)なのですが、この定義式が、すっと入ってきません。
この式さえ出されてもなぜ、「かつ」なのか、なぜ「割り算なのか」、なぜ「P(B)」なのか、分かりません。
教えて下さい。
242132人目の素数さん:2013/10/02(水) 21:53:08.62
俺もそうだよ
243132人目の素数さん:2013/10/02(水) 22:00:01.99
>>241
例えば百分率だったら1が100[%]になるように100で割る
この場合もBが起きることが前提だからP(B)を分母として考える
Bが起きた前提でAが起きるのだからAかつB
Bの中でAかつBの起きる割合なのだから、P(B)でP(AかつB)を割る
244132人目の素数さん:2013/10/02(水) 22:00:30.05
「かつ」は説明しづらい
それぐらいは直観でわかってくれ

全確率は1でなければならないので、定数1/P(B)倍しておくと辻褄が合う
245132人目の素数さん:2013/10/02(水) 22:13:31.10
>>241
教科書にベン図書いてなかった?
246132人目の素数さん:2013/10/02(水) 22:22:11.53
条件付き確率はベン図じゃなくていわゆる面積図で書いてあるんじゃないかな
247132人目の素数さん:2013/10/02(水) 22:46:35.42
>>246
はあ?
248132人目の素数さん:2013/10/02(水) 22:50:07.70
もし高校数学の範囲であれば、個数測度しか扱わないので、ベン図でも面積図でも同じこと
一般には面積図
249132人目の素数さん:2013/10/02(水) 22:54:40.78
>>248
測度論を前提としてるの?
確率空間は何?
250132人目の素数さん:2013/10/02(水) 22:58:21.26
>>243-244
理解しました、ありがとうございました。
251132人目の素数さん:2013/10/02(水) 22:59:13.17
他の皆様もありがとうございます。
252132人目の素数さん:2013/10/02(水) 23:02:11.09
と引っ込むのであった
253132人目の素数さん:2013/10/02(水) 23:16:01.51
縦横それぞれ3×2マスの道がある。
一番左下から一番右上までの最短経路は勿論5C2=10通りである。

では、一度通った道を通らないで一番左下から一番右上まで行く方法は何通りあるか?
計算式と答を両方述べなさい。
ただし、最短経路でなくても良いものとする。
254132人目の素数さん:2013/10/02(水) 23:16:41.98
高校数学に疎いバカが面積図という受験数学用語をだされてファビョッたのか、哀れな。
255132人目の素数さん:2013/10/02(水) 23:38:59.10
「面積図」は、受験算数用語じゃないのか?
256132人目の素数さん:2013/10/02(水) 23:43:38.56
受験用語なの?
単に、紙の上の面積が測度を反映するように描かれた図、くらいの意味だと思ったんだが
257132人目の素数さん:2013/10/02(水) 23:47:55.88
知らんけど線分図の2Dバージョンみたいなものか?
258132人目の素数さん:2013/10/03(木) 00:07:35.86
何言ってんだ。
面積が測度なのは、当然だろ。
259132人目の素数さん:2013/10/03(木) 01:55:11.23
面積図て初耳だな
何の事だか分かるけど
260132人目の素数さん:2013/10/03(木) 02:02:22.30
なら、面積図について教えて下さい!
知らないので
261132人目の素数さん:2013/10/03(木) 02:07:59.19
>>260

↑こういうやつ
262132人目の素数さん:2013/10/03(木) 02:25:15.50
カルノー図?
263132人目の素数さん:2013/10/03(木) 05:13:47.53
S⊆X、x∈Xとする。
S∪S^dは閉集合であることを証明せよ。ただしS^dはSの集積点全体の集合を表す。

よろしくお願いします。
264KingMathematician ◆LoZDre77j4i1 :2013/10/03(木) 05:52:11.67
Re:>>263 何が分からぬ.
265132人目の素数さん:2013/10/03(木) 05:59:30.80
>>263
閉集合の定義はよ
266狢 ◆yEy4lYsULH68 :2013/10/03(木) 05:59:51.22
>>264


■□■□■□■□■□■□■□■□■□■□■□■□■□■□■□■□■□■□■□■□■□■□■□■□■□■□■□■□■□■□■□■□■□
□■□■□■□■□■□■□■□■□■□■□■□■□■□■□■□■□■□■□■□■□■□■□■□■□■□■□■□■□■□■□■□■□■
■□■□■□■□■□■□■□■□■□■□■□■□■□■□■□■□■□■□■□■□■□■□■□■□■□■□■□■□■□■□■□■□■□
□■□■□■□■□■□■□■□■□■□■□■□■□■□■□■□■□■□■□■□■□■□■□■□■□■□■□■□■□■□■□■□■□■
■□■□■□■□■□■□■□■□■□■□■□■□■□■□■□■□■□■□■□■□■□■□■□■□■□■□■□■□■□■□■□■□■□
□■□■□■□■□■□■□■□■□■□■□■□■□■□■□■□■□■□■□■□■□■□■□■□■□■□■□■□■□■□■□■□■□■
■□■□■□■□■□■□■□■□■□■□■□■□■□■□■□■□■□■□■□■□■□■□■□■□■□■□■□■□■□■□■□■□■□
□■□■□■□■□■□■□■□■□■□■□■□■□■□■□■□■□■□■□■□■□■□■□■□■□■□■□■□■□■□■□■□■□■
■□■□■□■□■□■□■□■□■□■□■□■□■□■□■□■□■□■□■□■□■□■□■□■□■□■□■□■□■□■□■□■□■□
□■□■□■□■□■□■□■□■□■□■□■□■□■□■□■□■□■□■□■□■□■□■□■□■□■□■□■□■□■□■□■□■□■
■□■□■□■□■□■□■□■□■□■□■□■□■□■□■□■□■□■□■□■□■□■□■□■□■□■□■□■□■□■□■□■□■□
□■□■□■□■□■□■□■□■□■□■□■□■□■□■□■□■□■□■□■□■□■□■□■□■□■□■□■□■□■□■□■□■□■
267132人目の素数さん:2013/10/03(木) 07:02:04.91
>>263
A = S∪S^d = ((S - Sの孤立点全体) ∪ (Sの孤立点全体)) ∪ S^d(Sの集積点全体)
= (S - Sの孤立点全体) ∪ S^a(Sの触点全体)
= S^a
A^c = (S^a)^c の任意点 x は Sの触点ではない。
定義により Vx ∩ S = φ となる開近傍Vx が存在する。
この時、点 y ∈ Vx ∩ S^d が存在すると仮定すると、
Vx は yの開近傍でもあるので Vx ∩ S ≠ φ となり矛盾となる。
よって Vx ∩ A = φ、つまり A^c の任意点は全て内点(A^cは開集合)であり、
A = S∪S^d は閉集合である。
268132人目の素数さん:2013/10/03(木) 07:09:08.83
248=254
個数測度:測度の意味がわからず使ってみた
面積図:算数用語

以上
269132人目の素数さん:2013/10/03(木) 11:04:48.02
今度消費税が8%に上がりますが、今100円のものを買うと税込105円の物が、税込108円になるんですよね?
その差額3円が、増税分の3%だということはわかるのですが、実際に私たちが払う額を計算してみると、
108/105=1.02857…
となり、約2.86%増しになっているように感じます。
この計算はどこか間違っているのでしょうか?
どうして払う額が3%増しにならないのかわかるように説明していただけると助かります。
270132人目の素数さん:2013/10/03(木) 11:44:01.78
実際、3%増しじゃねーからだよ。
271132人目の素数さん:2013/10/03(木) 11:44:38.11
あなたが正しい。
本来は「税率が3パーセントポイントの増加」というべきで
272132人目の素数さん:2013/10/03(木) 12:12:10.17
底辺6√3、高さが6の直角三角形に内接している円の半径ってどうなりますか?
273132人目の素数さん:2013/10/03(木) 12:19:27.69
>>269
100 + 100*(5%+3%) = 108
あるいは
100 * (1 + 0.05 + 0.03) = 108
この「+0.03」のことを「(本体価格の)3%増し」と言ってる。
つまり、ポイント数は元々の本体価格100に対する値。
### 少なくとも「旧税込み価格の」ではない
274132人目の素数さん:2013/10/03(木) 12:23:59.62
市民の負担が3%も増えるんですよ!そこはどうなんですか、総理!
とかバカな国会答弁が見られそうな悪寒
275132人目の素数さん:2013/10/03(木) 12:30:06.80
>>274
負担は(3/5)*100=60%増しだよな。
276132人目の素数さん:2013/10/03(木) 12:36:30.76
>>272
マルチ
277狢 ◆yEy4lYsULH68 :2013/10/03(木) 16:36:47.17


○●○●○●○●○●○●○●○●○●○●○●○●○●○●○●○●○●○●○●○●○●○●○●○●○●○●○●○●○●○●○●○●○●
●○●○●○●○●○●○●○●○●○●○●○●○●○●○●○●○●○●○●○●○●○●○●○●○●○●○●○●○●○●○●○●○●○
○●○●○●○●○●○●○●○●○●○●○●○●○●○●○●○●○●○●○●○●○●○●○●○●○●○●○●○●○●○●○●○●○●
●○●○●○●○●○●○●○●○●○●○●○●○●○●○●○●○●○●○●○●○●○●○●○●○●○●○●○●○●○●○●○●○●○
○●○●○●○●○●○●○●○●○●○●○●○●○●○●○●○●○●○●○●○●○●○●○●○●○●○●○●○●○●○●○●○●○●
●○●○●○●○●○●○●○●○●○●○●○●○●○●○●○●○●○●○●○●○●○●○●○●○●○●○●○●○●○●○●○●○●○
○●○●○●○●○●○●○●○●○●○●○●○●○●○●○●○●○●○●○●○●○●○●○●○●○●○●○●○●○●○●○●○●○●
●○●○●○●○●○●○●○●○●○●○●○●○●○●○●○●○●○●○●○●○●○●○●○●○●○●○●○●○●○●○●○●○●○
○●○●○●○●○●○●○●○●○●○●○●○●○●○●○●○●○●○●○●○●○●○●○●○●○●○●○●○●○●○●○●○●○●
●○●○●○●○●○●○●○●○●○●○●○●○●○●○●○●○●○●○●○●○●○●○●○●○●○●○●○●○●○●○●○●○●○
○●○●○●○●○●○●○●○●○●○●○●○●○●○●○●○●○●○●○●○●○●○●○●○●○●○●○●○●○●○●○●○●○●
●○●○●○●○●○●○●○●○●○●○●○●○●○●○●○●○●○●○●○●○●○●○●○●○●○●○●○●○●○●○●○●○●○
278132人目の素数さん:2013/10/03(木) 17:03:54.97
>>269
よっかたね、つれて
279狢 ◆yEy4lYsULH68 :2013/10/03(木) 20:35:31.20


○●○●○●○●○●○●○●○●○●○●○●○●○●○●○●○●○●○●○●○●○●○●○●○●○●○●○●○●○●○●○●○●○●
●○●○●○●○●○●○●○●○●○●○●○●○●○●○●○●○●○●○●○●○●○●○●○●○●○●○●○●○●○●○●○●○●○
○●○●○●○●○●○●○●○●○●○●○●○●○●○●○●○●○●○●○●○●○●○●○●○●○●○●○●○●○●○●○●○●○●
●○●○●○●○●○●○●○●○●○●○●○●○●○●○●○●○●○●○●○●○●○●○●○●○●○●○●○●○●○●○●○●○●○
○●○●○●○●○●○●○●○●○●○●○●○●○●○●○●○●○●○●○●○●○●○●○●○●○●○●○●○●○●○●○●○●○●
●○●○●○●○●○●○●○●○●○●○●○●○●○●○●○●○●○●○●○●○●○●○●○●○●○●○●○●○●○●○●○●○●○
○●○●○●○●○●○●○●○●○●○●○●○●○●○●○●○●○●○●○●○●○●○●○●○●○●○●○●○●○●○●○●○●○●
●○●○●○●○●○●○●○●○●○●○●○●○●○●○●○●○●○●○●○●○●○●○●○●○●○●○●○●○●○●○●○●○●○
○●○●○●○●○●○●○●○●○●○●○●○●○●○●○●○●○●○●○●○●○●○●○●○●○●○●○●○●○●○●○●○●○●
●○●○●○●○●○●○●○●○●○●○●○●○●○●○●○●○●○●○●○●○●○●○●○●○●○●○●○●○●○●○●○●○●○
○●○●○●○●○●○●○●○●○●○●○●○●○●○●○●○●○●○●○●○●○●○●○●○●○●○●○●○●○●○●○●○●○●
●○●○●○●○●○●○●○●○●○●○●○●○●○●○●○●○●○●○●○●○●○●○●○●○●○●○●○●○●○●○●○●○●○
280132人目の素数さん:2013/10/03(木) 21:08:18.56
c_n="1からnまでの数を小さい順に並べた数"とする。
例えば c_5=12345 、c_13=12345678910111213 である。

1. ある整数 N が存在して、2<=n<=N について c_n が合成数であれば、n>N についても合成数であることを証明せよ。
2. n>=2 なる全ての c_n が合成数であることを証明するためのプログラムを書け。現実的な実行時間で終了するように工夫すること。
281132人目の素数さん:2013/10/03(木) 21:10:24.74
1.いやだ
2.すれち
282132人目の素数さん:2013/10/04(金) 21:08:08.12
S^{-1}Aが零環⇔0∈S
これはどのようにすれば証明できますか?
自明だそうですが、どの教科書にも証明がなくて困っています。
283132人目の素数さん:2013/10/04(金) 21:45:44.66
0∈S ならば任意のa/s∈S^{-1}A に対しa/s=a*0/s*0=0/0
S^{-1}Aが零環ならば1/1=0/1よりあるs∈Sがあってs(1*1-0*1)=0
284132人目の素数さん:2013/10/04(金) 21:45:52.91
0∈SならS^(-1)A=(0)は自明
S^(-1)A=(0)なら、あるs∈Sが存在して、s1=0 ∴s=0∈S
285132人目の素数さん:2013/10/04(金) 23:10:23.03
この問題がわかりません
優しい方解説お願いします
http://i.imgur.com/RMXlBDw.jpg
286132人目の素数さん:2013/10/04(金) 23:12:22.24
中心角と円周角(だっけ?名前忘れた)
287132人目の素数さん:2013/10/04(金) 23:20:06.64
>>285
算数の範囲なのでここで聞いてください
ttp://uni.2ch.net/test/read.cgi/math/1377905970/
288132人目の素数さん:2013/10/05(土) 01:31:34.75
>>283
>>284
ありがとうございます!
その証明を見て思ったのですが、0/0はS^{-1}Aの任意の元(特に単位元と零元)に等しくなってしまうということでしょうか?
289132人目の素数さん:2013/10/05(土) 05:16:31.66
なかっち 動画
http://www.youtube.com/watch?v=z2qK2lhk9O0s



みんなで選ぶニコ生重大事件 2012
http://vote1.fc2.com/browse/16615334/2/
2012年 ニコ生MVP
http://blog.with2.net/vote/?m=va&id=103374&bm=
2012年ニコ生事件簿ベスト10
http://niconama.doorblog.jp/archives/21097592.html


生放送の配信者がFME切り忘れプライベートを晒す羽目に 放送後に取った行動とは?
http://getnews.jp/archives/227112
FME切り忘れた生主が放送終了後、驚愕の行動
http://niconama.doorblog.jp/archives/9369466.html
台湾誌
http://www.ettoday.net/news/20120625/64810.htm
290132人目の素数さん:2013/10/05(土) 08:00:35.58
>>288
a/b がどういうものかを根本的なところで分っていないような気がする。
291132人目の素数さん:2013/10/05(土) 08:57:01.30
定義を読まない読めない読む気など更々ないんじゃ分かるわけないだろな
292132人目の素数さん:2013/10/05(土) 14:28:59.39
>>253の計算式は未だに見つかっていないんだろ?
むしろ今まで誰も公式を見つけられなかった不思議
293132人目の素数さん:2013/10/05(土) 19:03:24.69
面白味もないしやるひとがいないだけや
294132人目の素数さん:2013/10/05(土) 19:17:31.76
線形行列不等式でちょっとつまずいてます。

行列Aが与えられている時、
  P>0(正定) かつ PA + trace(PA)<0(負定)  を満たす対称行列Pが存在
 ⇔ P>I かつ PA + trace(PA)<0  を満たす対称行列Pが存在

とあるのですが、何故だがよくわかりません…。
教科書には「線形性より明らか」とか書いてないです。
どなかた分かる人いたらお願いします!
295132人目の素数さん:2013/10/05(土) 19:31:40.30
分かるわけねーよ
296132人目の素数さん:2013/10/05(土) 19:41:13.41
でたらめな PA + trace(PA) なんぞ書いてるようじゃな
297132人目の素数さん:2013/10/05(土) 19:44:07.01
>>292
単純に難しいから。やってる人はたくさんいる。計算機屋、数理物理屋、統計屋、etc.。
298132人目の素数さん:2013/10/05(土) 20:08:32.88
>>290
>>291
(ある元どころか)任意のSの元tでt(a*0-0*s)=0なので、0/0=a/sが成立するように感じるのですが…
299132人目の素数さん:2013/10/05(土) 21:04:38.33
a/bって同値類をそう書いているだけなんだけど、わかってる?
300132人目の素数さん:2013/10/05(土) 21:20:52.91
はい!
301132人目の素数さん:2013/10/05(土) 22:43:10.57
>>294
教科書の名前は?
302132人目の素数さん:2013/10/06(日) 01:47:44.69
Q速 - 分からない数学の問題に最速で解説を
http://qsoku.net/
303132人目の素数さん:2013/10/06(日) 02:07:54.23
この板と競合するな……
304132人目の素数さん:2013/10/06(日) 03:18:16.63
>Q速.netテスト版公開しました。
>分からない数学の問題に最速で解説を!
>対象:中学1年〜高校3年までの数学。
住み分け出来ていいじゃないか
30510人に一人はカルトか外国人:2013/10/06(日) 03:26:48.82
★マインドコントロールの手法★

・沢山の人が偏った意見を一貫して支持する
 偏った意見でも、集団の中でその意見が信じられていれば、自分の考え方は間違っているのか、等と思わせる手法

・不利な質問をさせなくしたり、不利な質問には答えない、スルーする
 誰にも質問や反論をさせないことにより、誰もが皆、疑いなど無いんだと信じ込ませる手法


↑マスコミや、カルトのネット工作員がやっていること

TVなどが、偏った思想や考え方に染まっているフリや常識が通じないフリをする人間をよく出演させるのは、
カルトよりキチガイに見える人たちを作ることで批判の矛先をカルトから逸らすことが目的。

リアルでもネットでも、偽装左翼は自分たちの主張に理がないことをわかっているのでまともに議論をしようとしないのが特徴。
306狢 ◆yEy4lYsULH68 :2013/10/06(日) 07:20:54.81
>>305
日本人がマインドコントロールを受け易いその理由は、その文化的背景が『他人を信用する
事を良しとする気質に根ざしてるから』です。実際に何事も疑って掛かる人は非常識という
見方をされる訳です。論理を無視して感情で生きてるから、そういう事になってしまいます。

307132人目の素数さん:2013/10/06(日) 12:36:48.24
>>302
つか、2chで宣伝しなければいけない程
人がいないのか?
淋しいサービスだな()
308132人目の素数さん:2013/10/06(日) 13:41:49.95
あっちは質問スレ
ここは問題を書くスレ
全く重複していないな
309132人目の素数さん:2013/10/06(日) 14:17:01.09
>>307
2chも他で宣伝していた淋しいサービスなんだが
310132人目の素数さん:2013/10/06(日) 14:29:53.38
淋しいんだね
311132人目の素数さん:2013/10/06(日) 16:01:31.58
>>309
2chができた頃はネット全体が淋しいところだったが
312狢 ◆yEy4lYsULH68 :2013/10/06(日) 16:29:06.28
>>311
大丈夫や。この馬鹿板かて今後は寂しい場所になるだけや。どうせアホしか居てへんしナ。

313132人目の素数さん:2013/10/06(日) 16:33:56.82
今後は?
既に十分ボロボロだろwww
314狢 ◆yEy4lYsULH68 :2013/10/06(日) 16:39:02.53
>>313
いいや、まだ馬鹿板として機能してるやろ。そやからもっと潰さなアカンやろナ。

ケケケ狢
315132人目の素数さん:2013/10/06(日) 16:54:06.70
と馬鹿板の主が申しております
316132人目の素数さん:2013/10/06(日) 17:15:41.53
ムジナの生産可能性フロンティアを書け
わかりません、よろしくお願いします
317狢 ◆yEy4lYsULH68 :2013/10/06(日) 17:54:03.97
馬鹿板の崩壊可能性フロンティアを示せ
協力しなさい、よろしくお願いしまっさー

コココ狢
318 ◆uCyT5LJ7l71e :2013/10/06(日) 18:56:35.16
分からない自作問題を記すだけ

以下扱う数は全て非負整数とする
いわゆる剰余を与える記号を % とする。即ち ( am + r ) % m = r ただし 0 ≦ r < m

f( p, c ) = Σ[ n = 1, 2p ]{ { c + Σ[ k = 1, n ] k } % p } と定める
ただし 2 ≦ p、 0 ≦ c < p

このf( p, c )の構造、特に p を固定した時に
f( p, c ) を最大と最小にする c の値の集合がそれぞれどうなるのかがよくわからない
p が 2 の累乗のとき f( p, c ) は c の値によらない定数になりそうではあるが
319132人目の素数さん:2013/10/06(日) 22:25:18.94
疑問があります。分散を求める式は
Σ(X-μ)^2 / n
ですが、分子を二乗しているのはプラスとマイナスの打ち消しを防ぐためでした。

しかし、なぜ分母はn^2ではなくnなんでしょうか?
分子だけ二乗することに納得がいきません。
320132人目の素数さん:2013/10/06(日) 22:28:46.07
分母も2乗して考えると何かウレシイ事があるならそれはそれでいいんじゃないの。
321132人目の素数さん:2013/10/06(日) 22:30:51.68
>>319
X の分散 V[X] は、二乗偏差 (X - E[X])^2 の期待値 E[(X - E[X])^2]。
322132人目の素数さん:2013/10/06(日) 22:31:21.34
>>319
教科書を読みましょう
式の導き、例を理解しましょう
323132人目の素数さん:2013/10/06(日) 22:38:30.73
>>319
たとえばコインの表を 2, 裏を 0 とすると、期待値は μ = 2/2 + 0/2 = 1。
各回の試行の期待値からのずれは (X - μ)^2 = 1 となるから、
  Σ_{i=1,...,n} (X_i - μ)^2/n
 = Σ_{i=1,...,n} 1/n
 = n/n
 = 1
となる。仮に分母が n でなく n^2 だったなら、答えは 1/n となって、試行回数 n を増やしていくほど分散はなくなることになる。
324132人目の素数さん:2013/10/06(日) 22:49:32.14
>分子を二乗しているのはプラスとマイナスの打ち消しを防ぐためでした。
違う。その目的なら二乗じゃなく絶対値でいい。
二乗は適当に決めてるわけじゃなく確率論から来ている。
ガウス分布を勉強しなはれ。

>なぜ分母はn^2ではなくnなんでしょうか?
サンプル1個当たりの値を出したいならnで割るのは当然
325132人目の素数さん:2013/10/06(日) 23:15:57.66
>>323>>324
分母が二乗だと不都合があるんですね。
なんとなくわかりました。ありがとうございます。
326KingMathematician ◆LoZDre77j4i1 :2013/10/06(日) 23:32:23.30
Re:>>306 悪いのは発言が不整合の奴也.
Re:>>319 n^2 と n は n→∞ の時の order が違う.
327132人目の素数さん:2013/10/06(日) 23:41:22.14
分散の分母はnが正しいのかn-1が正しいのかなんて言ってみて余計に引っ掻き回してみたりする。
328狢 ◆yEy4lYsULH68 :2013/10/06(日) 23:45:00.82
>>326


○●○●○●○●○●○●○●○●○●○●○●○●○●○●○●○●○●○●○●○●○●○●○●○●○●○●○●○●○●○●○●○●○●
●○●○●○●○●○●○●○●○●○●○●○●○●○●○●○●○●○●○●○●○●○●○●○●○●○●○●○●○●○●○●○●○●○
○●○●○●○●○●○●○●○●○●○●○●○●○●○●○●○●○●○●○●○●○●○●○●○●○●○●○●○●○●○●○●○●○●
●○●○●○●○●○●○●○●○●○●○●○●○●○●○●○●○●○●○●○●○●○●○●○●○●○●○●○●○●○●○●○●○●○
○●○●○●○●○●○●○●○●○●○●○●○●○●○●○●○●○●○●○●○●○●○●○●○●○●○●○●○●○●○●○●○●○●
●○●○●○●○●○●○●○●○●○●○●○●○●○●○●○●○●○●○●○●○●○●○●○●○●○●○●○●○●○●○●○●○●○
○●○●○●○●○●○●○●○●○●○●○●○●○●○●○●○●○●○●○●○●○●○●○●○●○●○●○●○●○●○●○●○●○●
●○●○●○●○●○●○●○●○●○●○●○●○●○●○●○●○●○●○●○●○●○●○●○●○●○●○●○●○●○●○●○●○●○
○●○●○●○●○●○●○●○●○●○●○●○●○●○●○●○●○●○●○●○●○●○●○●○●○●○●○●○●○●○●○●○●○●
●○●○●○●○●○●○●○●○●○●○●○●○●○●○●○●○●○●○●○●○●○●○●○●○●○●○●○●○●○●○●○●○●○
○●○●○●○●○●○●○●○●○●○●○●○●○●○●○●○●○●○●○●○●○●○●○●○●○●○●○●○●○●○●○●○●○●
●○●○●○●○●○●○●○●○●○●○●○●○●○●○●○●○●○●○●○●○●○●○●○●○●○●○●○●○●○●○●○●○●○
329132人目の素数さん:2013/10/06(日) 23:45:52.92
じゃ僕はせっかくなんでn-2で。
330132人目の素数さん:2013/10/07(月) 00:54:06.56
1/A + 1/B + 1/C + 1/D + 1/E + 1/F + 1/G + 1/H + 1/I + 1/2007 = 1
0 < A < B < C < D < E < F < G < H < I < 2007
A〜Iはすべて自然数.A〜Iを求めよ.


この問題をお願いします.
答えは,
A=2
B=3
C=7
D=72
E=223
F=446
G=669
H=1561
I=1784
だそうですが,どうやって導出すればいいのか分かりません.
331132人目の素数さん:2013/10/07(月) 00:57:14.41
ここはお願いスレではありません
332132人目の素数さん:2013/10/07(月) 01:02:01.68
答えを代入して計算する、話はそれからだ。
333狢 ◆yEy4lYsULH68 :2013/10/07(月) 01:15:35.42
低能を叩いて潰す、話はそれからだ。

コココ狢
334132人目の素数さん:2013/10/07(月) 02:45:11.68
>>329
それもアリだな
335狢 ◆yEy4lYsULH68 :2013/10/07(月) 06:35:27.94
そうなんや、低能は「叩いて潰す」っちゅうんもアリなんだわサ。

ケケケ狢
336KingMathematician ◆LoZDre77j4i1 :2013/10/07(月) 07:20:30.18
Re:>>330 A は 6 より小さい. 2007=9*223 で, 223 は素数だから, 1, 1/2, 1/3, 1/4, 1/5, 1/6, 1/7, 1/8 をいくつか足して 1/9 を足して既約分数にしたときに分子が 223 の倍数になるようにする. これは 256 通り調べればできる.
337132人目の素数さん:2013/10/07(月) 09:04:29.99
相変わらず馬鹿なkingで安心した
338狢 ◆yEy4lYsULH68 :2013/10/07(月) 09:39:55.39
>>336


○●○●○●○●○●○●○●○●○●○●○●○●○●○●○●○●○●○●○●○●○●○●○●○●○●○●○●○●○●○●○●○●○●
●○●○●○●○●○●○●○●○●○●○●○●○●○●○●○●○●○●○●○●○●○●○●○●○●○●○●○●○●○●○●○●○●○
○●○●○●○●○●○●○●○●○●○●○●○●○●○●○●○●○●○●○●○●○●○●○●○●○●○●○●○●○●○●○●○●○●
●○●○●○●○●○●○●○●○●○●○●○●○●○●○●○●○●○●○●○●○●○●○●○●○●○●○●○●○●○●○●○●○●○
○●○●○●○●○●○●○●○●○●○●○●○●○●○●○●○●○●○●○●○●○●○●○●○●○●○●○●○●○●○●○●○●○●
●○●○●○●○●○●○●○●○●○●○●○●○●○●○●○●○●○●○●○●○●○●○●○●○●○●○●○●○●○●○●○●○●○
○●○●○●○●○●○●○●○●○●○●○●○●○●○●○●○●○●○●○●○●○●○●○●○●○●○●○●○●○●○●○●○●○●
●○●○●○●○●○●○●○●○●○●○●○●○●○●○●○●○●○●○●○●○●○●○●○●○●○●○●○●○●○●○●○●○●○
○●○●○●○●○●○●○●○●○●○●○●○●○●○●○●○●○●○●○●○●○●○●○●○●○●○●○●○●○●○●○●○●○●
●○●○●○●○●○●○●○●○●○●○●○●○●○●○●○●○●○●○●○●○●○●○●○●○●○●○●○●○●○●○●○●○●○
○●○●○●○●○●○●○●○●○●○●○●○●○●○●○●○●○●○●○●○●○●○●○●○●○●○●○●○●○●○●○●○●○●
●○●○●○●○●○●○●○●○●○●○●○●○●○●○●○●○●○●○●○●○●○●○●○●○●○●○●○●○●○●○●○●○●○
339132人目の素数さん:2013/10/07(月) 09:53:17.46
1 - 1/2004 から始めて、
残りの和 ≧ 1/n となる最大の 1/n を
順に引いてゆけばいいだけじゃない?
340132人目の素数さん:2013/10/07(月) 10:15:48.54
その手順だと、
 1/2 + 1/3 + 1/7 + 1/43 + 1/18034 + 1/340452052 + 1/142655506996176507 + 1/88185905930790045309716267098281372 + 1/2007 = 1
ってのが得られますね。
341132人目の素数さん:2013/10/07(月) 11:07:36.63
斉次2階線形微分方程式
y"+ay'+by=0 …1
非斉次2階線形微分方程式
y"+ay'+by=Q(x) …2
とすると、1についてもしy1とy2が1の解ならばC1y1+C2y2も1の解であることを示せ
また2についてもしy1とy2が2の解ならばC1y1+C2y2は2の解といえるか?

という問題なのですが、正直全くわかりません。特性方程式を使うのでしょうか?
342132人目の素数さん:2013/10/07(月) 11:24:08.95
(C1y1+C2y2)'' + a (C1y1+C2y2)' + (C1y1+C2y2)
= C1(y1"+ay1'+by1) + C2(y2"+ay2'+by2) = ?
343132人目の素数さん:2013/10/07(月) 11:31:47.69
>>341
斉次の意味がわからない->教科書を読め
非斉次の解がわからない->教科書を読め
344132人目の素数さん:2013/10/07(月) 11:34:35.61
>>342
y1"+ay1'+by1=0になって全部0でQED ですか?
345132人目の素数さん:2013/10/07(月) 11:35:23.83
>>343
斉次って同時ってことですよね?
346132人目の素数さん:2013/10/07(月) 11:52:06.53
今年の単位は諦めろ
347132人目の素数さん:2013/10/07(月) 12:11:50.84
>>345
教科書の名は?
348132人目の素数さん:2013/10/07(月) 12:18:43.83
>>347
教科書ないです
先生が前に問題書いて解いてく
349132人目の素数さん:2013/10/07(月) 12:28:39.61
>>348
どこまでユトリなんだ・・・・
無ければ自分で買うなり図書館で漁るなりすればいいのに。
勉強したくないなら大学なんてさっさとやめちまえ
350132人目の素数さん:2013/10/07(月) 12:33:20.38
大学の先生が教科書を使うと言っていなかったので
教科書持ってません。


そういえば、パンツについても何も言ってなかったので
パンツ履かずに授業受けてます(^^)
351132人目の素数さん:2013/10/07(月) 12:45:09.45
>>348
ノート読めよ
352狢 ◆yEy4lYsULH68 :2013/10/07(月) 12:52:20.21
>>348
ノーと言えや

ケケケ狢
353132人目の素数さん:2013/10/07(月) 12:55:39.34
図書館行ってきます
354132人目の素数さん:2013/10/07(月) 13:01:02.16
ノートもないのか、そりゃわからんだろうな
355132人目の素数さん:2013/10/07(月) 13:07:50.52
356132人目の素数さん:2013/10/07(月) 14:45:50.98
(a^x)'=a^xから始めてa=lim_[n→∞] (1+1/n)^nに至ろうと思ったんですが
(a^x)'=lim_[Δx→0] (a^(x+Δx)-a^x)/Δx
=a^x lim_[Δx→0] (a^Δx-1)/Δx
lim_[Δx→0] (a^Δx-1)/Δx=1
lim_[Δx→0] a^Δx=lim_[Δx→0] 1+Δx
a=lim_[Δx→0] (1+Δx)^(1/Δx)
=lim_[n→∞] (1+1/n)^n
としたら他人に「極限の操作がおかしい」と言われました
その人に聞いても詳しく答えてくれなかったのですが、どの辺がオカシイでしょうか??
357132人目の素数さん:2013/10/07(月) 15:00:39.85
一般に、lim(a/b) ≠ lim(a)/lim(b)
358132人目の素数さん:2013/10/07(月) 15:18:00.16
>>356
lim_[Δx→0] a^Δx=lim_[Δx→0] (1+Δx)
は任意のa>0について成り立ってしまう
359132人目の素数さん:2013/10/07(月) 15:26:02.14
>>356
>(a^x)'=a^x
違うだろう?
360132人目の素数さん:2013/10/07(月) 15:31:07.19
仮定だろ
361132人目の素数さん:2013/10/07(月) 15:51:19.94
なるほど
よほど細かくやらないとトートロジーになりそうだな
362132人目の素数さん:2013/10/07(月) 16:02:28.19
  (a^x)' = ( e^(x*log(a)) )' = ( e^(x*log(a)) )*log(a) = (a^x)*log(a)
∴log(a) = 1
∴a = e = lim_[n→∞] (1+1/n)^n

これでいいだろ
363132人目の素数さん:2013/10/07(月) 16:04:28.63
ぷぷぷ
364132人目の素数さん:2013/10/07(月) 17:15:16.67
ちょっと横路だが
差分化すると
{a^(x+h)-a^x}/h=a^x
a^h=1+h
a=(1+h)^(1/h)→e(h→0)
365132人目の素数さん:2013/10/07(月) 17:46:58.04
a^h = 1 + h + o(h^2)
a = { 1 + h + o(h^2) }^(1/h)
= { (1 + H)^(1/H) }^(H/h)
→ e^1 = e (h→0)

途中で、H = h + o(h^2) と置いた
H → 0 (h→0)
H/h = 1 + o(h^2)/h → 1 (h→0)
366132人目の素数さん:2013/10/07(月) 18:43:58.05
【三辺の長さのいずれもが1をこえない三角形は半径1/√3の円に含められることを示せ。】

わからないというか、他の人がどう解くのか知りたくて.........
367132人目の素数さん:2013/10/07(月) 18:54:59.59
1が2√3
1/√3が2

の場合考えれば、すぐできるだろw
368132人目の素数さん:2013/10/07(月) 19:09:34.58
>>366
最長辺に正弦定理
369367:2013/10/07(月) 19:39:57.40
>>368過保護すぎねーかw
370KingMathematician ◆LoZDre77j4i1 :2013/10/07(月) 21:03:07.99
Re:>>337 そう思うなら何をしに来た.
Re:>>350 立ち続けるがよかろう.
371狢 ◆yEy4lYsULH68 :2013/10/07(月) 21:05:15.69
>>370


○●○●○●○●○●○●○●○●○●○●○●○●○●○●○●○●○●○●○●○●○●○●○●○●○●○●○●○●○●○●○●○●○●
●○●○●○●○●○●○●○●○●○●○●○●○●○●○●○●○●○●○●○●○●○●○●○●○●○●○●○●○●○●○●○●○●○
○●○●○●○●○●○●○●○●○●○●○●○●○●○●○●○●○●○●○●○●○●○●○●○●○●○●○●○●○●○●○●○●○●
●○●○●○●○●○●○●○●○●○●○●○●○●○●○●○●○●○●○●○●○●○●○●○●○●○●○●○●○●○●○●○●○●○
○●○●○●○●○●○●○●○●○●○●○●○●○●○●○●○●○●○●○●○●○●○●○●○●○●○●○●○●○●○●○●○●○●
●○●○●○●○●○●○●○●○●○●○●○●○●○●○●○●○●○●○●○●○●○●○●○●○●○●○●○●○●○●○●○●○●○
○●○●○●○●○●○●○●○●○●○●○●○●○●○●○●○●○●○●○●○●○●○●○●○●○●○●○●○●○●○●○●○●○●
●○●○●○●○●○●○●○●○●○●○●○●○●○●○●○●○●○●○●○●○●○●○●○●○●○●○●○●○●○●○●○●○●○
○●○●○●○●○●○●○●○●○●○●○●○●○●○●○●○●○●○●○●○●○●○●○●○●○●○●○●○●○●○●○●○●○●
●○●○●○●○●○●○●○●○●○●○●○●○●○●○●○●○●○●○●○●○●○●○●○●○●○●○●○●○●○●○●○●○●○
○●○●○●○●○●○●○●○●○●○●○●○●○●○●○●○●○●○●○●○●○●○●○●○●○●○●○●○●○●○●○●○●○●
●○●○●○●○●○●○●○●○●○●○●○●○●○●○●○●○●○●○●○●○●○●○●○●○●○●○●○●○●○●○●○●○●○
372132人目の素数さん:2013/10/07(月) 23:48:19.43
多くの数学書で実数体(NやZなども)を\mathbb{R}ではなく他の表記にしているのはなぜなのでしょうか?
373132人目の素数さん:2013/10/08(火) 09:36:37.01
>>372
本来、黒板ボールド体は名前の通り黒板などの手書きで太字を表すための便宜的な字体であって
ちゃんと太字を用意できる活字本で黒板ボールド体を使う必要がないから。
昔よくあった手書きの印刷物ならともかく、それを活字本で使う方がおかしい。
見慣れているということで使われるようになってきているというだけ。
374132人目の素数さん:2013/10/08(火) 10:35:43.45
http:
//m.imgur.com/KKbiOUn

逆さまになってごめなさい。

Σの計算方法でこのような置換計算はでのようにやるのでしょうか。
初歩的な問題ですみません。
375132人目の素数さん:2013/10/08(火) 11:27:00.76
>>374
置換じゃないいよ
376132人目の素数さん:2013/10/08(火) 11:37:27.37
>>375
lって何ですか?
377132人目の素数さん:2013/10/08(火) 12:18:09.78
>>376
しらんがなー、エスパーじゃあるまいし
378132人目の素数さん:2013/10/08(火) 12:26:26.58
>>374
明示的にいうと、等式が成立しない
379132人目の素数さん:2013/10/08(火) 13:27:10.62
>>374
3k+2-3m=3(k-m+1)-1だから
l=k-m+1
380132人目の素数さん:2013/10/08(火) 16:26:16.28
教えてえらい人

何重にもなっている指数表記,例えば3^3^3は(3^3)^3, それとも3^(3^3), どちらを意味するの?
381132人目の素数さん:2013/10/08(火) 16:28:17.51
タワー使え。通常は後者。指数は上から下。
382132人目の素数さん:2013/10/08(火) 16:29:02.35
零化イデアルとテンソル積の間に何か関係式は成立しますか?
例えばAnn(A×B)=Ann(A)×Ann(B)のような。
383132人目の素数さん:2013/10/08(火) 16:30:39.80
また暗記厨か
384132人目の素数さん:2013/10/08(火) 16:35:08.14
>>381
タワーてなに?
385132人目の素数さん:2013/10/08(火) 16:47:47.96
(3^3)^3=3^(3×3) に使う必要は無い
386132人目の素数さん:2013/10/08(火) 16:55:25.39
なるほど
387132人目の素数さん:2013/10/08(火) 17:56:35.69
括弧無しは、タワーの方だって。
388132人目の素数さん:2013/10/08(火) 18:49:26.74
>>380
むしろ、どっちの意味で使ってるのかと、お前(書いたやつ)が訊かれることも多いと思う。
389狢 ◆yEy4lYsULH68 :2013/10/08(火) 19:59:07.28


○●○●○●○●○●○●○●○●○●○●○●○●○●○●○●○●○●○●○●○●○●○●○●○●○●○●○●○●○●○●○●○●○●
●○●○●○●○●○●○●○●○●○●○●○●○●○●○●○●○●○●○●○●○●○●○●○●○●○●○●○●○●○●○●○●○●○
○●○●○●○●○●○●○●○●○●○●○●○●○●○●○●○●○●○●○●○●○●○●○●○●○●○●○●○●○●○●○●○●○●
●○●○●○●○●○●○●○●○●○●○●○●○●○●○●○●○●○●○●○●○●○●○●○●○●○●○●○●○●○●○●○●○●○
○●○●○●○●○●○●○●○●○●○●○●○●○●○●○●○●○●○●○●○●○●○●○●○●○●○●○●○●○●○●○●○●○●
●○●○●○●○●○●○●○●○●○●○●○●○●○●○●○●○●○●○●○●○●○●○●○●○●○●○●○●○●○●○●○●○●○
○●○●○●○●○●○●○●○●○●○●○●○●○●○●○●○●○●○●○●○●○●○●○●○●○●○●○●○●○●○●○●○●○●
●○●○●○●○●○●○●○●○●○●○●○●○●○●○●○●○●○●○●○●○●○●○●○●○●○●○●○●○●○●○●○●○●○
○●○●○●○●○●○●○●○●○●○●○●○●○●○●○●○●○●○●○●○●○●○●○●○●○●○●○●○●○●○●○●○●○●
●○●○●○●○●○●○●○●○●○●○●○●○●○●○●○●○●○●○●○●○●○●○●○●○●○●○●○●○●○●○●○●○●○
○●○●○●○●○●○●○●○●○●○●○●○●○●○●○●○●○●○●○●○●○●○●○●○●○●○●○●○●○●○●○●○●○●
●○●○●○●○●○●○●○●○●○●○●○●○●○●○●○●○●○●○●○●○●○●○●○●○●○●○●○●○●○●○●○●○●○
390狢 ◆yEy4lYsULH68 :2013/10/08(火) 20:56:20.16
アゲ狢
391132人目の素数さん:2013/10/08(火) 21:06:49.15
12x66の変形オセロを考える
最終局面を狢パターン>>389にすることは可能か?
392132人目の素数さん:2013/10/08(火) 23:13:48.14
不可能
393132人目の素数さん:2013/10/09(水) 09:39:23.41
最後の一手はどこやねん?
394132人目の素数さん:2013/10/09(水) 09:50:00.27
f(x+y)=f(x)f(y)+f(x)+f(y)+(xy)^2。
395132人目の素数さん:2013/10/09(水) 14:04:32.40
http://i.imgur.com/imGqQUz.jpg

これわかるやついる??
396132人目の素数さん:2013/10/09(水) 14:09:24.51
>>395
いるよ
397132人目の素数さん:2013/10/09(水) 14:14:23.38
>>396
おねがいします
おしえてください
398132人目の素数さん:2013/10/09(水) 14:51:46.08
ここはおねがいスレではありません
399132人目の素数さん:2013/10/09(水) 14:53:30.42
わからないので教えてください。

xy平面上で与えられたn≧3において、頂点が全て格子点であり、
さらにどの2つの頂点を結ぶ線分の長さも全て整数であるような凸n角形
が存在することを示せ。
400132人目の素数さん:2013/10/09(水) 14:54:39.28
ここは教えてスレではありません
401132人目の素数さん:2013/10/09(水) 15:17:01.48
ピタゴラス数くらい知ってるだろ
402132人目の素数さん:2013/10/09(水) 15:22:49.68
>>395
直交切線なんて知らん
403132人目の素数さん:2013/10/09(水) 15:26:15.80
>>397
いくら?
404132人目の素数さん:2013/10/09(水) 15:46:56.54
>>395
1 は題意を数式に翻訳整理すると同次形の方程式が得られる
405132人目の素数さん:2013/10/09(水) 16:16:30.87
>>395
2 は「直交截線」も調べると演習書にそのまんまの問題が出ている
406132人目の素数さん:2013/10/09(水) 16:36:19.88
本当に素人の質問なので、
出来る限り分かりやすいアドバイスお願いします

X^5ー1の解を求める問題のようですが
cos72とか関係あるらしいのですが、お願いします。
407132人目の素数さん:2013/10/09(水) 16:55:32.51
Kをバナッハ空間の中の閉凸集合とします
fをKで定義された弱半連続かつ凸な汎関数とします。x0をKの点とします。

任意の正の数εに対してある正の数δが存在して
||xーx0||<δならば、f(x0)<f(x)+ε
が成り立つとします。
このとき
ー∞<inf{f(x)|x∈K}
が成り立つみたいなのですが、何故ですか。
408132人目の素数さん:2013/10/09(水) 17:07:28.19
>>406
問題を正確に写せ。
409132人目の素数さん:2013/10/09(水) 17:08:29.98
ヒント:釣り
410132人目の素数さん:2013/10/09(水) 17:18:19.75
>>408
資料はないけど
この方程式を解けです。確か
72を5つ足すと360度になるから、なんか関係してるか、その程度です
5つ答えを書けばいいですかね?
見つからない課題なんんですが
411132人目の素数さん:2013/10/09(水) 17:27:28.73
>>410
教科書読め
新課程の高校数学V
教科書の入手が困難なら数研から出ている『体系数学5』 この本の24ページ以降を見ろ
412132人目の素数さん:2013/10/09(水) 17:45:16.46
>>406
>X^5ー1
等号を含まない式は方程式ではない
413132人目の素数さん:2013/10/09(水) 17:48:48.30
X^5ー1 =0です
これを解けといってます
ヒントとして72の角度が与えられました。
マジに信じられないくらい、わかんないんです。
414132人目の素数さん:2013/10/09(水) 17:50:05.70
>>413
x^5=1 でぐぐれ
415132人目の素数さん:2013/10/09(水) 17:56:47.80
どうも理解は出来てませんが
吟味していけば、理解できそうな資料がありました。
さんきゅーでした。
416132人目の素数さん:2013/10/09(水) 18:16:59.75
X=1だろ
417あのこうちやんは始皇帝だった:2013/10/09(水) 18:44:07.21
>>416
コイツ、30代の、無職の、女性恐怖症の、ゴミ・クズ・カス・無能・虫けらのクソガキ!

 無職のクソガキども!  大変なコトになるな!

憲法改正だ! 96条を改正してから、9条を改正する。 そして、何条を改正するか?
18条だ! そうして、国家総動員法ができて、オマエたち、無職のクソガキどもは、真っ先に徴兵だ!
オマエたちは、頭デッカチの虚弱児・ひ弱だから、最下等兵! すぐ戦死だ!

アハハハハハハハハハハ!!!!!!!!!!!!!!!!!!!!

 死にゆく、クソガキどもに、大伴家持の詩を贈ってやろう!

海行かば 水浸く屍 山行かば 草むす屍 大君の 辺にこそ死なめ かえりみはせじ!
418132人目の素数さん:2013/10/09(水) 20:46:10.38
高三の姪っ子に頼まれたのですが
最小値が0と言うのは解るのですが、最大値の求め方がわかりません。

x^2+y^2≦1,x≧0,y≧0のとき、y+2xの最大値と最小値を求めよ。

どなたか解説付きでお願いします。
419132人目の素数さん:2013/10/09(水) 20:47:46.28
>>413
1^5-1=1-1=0 なので解の一つは1であることがわかる。
次に、因数定理によってx^5-1=0の左辺はx-1で割り切れることが分るので
実際に割り算を実行して xの4次方程式が得られる。

まず、この4次方程式を求めよ。
話はそれからだ。
420132人目の素数さん:2013/10/09(水) 20:49:56.97
>>418
直線が円に接するとき
421132人目の素数さん:2013/10/09(水) 20:50:02.89
>>418
y+2x = k とおいてこれを直線の方程式と見る
この直線を上下に動かす
与不等式の表す領域と共有点をもつときで
y切片 k が最大最小になるときを捉える
422132人目の素数さん:2013/10/09(水) 20:52:21.68
>>418
x^2+y^2≦0、x≧0、y≧0が示す領域を図示せよ。
y+2x=mとおいて、直線 y=-2x+m がこの領域と交わるmの値の範囲を求める。
そこから最小値と最大値が求まる。(最小値が0となるのは、領域内の点(0,0)を通るときであることは分るだろう?)
直線 y=-2x+m を色々に動かしてみよ。
423狢 ◆yEy4lYsULH68 :2013/10/09(水) 20:54:57.33
>>417


○●○●○●○●○●○●○●○●○●○●○●○●○●○●○●○●○●○●○●○●○●○●○●○●○●○●○●○●○●○●○●○●○●
●○●○●○●○●○●○●○●○●○●○●○●○●○●○●○●○●○●○●○●○●○●○●○●○●○●○●○●○●○●○●○●○●○
○●○●○●○●○●○●○●○●○●○●○●○●○●○●○●○●○●○●○●○●○●○●○●○●○●○●○●○●○●○●○●○●○●
●○●○●○●○●○●○●○●○●○●○●○●○●○●○●○●○●○●○●○●○●○●○●○●○●○●○●○●○●○●○●○●○●○
○●○●○●○●○●○●○●○●○●○●○●○●○●○●○●○●○●○●○●○●○●○●○●○●○●○●○●○●○●○●○●○●○●
●○●○●○●○●○●○●○●○●○●○●○●○●○●○●○●○●○●○●○●○●○●○●○●○●○●○●○●○●○●○●○●○●○
○●○●○●○●○●○●○●○●○●○●○●○●○●○●○●○●○●○●○●○●○●○●○●○●○●○●○●○●○●○●○●○●○●
●○●○●○●○●○●○●○●○●○●○●○●○●○●○●○●○●○●○●○●○●○●○●○●○●○●○●○●○●○●○●○●○●○
○●○●○●○●○●○●○●○●○●○●○●○●○●○●○●○●○●○●○●○●○●○●○●○●○●○●○●○●○●○●○●○●○●
●○●○●○●○●○●○●○●○●○●○●○●○●○●○●○●○●○●○●○●○●○●○●○●○●○●○●○●○●○●○●○●○●○
○●○●○●○●○●○●○●○●○●○●○●○●○●○●○●○●○●○●○●○●○●○●○●○●○●○●○●○●○●○●○●○●○●
●○●○●○●○●○●○●○●○●○●○●○●○●○●○●○●○●○●○●○●○●○●○●○●○●○●○●○●○●○●○●○●○●○
424420:2013/10/09(水) 20:57:58.72
>>420
円と直線y=-2(x-1)の交点は、そこで直行するy=(1/2)x上にあるので二つの直線との交点を求める
425132人目の素数さん:2013/10/09(水) 21:38:57.13
最大値は、2x+yより、√(2^2+1)=√5。
426132人目の素数さん:2013/10/09(水) 21:40:37.70
皆さん有難うございます!理解できました!
張り切って姪に説明してきます!
ホントに有難うございました。
427132人目の素数さん:2013/10/09(水) 21:41:46.72
jk大勝利
428132人目の素数さん:2013/10/09(水) 21:46:49.82
72×5=360
複素数平面上に単位円描いて5等分しろ
429132人目の素数さん:2013/10/09(水) 21:48:01.82
姪 I ask you ?
ってやつだ。
430132人目の素数さん:2013/10/10(木) 01:01:15.17
ペル方程式X^2-2Y^2=1 (X,Yは整数)の解を生み出す漸化式は
上式と、点(1,0)を通り傾きm(mは有理数)の直線との
交点が有利点であることを考えると、
Xn+1=Xn^2+2Yn^2
Yn+1=2XnYn
と導ける

とあるのですが、導く過程が省かれていて全く理解できません。
どなたかよろしくお願い致します
431132人目の素数さん:2013/10/10(木) 01:28:25.12
>>430
その交点の座標を求めてはみたのかい?
432132人目の素数さん:2013/10/10(木) 03:00:10.14
1m^3の球@は質量は19.3*10^3kgで,1mm^3の球Aは似質量は8.93mgである。天秤で半径20mmの
Aと釣り合う@の半径を求めよ。
433132人目の素数さん:2013/10/10(木) 03:25:35.64
[ [ {1(m^3)/1(mm^3)} * [8.93(mg)/{19.3*10^3(kg)}] ] ^(1/3) ] * { 20(mm) }
434420:2013/10/10(木) 10:24:01.85
>>424
訂正:接線は(2x+y)/√5=1
435132人目の素数さん:2013/10/10(木) 10:29:37.74
>>379
サンクス
436132人目の素数さん:2013/10/10(木) 11:02:49.75
エスパー10級でした
437132人目の素数さん:2013/10/10(木) 11:43:05.49
よくある円錐に紐を回した時の最短経路の問題の発展なんだけど
円錐の展開図を書いた時にその中心角が180度以上だったらどうなりますか?
こんな感じの図です↓汚くてすみません
http://beebee2see.appspot.com/i/azuYlKmbCQw.jpg
438132人目の素数さん:2013/10/10(木) 11:52:24.37
>>437
最短経路(候補)と交わらない母線で切り開く
439132人目の素数さん:2013/10/10(木) 12:00:55.51
ぱっくまんふぃーばー
440132人目の素数さん:2013/10/10(木) 13:11:49.25
>>437
一周して元に戻るということなら
頂上に上って、小さく一周して下りてくるのが最短。
回る円をいくらでも小さくできるから最小値は存在しないが
頂上までを往復する距離が下限
441437:2013/10/10(木) 16:48:26.21
>>438-440
ありがとうございます
442132人目の素数さん:2013/10/10(木) 19:13:33.17
http://i.imgur.com/Z89t0ad.jpg

妹の中学受験塾の算数の問題がわかりません。
(4)の問題です。
443132人目の素数さん:2013/10/10(木) 19:19:01.47
444132人目の素数さん:2013/10/10(木) 19:27:06.22
>>442
相似な3つの三角形の面積比を出せ

相似比の2乗がそれだから平方根とればわかる
445132人目の素数さん:2013/10/10(木) 19:54:05.30
行列の要素の中に行列が入っている時ってどう処理するんですか?
446132人目の素数さん:2013/10/10(木) 19:56:49.73
>>445
大きい行列の中身を便宜上区切って書いてあるのとは違うの?
447132人目の素数さん:2013/10/10(木) 19:59:25.80
>>445
途中で送信してしまいました
今大学で 計量経済学をやっているんですが
行列を使うのがイマイチピンときません

計算の途中で行列の中に行列が出てきてしまうのです
色々調べてみても書いてないので教えていただきたいです
448132人目の素数さん:2013/10/10(木) 20:15:09.61
>>447
画像あげてみ
449132人目の素数さん:2013/10/10(木) 20:51:14.27
20131010_204047.jpg
20131010_204114.jpg
20131010_204506.jpg
450132人目の素数さん:2013/10/10(木) 20:56:54.24
pgr
451132人目の素数さん:2013/10/10(木) 20:57:20.16
どこや
452132人目の素数さん:2013/10/10(木) 20:58:40.26
https://www.google.com/search?q=%2220131010_204047.jpg%22
"20131010_204047.jpg" に一致する情報は見つかりませんでした。

検索のヒント:
キーワードに誤字・脱字がないか確認します。
別のキーワードを試してみます。
もっと一般的なキーワードに変えてみます。
453132人目の素数さん:2013/10/10(木) 21:01:03.19
すみません遅くなりました

http://uploda.cc/img/img525694fe7dbd4.jpg

http://uploda.cc/img/img52569636628ac.jpg

http://uploda.cc/img/img5256968616c56.jpg

この最初の写真の定義で
この問題の4をといたら
行列のなかにxが入ってしまいました
454狢 ◆yEy4lYsULH68 :2013/10/10(木) 21:05:00.96
舐めるんじゃねえ。この低能め。



>550 名前:仙谷60 :2013/10/10(木) 20:53:12.86
> >>549
>で?
>阿呆は黙ってろ
>
455132人目の素数さん:2013/10/10(木) 21:05:12.64
ただのブロック分け
解散
456132人目の素数さん:2013/10/10(木) 21:15:29.90
>>453
(4)なら、3x2行列、2x2行列、2x3行列の積だから
答えは3x3行列でいいんじゃないの。
457132人目の素数さん:2013/10/10(木) 21:26:21.44
>>455 456 さん
ブロック分けというものを検索してみます

そういう見方もできるのですね

ついでにもう一つお伺いしたいです
理系を専攻されている人にとって経済学は似非科学だと思われるのでしょうか?
458132人目の素数さん:2013/10/10(木) 21:27:36.26
>>457
かならずしもそうおもわないけど、byマンキュー
459132人目の素数さん:2013/10/10(木) 21:42:09.19
>>457
徹頭徹尾、仮定を明確に提示して論を進める限り似非科学とは言われない。
460458:2013/10/10(木) 21:48:52.94
>>457
常に「トレードオフがある」だから真実は一つではないわな
461132人目の素数さん:2013/10/10(木) 22:04:33.95
>>459
数学なら現実の現象との整合性を気にする必要がないから、その言い分も通らなくはないんだけどね
462132人目の素数さん:2013/10/10(木) 22:09:16.75
>>461
ほら、読み間違えている。
463132人目の素数さん:2013/10/10(木) 22:13:41.08
>>462
「経済理論」ではなく「単なる数学遊び」って意味だよ
464458:2013/10/10(木) 22:15:48.86
>>457
データがたとえ大本営発表でもそれしかなければ使わざるをえない
465132人目の素数さん:2013/10/10(木) 22:18:43.07
違うよ。
似非科学と呼ばれるのは、どこかに論理的な説明を拒む要素が潜んでいるから、ということ。
466132人目の素数さん:2013/10/10(木) 22:19:49.18
>>465>>463へのレスね。
467132人目の素数さん:2013/10/10(木) 22:24:21.43
論理的でさえあれば、どんな無茶な仮定でもいい
なんてのは数学でさえ歓迎されない態度だよ
だから「単なる数学遊び」
経済学の名に値しない
468132人目の素数さん:2013/10/10(木) 23:01:23.97
また読み間違えている
469132人目の素数さん:2013/10/11(金) 00:11:01.77
ド・ラーム「コホモロジー」、ガロア「コホモロジー」、層の「コホモロジー」、エタール「コホモロジー」と、数学ではホモロジーよりもコホモロジーの方が出番が多い気がするのですが、なぜでしょうか?
470132人目の素数さん:2013/10/11(金) 00:17:07.38
>>399解けるヤツいないの?クズばっかだなここ
471132人目の素数さん:2013/10/11(金) 00:25:09.10
ここ問題解くスレじゃないし
472132人目の素数さん:2013/10/11(金) 00:27:38.98
>>470
ここは取り下げて、解いてスレにでも行ったら
473132人目の素数さん:2013/10/11(金) 00:28:29.41
>>469
なかなかいい質問だ。
実際、コホモロジー代数とは言わない。
ホモトピーは出てくるがコホモトピーは
殆ど出てこない。何故か?
474132人目の素数さん:2013/10/11(金) 00:30:08.83
475132人目の素数さん:2013/10/11(金) 00:30:10.10
>>470
凹2m角形に直角三角形をはめ込んで凸n角形作るだけじゃ
476132人目の素数さん:2013/10/11(金) 01:25:44.65
>>473
質問に質問で(ry
477132人目の素数さん:2013/10/11(金) 04:50:59.08
>>469 "空間"そのものより,その上の"関数たち"のほうが人間に扱いやすいからだろう
 その場合,当然関手はひっくり返る.
478132人目の素数さん:2013/10/11(金) 06:25:50.92
>>470
「クズ」に聞くおまえはクズ以下でOK?
479132人目の素数さん:2013/10/11(金) 13:47:02.79
U,V〜U(0,1) (互いに独立な確率変数)
X=max(U,V)
Y=min(U,V)
とするとき、XとYの同時確率分布関数G(x,y)を求めよ

誰かお願いします
480132人目の素数さん:2013/10/11(金) 15:52:35.02
P(X≦x∧Y≦y)=P(U≦x∧V≦x∧(U≦y∨V≦y))
x≦y なら
P(X≦x∧Y≦y)=P(U≦y∧V≦y)+P(U≦y∧y<V≦x)+P(y<U≦x∧V≦y)
=P(U≦y)P(V≦y)+P(U≦y)P(y<V≦x)+P(y<U≦x)P(V≦y)
481132人目の素数さん:2013/10/11(金) 15:54:23.30
おっとy<xだった
482132人目の素数さん:2013/10/11(金) 16:34:41.32
そんな難しく考えなくても...、3D図を脳内作図して水平面でスライスしてみればいいじゃん

G(X)=k*X
∫[X=0,1] G(X)dX = 1 より k=2

G(Y|X) = k' * 1
∫[Y=0,X] G(Y|X)dX = 1 より k'=1/X

よって (X,Y) ∈ { (X,Y) | X≧Y } では
G(X,Y) = G(X)*G(Y|X) = 2
他では、
G(X,Y) = 0
483132人目の素数さん:2013/10/11(金) 20:01:33.65
>>480
これで理解出来ました!
ありがとうございます。
484132人目の素数さん:2013/10/11(金) 20:08:35.03
わかったんかい、すごいね
485132人目の素数さん:2013/10/11(金) 20:44:56.39
少しだけ方針が変わってしまいましたが

G(x,y) = P(max(X,Y)≦x , min(X,Y)≦y))
= P( (U≦x , V≦x)∨(U≦y , V≦y) )
= P( (U≦x , V≦x , U≦y)∨(U≦x , V≦x , V≦y) )
= P( (U≦y , V≦x)∨(U≦y , V≦x) ) (∵y≦x)
= xy + xy - y^2 ( ∵P(A∨B) = P(A)+P(B)-P(A∧B) )
= 2xy - y^2

こう解きました。
486132人目の素数さん:2013/10/11(金) 21:06:42.72
ガウス型分布は関係ないの?
487132人目の素数さん:2013/10/11(金) 21:14:02.75
U(0,1) のことでしたら一様分布のつもりで記述しました
それは xy+xy-x^2 のところで使いました
488132人目の素数さん:2013/10/11(金) 21:19:01.94
あーそー
489132人目の素数さん:2013/10/11(金) 21:21:41.06
レスありがとうございました
490132人目の素数さん:2013/10/11(金) 21:45:47.85
>>482 は確率密度関数の勘違いね
491132人目の素数さん:2013/10/11(金) 23:28:00.21
√i を求めたいのですが、高校数学の知識で求められますか?
492132人目の素数さん:2013/10/11(金) 23:46:06.50
>>491
√iという表記はよくない。

(a+bi)^2=i (a,bは実数)とすると
(a^2-b^2)+(2ab-1)i=0
a^2-b^2, 2ab-1はともに実数だから
a^2-b^2=0かつ2ab-1=0

これを解くと、
(a,b)=(±1/√2,±1/√2)
よって2乗してiとなる数は±(1+i)/√2
493132人目の素数さん:2013/10/11(金) 23:50:41.48
i=e^(iπ/2)
∴√i=e^(iπ/4)=1/(√2)+i/(√2)
494132人目の素数さん:2013/10/11(金) 23:57:05.71
>>491
複二次式はやったか?
x^2=i
x^4=-1
x^4+1=0
(x^2+1)^2 -2x^2=0
{x^2+(√2)x+1}{x^2-(√2)x+1}=0
を解いて出てくる4つの解のうちの2つがx^2=iの解
残り2つはx^2=-iの解
495132人目の素数さん:2013/10/12(土) 02:16:22.34
x+y=170
x-y=32

この時のx、yの値を求めよ
496132人目の素数さん:2013/10/12(土) 02:27:31.88
x=101 y=69
497132人目の素数さん:2013/10/12(土) 02:30:52.94
球面座標や円柱座標などの一般座標におけるラプラシアンやdiv,gradを
計量で表す表記はよく見かけるのですがrotは計量で簡単に表されないのでしょうか
498KingMathematician ◆LoZDre77j4i1 :2013/10/12(土) 02:42:37.24
Re:>>495 そこで, x=(170*(-1)-1*32)/(1*(-1)-1*1), y=(1*32-170*1)/(1*(-1)-1*1).
Re:>>497 何を以ち簡単とする.
499132人目の素数さん:2013/10/12(土) 02:44:42.90
>>498 「簡単」は蛇足でした。計量を用いた表記ができるのでしょうか。
500KingMathematician ◆LoZDre77j4i1 :2013/10/12(土) 02:54:53.52
Re:>>499 grad ができれば rot もできるだろう.
501132人目の素数さん:2013/10/12(土) 02:56:03.18
>>500 ぜひ教えて欲しいです
502壊の国の狢 ◆yEy4lYsULH68 :2013/10/12(土) 02:57:02.18
503KingMathematician ◆LoZDre77j4i1 :2013/10/12(土) 03:03:17.80
Re:>>501 //www.akita-nct.ac.jp/yamamoto/study/electromagnetics/laplacian/html/node2.html
504132人目の素数さん:2013/10/12(土) 03:16:55.21
>>503 ありがとうございます
505職なし人:2013/10/12(土) 10:12:52.97
線形偏微分方程式
  p - q = f(y){Cx -z }、ただし、x = 0のときz = 0 の解を求めたいのですが、
ここで、p は zのxに関する偏微分、qはzのyに関する偏微分で、Cは与えられた定数です。
特性微分方程式は
  dx/ x=dy/-1=dz/f(y)(cx-z)
の一般解は
   x+y=a, z=e^{∫f(y)dy}[C ∫f(u)(u-a) e^{-∫f(v)dv} du +b]
となり、
  Φ(x+y, b)=0
が一般解になりそうですが、初期条件の使い方がわかりません。
506132人目の素数さん:2013/10/12(土) 12:40:42.82
>>492
>>494

分かりやすい解説、ありがとうございました。
507132人目の素数さん:2013/10/12(土) 13:02:14.23
508132人目の素数さん:2013/10/13(日) 00:48:18.01
A:n次正方行列
Aが冪零行列ならばA^n=O

これをケーリーハミルトンも線形写像も使わず、
一次独立・一次従属と背理法のみで示せますか?
509132人目の素数さん:2013/10/13(日) 01:11:39.80
A^nがゼロでないとしてこれでゼロに映らないベクトルを持ってくる。
するとn+1この独立なベクトルが存在することになり矛盾。
510132人目の素数さん:2013/10/13(日) 01:52:15.15
>>509
何故n+1個の1次独立なベクトルが存在すると矛盾するのか詳しく教えて頂けますか?
511132人目の素数さん:2013/10/13(日) 02:23:32.54
A:n次正方行列,(∃m∈N)A^m=0のとき
あるベクトルv、自然数kについて(A^k)v≠0ならばk+1個のベクトル
v,Av,…,(A^k)vは一次独立であることを示せ
512132人目の素数さん:2013/10/13(日) 03:20:22.14
>>511
k≧mとするとA^k=Oとなるから、k<m
c[0]v+c[1]Av+…+c[k](A^k)v=0 (0≦i≦k,c[i]∊K) (*)
が成り立つのがc[0]=c[1]=…=c[k]=0の場合に限ることを示す
(*)の両辺にA^m-1(≠O)を掛けると、A^m=Oより、
c[0](A^m-1)v=0⇔c[0]=0または(A^m-1)v=0
(A^m-1)v=0のとき、(*)の両辺にA^m-2を掛けると、
c[0](A^m-1)v=0⇔c[0]=0または(A^m-2)v=0
これを繰り返して、c[0](A^k)v=0
(A^k)v≠0より、c[0]=0
(*)に代入して以下同様の議論を繰り返し、
c[0]=c[1]=…=c[k]=0

こうですか?
ところでどうして一次独立なベクトルがn+1個存在してはいけないのでしょうか?
513132人目の素数さん:2013/10/13(日) 06:44:27.61
>>508
アホだろう
514132人目の素数さん:2013/10/13(日) 07:53:07.33
>>512
n次元 列ベクトル a1, a2, ... ,am に対して nxm行列 A を
A=[a1, a2, ... , am ] と置き、
左基本変形の重ね掛け(P)で階段行列に変形します。
PA=
1xxxxxxxxxxxxx
00001xxxxxxxxx
0000001xxxxxxx
00000000000000
00000000000000
例えばこんな形に。
PA から拾い取れる一次独立な列ベクトルの最大数 r (上の例ならr=3) が n を越えない事は明らかです。
Pは可逆なので、列ベクトル間の線形関係は不変です。
よって、a1, a2, ... , am から拾い取れる一次独立な列ベクトルの最大数は n 以下と分かります。
n+1 個のベクトルは、どのように選んでも一次従属になってしまうんです。
515132人目の素数さん:2013/10/13(日) 09:03:16.62
>>514
わかりました
ありがとうございました
516132人目の素数さん:2013/10/13(日) 09:09:37.52
以上で線型性の使用の有無について要検討
517132人目の素数さん:2013/10/13(日) 13:38:46.69
(A,M)が局所環のとき、supp[A](A/M)={M}であることはどのように示せばいいのでしょうか?
右が左に含まれることはよいのですが。
518132人目の素数さん:2013/10/13(日) 14:12:38.80
d次元単位球の「表面積」の定義と,その求め方を教えてください.
どなたかよろしくお願いします.
519132人目の素数さん:2013/10/13(日) 14:43:02.42
半径rのn次元の球の表面積S(n, r)、体積V(n, r)
V(n, r)=∫[0,r]S(n, r)dr
S(n, r) = V(n, r)/dr
520132人目の素数さん:2013/10/13(日) 14:49:40.95
V(n, r)=∫[-r,r]V(n-1, √(r^2-x^2))dx
521132人目の素数さん:2013/10/13(日) 14:54:04.48
>>518
d 次元 (d ≧ 2) にて、
半径rの超球表面積: S(r) = α*r^(d-1)
半径Rの超球体積: ∫[0,R] dr S(r) = ∫[r=0,R] dV(r) = V(R)
ガウス積分により
∫[-∞,+∞]dx ∫[-∞,+∞]dy... e^-(x^2)*e^-(y^2)*... = π^(d/2)
= ∫[0,∞]dr S(r) e^(-r^2) = αΓ(d/2) /2
よって
単位半径の超球表面積: α = 2*π^(d/2)/Γ(d/2)
522132人目の素数さん:2013/10/13(日) 15:22:52.84
523壊の国の狢 ◆ghclfYsc82 :2013/10/13(日) 21:11:55.47


>689 名前:132人目の素数さん :2013/10/13(日) 19:59:50.55
> 客観的に芳雄が正しいだろ
> 京大数学科に入れない学力だった癖に偉そうに批判するな
>
524518:2013/10/14(月) 16:19:14.12
すみませんが定義から教えて頂けないでしょうか.
ここに書くのが面倒なのでしたら,私の質問に対する答えが書いてある学部1〜2回生向けの数学の本の書名を教えてください.

どなたかよろしくお願いします.
525132人目の素数さん:2013/10/14(月) 17:07:29.46
中心から、半径rの領域にある点の集合の大きさ
526132人目の素数さん:2013/10/14(月) 17:13:34.83
∫{|x|=r} dV = 0
527132人目の素数さん:2013/10/14(月) 17:24:54.98
集合の大きさ → n-1次元の集合の大きさ
528132人目の素数さん:2013/10/14(月) 18:15:50.50
とりあえず高校生が書いているこれでも読んどけ
ttp://nadamath2012.web.fc2.com/bushi/2009_tau-2.pdf
529132人目の素数さん:2013/10/14(月) 18:46:57.88
高校生らしい


興味のある方は調べてみてください(僕は興味なんて全く無いので調べる気になりません。)
530132人目の素数さん:2013/10/14(月) 19:27:27.03
そんな揚げ足取りして虚しくならないのかね
531132人目の素数さん:2013/10/14(月) 19:32:18.82
高校生らしく頭の悪そうなpdf
可愛いじゃないか
532132人目の素数さん:2013/10/14(月) 19:34:17.85
そんな本当のこと言ったら泣いちゃうだろ
533132人目の素数さん:2013/10/14(月) 19:47:10.47
せめてもうちょい時間空けてから書きこめや
534132人目の素数さん:2013/10/14(月) 20:01:03.06
せめて多様体の例ぐらいにしといて
535132人目の素数さん:2013/10/14(月) 20:01:49.92
長さ1の正方形が書かれています。

コンパスと定規の作図で
長さ √17/ √2 の線分って作れますか?
536132人目の素数さん:2013/10/14(月) 20:03:23.79
>>535 √は必ず作れる
537132人目の素数さん:2013/10/14(月) 20:08:15.29
>>535
長さが1の正方形はない。一辺の長さかな。それとも面積が1の正方形かな。
有理化して√34/2なので、√34をつくればよい。
34=9+25なので直角をはさむ2つの辺が3と5の直角三角形の斜辺を二等分する。
538132人目の素数さん:2013/10/14(月) 20:28:42.85
>>537
ありがとうございます

>>536
え、そうなん!?
確かに
√2 , √5 はすぐに作れるし、
√3 も工夫すれば作れる・・・
まさか全ての整数で √ なんとか が作れるのか・・・
539132人目の素数さん:2013/10/14(月) 20:44:27.64
みんな頭いいな、旧帝大か!?
540132人目の素数さん:2013/10/14(月) 20:48:24.77
折り紙は3乗根作れるらしいね
541132人目の素数さん:2013/10/14(月) 20:57:40.31
>>540
折り紙の世界では、
√ √ 2 とか √π が作れるっていう意味?

逆にいうと、コンパスと定規だけでは
長さ √ √ 2 は作れないのか。
542132人目の素数さん:2013/10/14(月) 21:36:44.90
>>541
コンパスと定規の作図では
作図可能な数xに対し√xは作図可能だから
√√2は作れる。
√πはπが作図できないから無理。
543132人目の素数さん:2013/10/14(月) 21:42:37.41
折り紙なら2^(1/3)とか作れる
コンパスと定規だけでは作れない
544132人目の素数さん:2013/10/14(月) 21:53:07.60
箱を作って…
と思ったが、平方と逆数を作ればできるな
545132人目の素数さん:2013/10/14(月) 22:01:15.69
>>542-543
なるほど。

しかも √ √ 2 は四乗根 と等しいのであって、
3乗根とは 何も関係ねーな。
546132人目の素数さん:2013/10/15(火) 04:18:02.30
関数のグラフの問題です。
Y=-1/3x2乗
をグラフにするのに、
どうやってグラフにしたらいいのか
全く分かりません。
答えというより、解き方を教えてください。
547132人目の素数さん:2013/10/15(火) 04:27:05.10
頂点と通る点をどこか適当に1,2個書いてそれっぽく曲線を書いてください。
548132人目の素数さん:2013/10/15(火) 04:48:13.11
>>547
頂点は分かるのですが、
通る点の出し方が分からないのです…
549132人目の素数さん:2013/10/15(火) 07:52:43.81
x=1のとき y=-1/3
x=2のとき y=-4/3
x=3のとき y=-3
x=4のとき y=-16/3
x=5のとき y=-25/3
・・・・・・

こういう点をつなげばよかろうもん
550132人目の素数さん:2013/10/15(火) 07:52:49.39
>>548
xに適当な数字を入れたときのyの値を計算出来ないということ?
551132人目の素数さん:2013/10/15(火) 07:58:37.38
折り紙は極限操作ができるからか
552132人目の素数さん:2013/10/15(火) 08:07:01.56
出来るということにしているに過ぎない気もするけどなあ
553132人目の素数さん:2013/10/15(火) 08:24:12.90
んなこと言ったら、定規で直線を描けることにしているだけだよ
同じことだ
554132人目の素数さん:2013/10/15(火) 09:12:12.00
何を出来ることにしているのかを明確にしないとな。
定規とコンパスの場合のお約束ってどの段階で学ぶんだ?
曖昧なまま進んじゃうんだっけ?
555132人目の素数さん:2013/10/15(火) 10:06:42.09
定規とコンパスならこれくらいしかできないだろう。

2点を通る直線を引くことができる
1点を中心とする円を描くことができる
円や直線の交点を求めることができる。
556132人目の素数さん:2013/10/15(火) 10:25:46.51
コンパスは小学校ではじめて習うんだっけ?

たしか、2つの三角形の相似形の判定と一緒に
習ったような気がする。
ピタゴラスの3つの平方の定理は
まだ習っていない状態だった。

しかし、ピタゴラスの3平方定理を使わずに
作図をさせるってクレイジーだよな。
たいした作図もできんし、まったく教育的じゃない。
557132人目の素数さん:2013/10/15(火) 13:11:03.06
すいません、これって求まりますか?不定積分なんですけど

∫(1-a*e^(2x))/(1+a*e^(2x))dx

aという係数がなかったら
loglcoshxl

とすぐに求まるのですがaが係っていてわかりません・・・
558132人目の素数さん:2013/10/15(火) 13:23:04.88
559132人目の素数さん:2013/10/15(火) 13:23:06.71
>>557
ヒント:
a>0 の場合 a*e^(2x) = e^{ log(a) + 2x } = e^{2 (x + (log|a|)/2) }
a<0 の場合 a*e^(2x) = -e^{ log(-a) + 2x } = -e^{2 (x + (log|a|)/2) }
α≠0の場合 ∫f(α(x+β))dx = F(α(x+β))/α
560132人目の素数さん:2013/10/15(火) 13:44:24.14
おまえらなんで即答できるんだよ。
マジで 旧帝大生か何かなの?
561132人目の素数さん:2013/10/15(火) 14:22:05.91
>>558
>>559
有り難う御座いました><
562132人目の素数さん:2013/10/15(火) 15:42:41.40
>>560
いや、旧旧帝大生。
旧帝大生だと学生になっちまうからな。
563132人目の素数さん:2013/10/15(火) 15:48:37.58
旧帝大がどこかすら知らんな
564132人目の素数さん:2013/10/15(火) 15:50:19.55
実際旧帝大生多いと思うけどどこの大学かとかどうでもいいわー
565132人目の素数さん:2013/10/15(火) 16:34:16.71
>>563
東大、京大、阪大、城大、名大、北大、台大、東北大、九大
566132人目の素数さん:2013/10/15(火) 16:55:51.83
九州帝国大学
567132人目の素数さん:2013/10/15(火) 17:40:21.06
神戸帝国大学です!

ち、ちなみに謙虚な神戸大卒TOEIC700です… (; '‘ω‘`)
568132人目の素数さん:2013/10/15(火) 17:43:30.84
神戸って帝国大学だっけ?
違うような気もするが素でわからん
569132人目の素数さん:2013/10/15(火) 17:45:44.76
フカしてるだけです、サーセン!
570132人目の素数さん:2013/10/15(火) 18:34:50.37
>>568
565に書かれている通り。丁寧に書くと、
東京、京都、東北、九州、北海道、ソウル、台湾、大阪、名古屋
(順番は、設置順)
571132人目の素数さん:2013/10/15(火) 19:44:37.76
>>567
神戸は商科大学
572132人目の素数さん:2013/10/15(火) 20:04:21.10
旧三商って神戸とどことどこだっけ…
573132人目の素数さん:2013/10/15(火) 20:21:22.86
一橋と大阪市立
574132人目の素数さん:2013/10/15(火) 20:32:07.60
縦横3×3なら、端から端への最短経路は6C3=20通りだけど、縦横高さが3×3×3の骨組みなら端から端(左手前下から右奥上)の最短経路は何通り?
575132人目の素数さん:2013/10/15(火) 20:36:10.57
>>574
なぜ前者が6C3なのかわからんということ?
576132人目の素数さん:2013/10/15(火) 20:39:15.02
>>575
そうじゃない。
前者は例題で、解き方を知りたいのは後者
577132人目の素数さん:2013/10/15(火) 20:49:05.83
>>565>>570
へー、城大は京城で台大は台湾なのか、日本だけだと思った
韓国は旧帝大て隠してるんだろうな
578132人目の素数さん:2013/10/15(火) 20:50:12.91
>>576
なぜ前者が6C3なのかがわかれば後者も考え方は同じだろう。
579132人目の素数さん:2013/10/16(水) 00:47:43.45
A,Bがエルミート行列のときこれらが可換ならば同時に対角化可能(つまり同じユニタリ行列で
対角化できる)であることをどうやって示すのでしょうか。
580132人目の素数さん:2013/10/16(水) 01:14:40.23
片方を対角化して積の成分を計算してみろ
581132人目の素数さん:2013/10/16(水) 01:30:35.95
>>580
Aをあるユニタリ行列Uで対角化しそれをA'とする。
固有値a_iに対する固有空間をR_iとし、B'=U^†BUとすると
A'B'=B'A'なのでR_iに属する任意のベクトルxに対しA'B'x=B'A'x=a_iB'x
よってB'xはR_iに含まれる。

ここまではわかるのですがこのつぎ、ある演習書には
「B'がエルミートであることからB'は各ブロックがエルミート行列である
ようなブロック対角の形で書ける」と書かれていてここがわかりません
582132人目の素数さん:2013/10/16(水) 02:51:48.06
具体的にブロックで表わして確認してみろ
583132人目の素数さん:2013/10/16(水) 06:58:35.72
>>578
前者が何故6C3になるのかは分かるが、後者がどう考えたらいいのか分からない
584132人目の素数さん:2013/10/16(水) 07:16:11.34
>>582 どういうことですか?
585132人目の素数さん:2013/10/16(水) 07:45:11.86
>>583
組み合わせではなくて「同じものを含む順列」と見たら
586132人目の素数さん:2013/10/16(水) 07:51:46.56
>>583
例.
xxxyyyzzz
xxzyyyzzx
xxyzzyyzx
...
.
587132人目の素数さん:2013/10/16(水) 08:29:50.43
>>583
前者がなぜ6C3になるのか説明してみてもらえる?
588132人目の素数さん:2013/10/16(水) 08:31:17.14
前者の解き方がわかって後者がわからないというのは、いったいどういうことなんだろう?
前者の解き方をどう考えてるんだ?
589132人目の素数さん:2013/10/16(水) 08:32:49.20
>>576
まず、最短経路の一つを選び、骨組をなす経路の右行、左行、上行の個数を数えてみたら。
590132人目の素数さん:2013/10/16(水) 15:29:10.72
正五角形の作図って 義務教育に含まれてるっけ?

あれの証明をした記憶がない。
591132人目の素数さん:2013/10/16(水) 16:49:41.67
最近の指導要領の改訂で、高校数学にも作図が導入された。
中学数学では、角の二等分線、垂直二等分線、定点を通る垂線を主に扱う。証明はほとんど課されない。
高校数学ではもう少し高度に、内分点、長さ√n の線分の作図などを扱う。証明も(多分)する。
(実際の現場がどうなってるかは知らないが)

新指導要領の某問題集では、章末問題の最後の問題が正五角形の作図だったな
592132人目の素数さん:2013/10/16(水) 17:12:17.26
呆れた
そんなことをやって何の役に立つというのだ?
593132人目の素数さん:2013/10/16(水) 17:13:53.92
>>591
受験板でやれ
594132人目の素数さん:2013/10/16(水) 17:16:26.10
内分点、長さ√n の線分の作図ごときを高校でわざわざやるのかよ
アホくさ
595132人目の素数さん:2013/10/16(水) 17:21:23.54
>>591
中学まででやってたことを高校でやるということ?
596132人目の素数さん:2013/10/16(水) 18:40:33.36
虚数αは次の条件を満たしている
・α、α^2、α^3、α^4、α^5は相異なり、これら5つの数を解に持つ実数係数の5次方程式が存在する。

このような虚数αは全部で何個あるか。

友人からの問題、友人も悩んでいる
597132人目の素数さん:2013/10/16(水) 18:42:04.45
√の概念が高校なんだからしょうがないだろ。

おまえは小学生の頃に √n の作図を出来たのか。
598132人目の素数さん:2013/10/16(水) 18:48:29.50
はあ?
599132人目の素数さん:2013/10/16(水) 19:08:45.92
>>596
俺も悩んでいる
600132人目の素数さん:2013/10/16(水) 19:11:10.55
αをスカラー倍しても条件満たすんだから無数にあるだろ
601132人目の素数さん:2013/10/16(水) 19:13:26.95
虚数αか複素数かと思ってた
602132人目の素数さん:2013/10/16(水) 19:40:23.18
人員は9人。
ひと月(30日)の一人あたりの出勤日数は20日。
一日の出勤人数は6人。

この条件を満たすシフトは全部で何パターンありますか?
603132人目の素数さん:2013/10/16(水) 19:48:04.82
tf'(t)=0の微分方程式の解き方を教えてください。
604132人目の素数さん:2013/10/16(水) 21:07:12.19
それ f'(t) = 0 ⇒ f(t) = const. とどう違うんだ。
605132人目の素数さん:2013/10/16(水) 21:13:57.63
>>596
α_:αの複素共役, α_∈S:={α, α^2, ..., α^5}
α_=α^k (k=2〜5), |α|=1, α^(k+1)=1
k<4 => #S<5, NG
k=4 => α=exp(2nπi/5), n=1〜4
k=5 => α=exp(2nπi/6), n=1,5
計4+2=6コ
606132人目の素数さん:2013/10/16(水) 21:17:36.48
>>604
もとは
f'(t)+tf”(t)=0という方程式です。積分したらそうなってしまいました。
607132人目の素数さん:2013/10/16(水) 21:19:25.28
>>606
落ち着け。
積分したら 右辺は定数になるだろう。
608132人目の素数さん:2013/10/16(水) 21:36:24.13
>>607
微分方程式習ってなくてただ積分だけでやっていってますが、、一応
f'(t)=-tf”(t)
両辺をtで積分して
f(t)=-∫tf”(t)dt
=-tf'(t)+∫f'(t)dt
=-tf'(t)+f(t)
となってしまった、、
609132人目の素数さん:2013/10/16(水) 21:39:08.55
>>608
d/dt(tf'(t))=f'(t)+tf''(t)=0 だから tf'(t)=C(定数)
これから f'(t)=C/tにより f(t)=Clog(t)
610132人目の素数さん:2013/10/16(水) 21:39:24.76
>>608
不定積分を復習のこと
611132人目の素数さん:2013/10/16(水) 21:43:05.43
>>609
理解しました
定数部分落としてますね>>610ありがとうございます
612132人目の素数さん:2013/10/16(水) 21:46:16.98
>>609
さらに積分定数がいるのでは
613132人目の素数さん:2013/10/16(水) 21:47:50.02
>>597
>√の概念が高校なんだからしょうがないだろ。

最近は高校なの?
中学くらいで習った記憶はあるけど
614132人目の素数さん:2013/10/16(水) 21:53:15.42
>>608
習ったあとにおいで
615132人目の素数さん:2013/10/16(水) 21:53:30.40
>>612
代表元の提示で勘弁してくれ
616132人目の素数さん:2013/10/16(水) 22:11:48.02
>>602です
どなたかお願いします
617132人目の素数さん:2013/10/16(水) 22:13:19.27
いまやってます
618132人目の素数さん:2013/10/16(水) 22:16:46.82
>>602>>616
解く必要のない問題
619132人目の素数さん:2013/10/16(水) 22:56:23.80
>>617
ありがとうございます。
620132人目の素数さん:2013/10/16(水) 23:27:49.28
>>597
今どきの中学じゃ二次方程式もやらんの?
どんだけゆとりなんだか
621132人目の素数さん:2013/10/16(水) 23:29:29.85
1から160までの数字が書かれたカードが箱の中に160枚入っています。
その中から一枚ずつ引いてその数字を控えてからまた箱に戻します
これを繰り返して特定の6種類(1,2,3,4,5,6など)を少なくとも1回以上引き当てようとした時
試行回数が何回であればおよそ90%以上の確率で特定の6種類を1回以上引き当てられるのでしょうか
622132人目の素数さん:2013/10/16(水) 23:31:37.53
http://i.imgur.com/3UnbxAM.jpg

この@Aの式変形をどのようにすれば良いのか分かりません
どなたか教えて頂けないでしょうか
623132人目の素数さん:2013/10/16(水) 23:35:19.62
偏微分の定義
624132人目の素数さん:2013/10/16(水) 23:42:01.43
(1)テーラー展開
(2)わりざん、速度の定義
625132人目の素数さん:2013/10/16(水) 23:51:25.40
>>623
>>624
出来ました! ありがとうございました
626132人目の素数さん:2013/10/17(木) 01:18:03.32
横 w 高さ h の長方形がある。

1.これと面積が等しくなる正方形を作図せよ。
2. 面積が3倍の正方形を作図せよ。
3. 面積が7倍の正方形を作図せよ。


これって全部、高校の知識で解ける?
627KingMathematician ◆LoZDre77j4i1 :2013/10/17(木) 05:20:19.58
0 の平方根は複素数の範囲でひとつだけあり, 0 でない複素数の平方根は複素数の範囲で二つあり, 二つを足すと 0 になり,
複素数 z の平方根の虚数部分が 0 になる事と z の実数部分が 0 か 0 より大きく z の虚数部分が 0 になる事と同値である.

Re:>>626 相似直角三角形でできる.
628132人目の素数さん:2013/10/17(木) 06:44:27.70
>>626
受験板へ
629132人目の素数さん:2013/10/17(木) 07:56:35.56
>>626は高校数学のスレ行きだとは思うが
受験板に誘導するのも間違いだろ全く
630132人目の素数さん:2013/10/17(木) 09:48:11.08
>>587
合計6マス移動する訳であり、6マス中横移動が3マス。
6マスの中で横移動する回3つを選ぶから6C3
631132人目の素数さん:2013/10/17(木) 10:08:21.86
>>630
じゃあ、後者も同様に考えりゃいいだろ。
632132人目の素数さん:2013/10/17(木) 10:18:36.27
C[6.3]
633132人目の素数さん:2013/10/17(木) 10:56:51.65
>>628-629
ここは総合スレだから他へ誘導する必要は無いよ。
634132人目の素数さん:2013/10/17(木) 11:05:30.65
>>633
馬鹿は受験板へ
635132人目の素数さん:2013/10/17(木) 11:20:47.74
>>634
誰か説得できた?
636132人目の素数さん:2013/10/17(木) 11:23:16.78
>>634
じゃあ真っ先に受験板に行かねばならないおまえはこのスレを見ていない筈だよな。
637132人目の素数さん:2013/10/17(木) 11:34:34.10
作図 「私のためにー争わーないでー」
638132人目の素数さん:2013/10/17(木) 11:42:58.53
>>626
1
一直線上に3点A,B,Cをこの順に取り
AB=h
BC=w
とする。
Bを通るACに垂直な直線と
ACを直径とする円の交点をPとすると
△PBC∽△ABPであり
PB=√(hw)
これを一辺とする正方形を描けばよい。

2
1において√(hw)という長さの線分が得られたが
これを元に
AB=3√(hw)
BC=√(hw)
と取り直すと
PB=√(3hw)
これを一辺とする正方形の面積が3hw

3
AB=7√(hw)
BC=√(hw)
と取り直すと
PB=√(7hw)
これを一辺とする正方形の面積が7hw
639132人目の素数さん:2013/10/17(木) 11:57:10.72
640132人目の素数さん:2013/10/17(木) 12:01:54.12
「三次元空間内の格子点を3頂点とする正三角形の辺長の二乗は、6で割ると割り切れるか2余る」
この命題が正しいならば証明し、正しくないならば反例を挙げよ
641132人目の素数さん:2013/10/17(木) 12:10:08.68
>>626
高校というよりも中学の知識
1. ∠A=90゚の△ABCでAから辺BCに垂線AHを下ろすと、AH^2=BH・HCとなることを利用する。
  BH=w, HC=hとして、BCを直径とする円を描いてHでたてた垂線との交点を求める。
2. 三平方の定理を何回か使えばそのうち出てくる。(√2)^2=1^2+1^2, (√3)^2=(√2)^2+1^2
  または、2^2=(√3)^2+1^2を利用。
3. √3を利用すると、(√7)^2=(√3)^2+2^2
  または、4^2=(√7)^2+3^2や三平方の定理を6回使うなど。
642132人目の素数さん:2013/10/17(木) 12:30:21.56
>>639
「〜の学習範囲で解けるか」と受験は関係深いが
かといって完全に同一のカテゴリとは限らない
お前は先走り過ぎ
643132人目の素数さん:2013/10/17(木) 12:40:44.75
>>642
ようは雑談がしてーんだろ
644132人目の素数さん:2013/10/17(木) 12:43:36.47
>>535
いいエサだった、感動した
645132人目の素数さん:2013/10/17(木) 12:51:18.28
>>638 >>641

あざっす!
何でみんなすぐ解けるの?

こんなのウチの地元の中学ではやらなかったような気がする。
646132人目の素数さん:2013/10/17(木) 12:55:52.16
そのままのことは学校では教えない。中学の知識でということだ。
647132人目の素数さん:2013/10/17(木) 14:14:58.54
人員は9人。
ひと月(30日)の一人あたりの出勤日数は20日。
一日の出勤人数は6人。

この条件を満たすシフトは全部で何パターンありますか?
再度質問します
648132人目の素数さん:2013/10/17(木) 16:22:55.39
かなりのパターン数になると思うけど、もっと条件ないの?
三日に一度は必ず休みを入れるとか、
連休はなしとか、
同じ出勤パターンの人はいてはいけないとか、
一緒に仕事をした6人は、別の日には同じ6人で一緒に仕事をしないとか...
649132人目の素数さん:2013/10/17(木) 16:47:47.14
>>647
出典や前後の文脈を明らかにするように。
650132人目の素数さん:2013/10/17(木) 16:51:48.70
>>642,637,642,633
あげといたぞ
651132人目の素数さん:2013/10/17(木) 16:53:49.24
時間かけてプログラム作って何時間もかけて数えさせたら大体どのくらいなのか知りたかっただけと後出しされそうな問題
652132人目の素数さん:2013/10/17(木) 17:41:29.44
>>648
条件をそのままお願いします。
>>649
出典も前後の文もありません。
私のバイト先の形態です。
>>651
ここはプログラム板ではないので
計算方法とその答えを教えて欲しいのです
653132人目の素数さん:2013/10/17(木) 17:55:31.12
>>652
>私のバイト先の形態です。
なら労働基準法の縛りがあるんじゃねーの?
654524:2013/10/17(木) 17:59:26.37
何かいい本はありませんか?
あったら教えてください
655132人目の素数さん:2013/10/17(木) 18:17:08.48
>>652
>私のバイト先の形態です。

要は、最底辺の馬鹿が作った自作問題ってことでいいのか?
656132人目の素数さん:2013/10/17(木) 18:28:29.78
>>655
お前にバカ呼ばわりされる筋合いはないよ
問題が難しくて解けないんなら黙ってROMってろや、雑魚
657132人目の素数さん:2013/10/17(木) 18:37:06.24
manpower scheduling problem とか nurse scheduling problem は
結構難しそうな問題よね。全員の希望を出来るだけ叶えるアルゴリズムが色々研究されてる。
658132人目の素数さん:2013/10/17(木) 18:45:47.08
大学の授業のコマ割り も
最終的に人力で調節しているらしい。

学年が異なる複数の必修科目が
同じ時間に重なったりすると、
たまたまその1科目を落としただけで留年が確定したり 酷いことになるからな。
659132人目の素数さん:2013/10/17(木) 18:55:01.17
定数係数斉次線形差分方程式の解は
指数関数と多項式で解けますが、
非斉次になったら一般的な解法は無いですか?

特殊解が見つかれば一般解が書けそうですが、
その特殊解の見つけ方とか

宜しくお願いいたします
660132人目の素数さん:2013/10/17(木) 19:11:18.28
すごろくで、1マス先を踏む確立は、当然1/6ですが
2マス先を踏む確立はいくつですか?
1/6 (2が出る確率)
+ 1/6*1/6 (1が出て、更に1が出る確率)
で = 7/36ですか?

同様にして、3マス先を踏む確立
4マス先〜6マス先を踏む確立はいくつになるのでしょうか?
661132人目の素数さん:2013/10/17(木) 19:41:24.17
662132人目の素数さん:2013/10/17(木) 19:48:09.49
>>596 高校生向け
(x-a)(x-a^2)(x-a^3)(x-a^4)(x-a^5)
=x^5-r x^4+a^2(1+a^2)r x^3-a^5(1+a^2)r x^2+a^9r x-a^15,r=a+a^2+a^3+a^4+a^5
a+a^2+a^3+a^4+a^5,a^2(1+a^2),a^5(1+a^2),a^9,a^15が実数、または、a+a^2+a^3+a^4+a^5=0,a^15が実数

前者は、
→a+a^2+a^3+a^4+a^5,a^2(1+a^2),a^3が実数→1/a^2+1/a+1+a+a^2,1/a+a,a^3が実数→1/a+a,a^3が実数
「1/a+aが実数」からは、単位円かx軸上の点であることが要請され、「a^3が実数」からは偏角がπ/3の倍数であることが要請され、
さらに、題意にそぐわないものを除くと、exp(iπ/3)、exp(i5π/3)が、残る。

後者からは、exp(ikπ/5)、k=1,2,3,4
663132人目の素数さん:2013/10/17(木) 19:53:39.12
>>660
Mマス目を踏む確率 =
その1手前に Mから6マス以内 の距離に到達しており、その時に、それぞれ 1〜6の目を出せばよい


f(m) = f(m-1)*1/6 + f(m-2)*1/6 + f(m-3)*1/6 +
f(m-4)*1/6 + f(m-5)*1/6 + f(m-6)*1/6


初期条件、 0手目には、 0マス目にいるので、 f(0) = 1
また、m が負の値の時は f(m) = 0 となる。
664132人目の素数さん:2013/10/17(木) 19:54:18.43
>>660
「○マス進む」「□マス戻る」等のマスの配置次第
665132人目の素数さん:2013/10/17(木) 19:59:40.37
>>660
任意の場合の厳密な数値ではなく、
適当な数までの近似値で良ければExcel等で計算するのが分かりやすいと思う。
すごろくにかぎらずマルコフ連鎖の問題はExcelお薦め。
666KingMathematician ◆LoZDre77j4i1 :2013/10/17(木) 21:00:24.78
すごろくで何を確立するつもりか.
667132人目の素数さん:2013/10/17(木) 21:03:42.92
平成X年である時、西暦何年であるかを考えます。
ただし、途中で元号が変わったりする事はありません。平成25年は2013年です。

平成494243672179443681年は西暦何年ですか。
668132人目の素数さん:2013/10/17(木) 21:04:17.52
>>659
無い。
微分方程式と同じ事情で
存在しても既知の函数で書けるかどうかはまた別の話だ。
669132人目の素数さん:2013/10/17(木) 21:05:14.02
>>652
バイト先に、「僕が計算します」って言ったんなら、難しくてできませんでした、と誤った方がいい。
プログラム作るのだけでも大変そう。
それで動かしても当分(太陽系が無くなる頃になっても)計算は終了しない悪寒。
670132人目の素数さん:2013/10/17(木) 21:18:55.48
すいません 個人的にいろいろ考えてたのですが、分からないのでここで質問させてください

1回引くのに20円かかるクジがあります。クジの当選確率は1/300で、当たると5000円貰えます。
このクジに当選した際の60%の確率でボーナスタイムに入ります。
 ボーナスタイム中は、当選確率は1/30になり1回あたり0円でクジを引けます。
 この間も当選の60%はボーナスタイムになります。
当選した際の残りの40%はボーナスタイムには入りませんが、サービスタイムになり100回上限で0円でクジを引けます。
いずれの当りも再度当選した場合に、各タイムには再突入


というクジがあった際に、平均連続回数は、1/(1-60%)=2.5回
サービスタイム中に再度当りを引く場合も考慮した平均連続回数は約3.9回

というのまでは分かったのですが、この平均連続回数中の、0円でクジを引いている平均の回数を求めるにはどうすればよいでしょうか。

ぶっちゃけパチンコです。気に障られた片いたらごめんなさい。また、分かり難かったらごめんなさい 
もし分かる方いたらよろしくお願いします。
671132人目の素数さん:2013/10/17(木) 21:37:47.10
質問です。
googleで「分数」と検索すると画像検索を除けば6番目にヒットするサイトなのですが、このサイトによると「分数は数字ではない」とのこと。
しかしながら、数字とは数を表す記号であり、分数は有理数(実数)を表しているわけですから、分数を数字とみることを否定する理屈が理解できません。
↓このサイトの説明は妥当なのでしょうか?

http://reonreon.com/bunnsu.html
672132人目の素数さん:2013/10/17(木) 21:39:18.63
>>671
しらね
673132人目の素数さん:2013/10/17(木) 21:39:32.61
>>647
答えのオーダーを評価するために、かなり厳しい拘束条件を加えてみる
30日を3日を単位とする10のサイクルに分け、各人は、1サイクルの中で必ず一回休日をとらねばならないこととする
これならば、簡単に計算でき、初日は、84(=C[9,3])パターン、第二日は20(=C[6,3]パターン、第三日は1パターン。
1サイクルで1680パターンがある。10サイクルなので、1680^10=179098858256364459786240000000000≒1.8*10^32通りとなる。
この計算では、3連休以上を取るパターンは全くカウントされていない。
2連休もたまたまのは入っているが、すべての2連休がカウントされているわけではない。
したがって実際の数は、何万倍か、何億倍か、いや、もっと多いかもしれない。
そのような数に興味があるのか?
674132人目の素数さん:2013/10/17(木) 21:40:25.87
675132人目の素数さん:2013/10/17(木) 21:43:45.33
>>673
>>671を下げてそちらで聞いてみます。
ありがとうございました。
676132人目の素数さん:2013/10/17(木) 21:48:42.41
>>671
有理数を表す記号だろう。数字ではない。
勿論、それが表しているのは「数」だが。
677132人目の素数さん:2013/10/17(木) 21:55:49.21
>>673
お願いします
678132人目の素数さん:2013/10/17(木) 23:09:36.91
三次方程式 x^3+ax^2+b=0…@があり、x=1は方程式@の解である。
(1)bをaを用いて表せ
(2)@が異なる三つの実数解を持つようなaの値の範囲を求めよ
(3)(2)のとき@の異なる実数解をα、β、γとする。2β=α+γを満たすときaの値を求めよ

この問題の(3)がわかりません…。
解と係数の関係を使ってゴリ押しするしかないですか?
679132人目の素数さん:2013/10/17(木) 23:23:48.87
>>678
>解と係数の関係を使ってゴリ押し
ともかく試してみたら
680132人目の素数さん:2013/10/17(木) 23:34:50.88
>>679
式がぐちゃぐちゃになってどうしても途中で止まってしまいます…
(2)ででた範囲が a≦−1,3≦a
解と係数の関係と(3)の問題文から
α+β+γ=−a
αβ+βγ+βα=0
αβγ=a+1
2β=α+γ

の四つの式が出たのですが、これでいいんでしょうか?
681132人目の素数さん:2013/10/17(木) 23:35:56.82
>>671
2/3 は 2÷3 だから項とか式に近いって 言いたいんじゃないの。

2/3 ていう分数をパッとみた時に、
100 とか 3.14 よりも
7*3 とか 11*5 に近い感じがするじゃん?
682132人目の素数さん:2013/10/18(金) 00:33:34.00
>>680
β=(α+γ)/2という式を見てβが何か分からないか?
683132人目の素数さん:2013/10/18(金) 00:53:23.39
βを任意性で既知解とするなら、他根の和は(α+γ)=2。
一方(2)より他の根の和は-(1 + a)で範囲確認かな(黒猫
684132人目の素数さん:2013/10/18(金) 01:06:31.90
>>682
βは危険です。
685132人目の素数さん:2013/10/18(金) 01:31:27.66
>>640
正しい:
平行移動を考えることにより, 正三角形の頂点の一つは原点として構わない
残りの二頂点を(a,b,c),(x,y,z), 辺長をLとする
LL=aa+bb+cc=xx+yy+zz=(x-a)^2+(y-b)^2+(z-c)^2
LL=2(ax+by+cz)=aa+bb+cc=xx+yy+zz
LL=0(mod2)

以下 = はmod3, (x+a)^2+(y+b)^2+(z+c)^2=0
case (x+a,y+b,z+c)=(0,0,0)
正三角形の重心∈Z^3, (重心-頂点間距離)^2=(L/√3)^2∈Z, LL=0
case((x+a)^2,(y+b)^2,(z+c)^2)=(1,1,1)
a±b±c=0(複号任意), LL=aa+bb+cc=-(b±c)^2≠1
---(mod3制限終了)---

結局 LL=0,2(mod6)
686132人目の素数さん:2013/10/18(金) 01:54:20.17
>>677
大雑把な評価による下限 1680^10≒1.8*10^32
大雑把な評価による上限1 C[30,10]^8≒6.64*10^59
大雑把な評価による上限2 C[9,6]^29≒6.369*10^55

大雑把な評価
初日は84通り、最終日は1通りの選択肢がある
残りの28日を、この2点で直線的に補間すると
Π[k=0,29](84-83k/29)≒5.685*10^44
四分円弧で補間すると、
Π[k=0,29](1+83√(1-(k/29)^2))≒3.07989*10^52

この辺りに、求めるべき数があると思われる。
いずれにしても、PCによる数え上げ可能な範囲をはるかに超えていて、何らかの工夫が必要。
人間による対称性を最大限に考慮しても、9!で割るのがせいぜいだが、たとえ5桁下がっても、全く解決にはつながらない。
現実的な、数え上げ方法は、思い浮かびません。
687132人目の素数さん:2013/10/18(金) 02:16:12.10
>>686
ありがとうございます。
パソコンを持ってしても無理なんですね。。。
688KingMathematician ◆LoZDre77j4i1 :2013/10/18(金) 05:39:32.98
分数は, 単位を分母等分したものの分子個の事である.
それが除算と同じになるので除算にもなる.
689132人目の素数さん:2013/10/18(金) 06:31:20.23
>>688
×分数 ○分数の値が示す量
690132人目の素数さん:2013/10/18(金) 06:45:44.37
>>680
(2)の答えが違う
(3)α=1、β=1の場合に分けて地道に解く
691最初の質問者ではないが:2013/10/18(金) 07:41:32.62
>>685
case((x+a)^2,(y+b)^2,(z+c)^2)=(1,1,1)
↑(ここまでは理解出来た)
↓(ここへの流れがわからない)
a±b±c=0(複号任意), LL=aa+bb+cc=-(b±c)^2≠1
692132人目の素数さん:2013/10/18(金) 08:14:22.68
>>668
さんくす!
693132人目の素数さん:2013/10/18(金) 08:16:22.24
平面内の原点を中心とした半径rの円内にある格子点の数をf(r)とすると
f(r)=πr^2+O(r)、r→∞のとき
わかりません。よろしくお願いします。
694132人目の素数さん:2013/10/18(金) 08:21:20.86
回帰分析について疑問です。
例えば y = 10x + 3 という結果が出たとします。
これを検定するとき、帰無仮説としてxの係数を0とおいて、それが棄却されれば
「yとxは関係がある」という結論になりますよね。

でもこの検定って、y = 10x + 3 という式自体の正当性については
何も説明できていませんよね?
xの係数が0でないことはわかっても、0.0001である可能性は棄却されていません。
ましてや、「10である」ことの証明なんてどこにもありません。

それなのに、y = 10x + 3 という式に意味はあるのでしょうか?
695132人目の素数さん:2013/10/18(金) 08:50:25.35
696132人目の素数さん:2013/10/18(金) 12:07:58.72
>>691
以下 = はmod3
(p,q,r):=(x+a,y+b,z+c), p〜r=±1
aa+bb+cc=(-a+p)^2+(-b+q)^2+(-c+r)^2
2(ap+bq+cr)=pp+qq+rr=0, a±b±c=0 (複号任意)
LL=(b±c)^2+bb+cc=2((b#c)^2±3bc)=-(b#c)^2 (#は±逆,複号同順)
697640:2013/10/18(金) 17:21:07.24
>>685 >>696
ちょっと、読み取れない部分もありましたが、証明手順はわかります。
改めて精読してみたいと思います。お見事でした。
698132人目の素数さん:2013/10/18(金) 17:23:07.13
よろしくお願いします
>>693
699132人目の素数さん:2013/10/18(金) 17:43:05.53
格子点の数=面積+O(境界)ってまんまじゃないの
700691:2013/10/18(金) 19:08:15.20
>>696
ありがとうございます。全体が理解出来ました。
701132人目の素数さん:2013/10/18(金) 19:12:12.39
>>699
そのなんとなくそりゃそうだろって事をちゃんと式で示しなさいって事なんでしょ
ちょっと考えたけど俺には分からんかった。
702640:2013/10/18(金) 20:15:07.21
>>693 r≧1に対し、おそらくf(r)は、下の式で表される
f(r)=1+4Σ[k=1,[r]]([√(r^2-k^2)]+1)
第一項の1は、原点に対応、[√(r^2-k^2)]+1は、xがkで、yが0以上の格子点を数えている。
この項は、x軸上の格子点は数えているが、y軸上の格子点は除いている
(Σ[k=0,[r]][√(r^2-k^2)]として、y軸上の格子点を数え、x軸上の格子点を除いてもよい)

Σ[k=1,[r]]([√(r^2-k^2)]+1)をヒストグラム状の図形の面積として表し、
これが、半径r+1の円内に完全に含まれ、半径r-1の円を完全に含むことを示して、挟み撃ち、でどう?
703132人目の素数さん:2013/10/19(土) 18:25:31.30
1/ √n の線分を作図してしまった。

紀元前だったら今頃、おれは天才扱いされてるはず。
704132人目の素数さん:2013/10/19(土) 18:39:26.65
>>661 >>663-665
ありがとうございました
数が多くなってくると
なかなか綺麗に短い数式では表しにくい計算なのですね
705132人目の素数さん:2013/10/19(土) 23:20:25.43
F:=x^2+5xy+3y^2の値の範囲を調べる問題で、
まず平方完成してF=(x+5y/2)-13y^2/4
ここから、x=-5y/2のとき、F=-13y^2/4
yが実数全体を動く時、-13y^2/4は0以下の実数を全てとる
次に、y=0のとき、F=x^2, xが実数全体を動く時、x^2は0以下の実数を全てとる

よってFはすべての実数値をとる
となってるんですが、
それぞれの文「個々」については理解できるのですが、それらがなぜ結論に結びつくのか理解できません。y=0のとき〜←なぜ
706132人目の素数さん:2013/10/19(土) 23:30:36.73
>>705
xy平面上で定義された関数 F(x,y)=x^2+5xy+3y^2 の、
直線 x=-5y/2 上での振る舞いと、直線 y=0 上での振る舞いを調べた、てこと
707132人目の素数さん:2013/10/19(土) 23:32:16.66
>>706
なぜそのx=-5y/2とy=0を採用したんですか?
708132人目の素数さん:2013/10/19(土) 23:35:52.60
>>705
z=F(x,y)のグラフを
点(x,y)が
直線 x=-5y/2 の上を動いているとき、
また、 直線 y=0の上を動いているとき
の二つの場合を思い浮かべてごらん。
709132人目の素数さん:2013/10/19(土) 23:36:39.85
かぶった
710132人目の素数さん:2013/10/19(土) 23:41:35.48
-13y^2/4とx^2の値の範囲を調べてるように見える...この2つが0以下と0以上を動くからFは実数全体???ん??
711132人目の素数さん:2013/10/19(土) 23:50:48.35
>>710
単純にすると、 x^2-y^2 のとりうる値の範囲はどうなりますか?
712132人目の素数さん:2013/10/19(土) 23:58:28.00
>>711
0以上っぽい感じ
分かりません。
713132人目の素数さん:2013/10/19(土) 23:58:59.12
0以上ではないですね、、
714132人目の素数さん:2013/10/20(日) 00:02:17.27
0以上のものから0以上のものを引いたら、いくらでも大きく小さくできるのかな、、
極めて感覚的ですけど、数学的に処理できないです。
715132人目の素数さん:2013/10/20(日) 00:05:04.24
>>712
x=0のときyを色々に動かしてみる、
また、y=0のとき、xを色々に動かしてみる
716132人目の素数さん:2013/10/20(日) 00:08:17.65
>>715
x=0のとき、-y^2でyを動かすと0以下
y=0のとき、x^2でxを動かすと0以上
ってことはx^2-y^2は実数全体ですか?

でもなぜx=0,y=0を選んだのかと、x=0を選んだときになぜyを動かしたのか、y=0を選んだときになぜxを動かしたのか、これらの操作がなぜx^2-y^2の値の範囲を調べてることと同値なのかが理解できないんです。
717132人目の素数さん:2013/10/20(日) 00:10:57.08
もっと単純にすると、2つの自然数 a, b があるとき a-b のとりうる値の範囲はどうなりますか?
718132人目の素数さん:2013/10/20(日) 00:12:40.55
>>717
整数をとりえますか。
719132人目の素数さん:2013/10/20(日) 00:16:18.04
>>716
(x^2-y^2の値の範囲)⊇(x=0のときのx^2-y^2の値の範囲)∪(y=0のときのx^2-y^2の値の範囲)
であって一般には同値じゃない。この場合は
(x=0のときのx^2-y^2の値の範囲)∪(y=0のときのx^2-y^2の値の範囲)=(実数全体)
で、そもそも
(実数全体)⊇(x^2-y^2の値の範囲)
だから一致するというだけ。
720132人目の素数さん:2013/10/20(日) 00:22:05.25
>>716
整数全体になるね
721132人目の素数さん:2013/10/20(日) 00:22:38.82
>>719


【なぜx=0,y=0を選んだのかと、x=0を選んだときになぜyを動かしたのか、y=0を選んだときになぜxを動かしたのか】が分かりません。
722132人目の素数さん:2013/10/20(日) 00:27:57.23
>>721
試しに
723132人目の素数さん:2013/10/20(日) 00:30:07.25
>>722
724132人目の素数さん:2013/10/20(日) 00:32:30.80
>>721
今の問題はx,yの2変数ですが、3x^2-5y^2+4z^2 のとりうる値の範囲とか、5x^2+y^2+7z^2のとりうる値の範囲のときはどうしますか?
725132人目の素数さん:2013/10/20(日) 00:33:24.75
>>724
そもそも、2変数の時点で分からないのでそこから教えて欲しいのですが。
726132人目の素数さん:2013/10/20(日) 00:38:05.61
>>721
「なぜそうしたか」への答は「経験で培われた勘によって、そうすればいいだろうと見当がついたから」だろうな。で、後付けで証明を整えると。
727132人目の素数さん:2013/10/20(日) 00:39:54.91
>>726
x=3とかでもいいんですか?
728132人目の素数さん:2013/10/20(日) 00:47:13.16
>>727
いいよ。でもそれだけじゃ不十分でもっといろいろ試す必要があるだろうとは思うけど。
729132人目の素数さん:2013/10/20(日) 00:47:37.86
>>728
なるほど...
730132人目の素数さん:2013/10/20(日) 00:51:42.16
これだと3変数とかになったときにもっと分からなくなりそうです、、、。
731132人目の素数さん:2013/10/20(日) 00:52:31.91
>>725
別にx=0とする必要はありません。わかりにくくなるだけです。例えば、x=2とすると 4-y^2は4以下の実数をとる。
この範囲は0以下の実数を含んでいる。

極座標で示す方法もあります。
x=rcosθ, y=rsinθ (rは負でない実数)とすると
x^2-y^2=r^2cos2θとなって、-1≦cos2θ≦1より -r^2≦x^2-y^2≦r^2
r^2は負でない実数全体をとるので、x^2-y^2は実数全体をとる。
732132人目の素数さん:2013/10/20(日) 01:00:41.93
>>731
ならば改めて>>705の問題なんですが
僕的にはx=-5y/2,y=0は
F=0のときに、x=-5y/2,y=0なので、この2つを選んだのかなと思いました。
で、x=-5y/2のときにxを固定しててyの取りうる値の範囲を考えているのかなと。y=0についても同じです。
>>705の場合はグラフをイメージにして考えているのですか?
733悩める中3:2013/10/20(日) 11:18:04.37
中3です。教えてください。△ABCで∠B=∠C=48°△BCD
で∠C=∠D=72°。A,DはBCに対して同じ側にあるとき、
∠ADBを求めなさい。昨日から、5時間ぐらい考えています。わからないお願いします。
734132人目の素数さん:2013/10/20(日) 11:28:37.34
735悩める中3:2013/10/20(日) 11:42:36.31
733のものです。ありがとうございます。答えは30°とわかりましたが、
考え方がわかりません。すみません。教えてください。
736132人目の素数さん:2013/10/20(日) 12:00:43.75
>>735
ラングレー問題て書いてあるだろう
737132人目の素数さん:2013/10/20(日) 12:24:28.02
あんぐれえの問題は、解けねえとな。
738132人目の素数さん:2013/10/20(日) 12:27:05.64
http://i.imgur.com/LeCmMsu.jpg

この証明の仕方がわかりません
お願いします
739132人目の素数さん:2013/10/20(日) 12:29:04.01
>>738
自明。
740132人目の素数さん:2013/10/20(日) 12:31:21.95
>>739
証明せよという問題なので自明とは書けないです……
741132人目の素数さん:2013/10/20(日) 12:35:03.45
>>738
定義を書けよ
742132人目の素数さん:2013/10/20(日) 12:41:40.13
>>741
φについての定義は書かれてないです
練習問題なので……

問題の序文に以下のことが書かれています

スカラー関数f(x, y, z)について以下のように定義される

http://i.imgur.com/7EVHg32.jpg
743132人目の素数さん:2013/10/20(日) 12:43:31.96
>>742
問題の両辺をそれで書き下してみれば
744132人目の素数さん:2013/10/20(日) 12:43:41.55
>>742
x成分の定義は?
745132人目の素数さん:2013/10/20(日) 12:45:06.42
まさか大学生じゃないよな
746132人目の素数さん:2013/10/20(日) 12:47:04.49
>>743
書き下してもdf/dφは出ない気がするんですが……

>>744


>>745
Fラン大学2年生です……
747132人目の素数さん:2013/10/20(日) 12:55:23.57
>>746
さようなら
748132人目の素数さん:2013/10/20(日) 13:05:32.58
(∂f/∂φ)(∂φ/∂x)e_x=(∂f/∂x)e_x
749132人目の素数さん:2013/10/20(日) 13:10:14.99
>>732お願いしたいです。
750132人目の素数さん:2013/10/20(日) 13:12:39.75
お願い事は流れ星にでもしてください。
751132人目の素数さん:2013/10/20(日) 13:21:27.84
流れ星に願いを叶えてもらうよりかは、その儚き輝きに願いを投じる僕は彦星ですか?
752132人目の素数さん:2013/10/20(日) 13:21:58.88
くっそ意味わかんねえ
753KingMathematician ◆LoZDre77j4i1 :2013/10/20(日) 15:37:19.82
流れ星は地球の空気に紛れて地上全体に分布するか.
754132人目の素数さん:2013/10/20(日) 15:50:08.73
問題が回収されたので選択肢が思い出せないです。

・4人が持ち寄ったそれぞれが異なるカード1枚ずつを集めてシャッフルして、全員で引く。
1人だけが自分のカードを引く確率は?

自分の考え
1人が確定なら、残る3枚で考える。
2/3×1/2=1/3
もしくは1/4×2/3×1/2×4人分=1/3

・部品を受注した。注文を受けた翌日から作りはじめ、納品日の朝一に納品。
毎日45ずつ作れば30足りない数ができる。
毎日48ずつ作れば納品前日に24作ればよくなる。
受注の個数は?

自分の考え
45x=a-30
48(x-1)+24=a
a=540←選択肢になかた
755KingMathematician ◆LoZDre77j4i1 :2013/10/20(日) 16:01:13.33
Re:>>754 540=45*12, 540=48*11+12.
756132人目の素数さん:2013/10/20(日) 17:06:17.34
F:=x^2+5xy+3y^2の値の範囲を調べる問題で、
まず平方完成してF=(x+5y/2)-13y^2/4
ここから、x=-5y/2のとき、F=-13y^2/4
yが実数全体を動く時、-13y^2/4は0以下の実数を全てとる
次に、y=0のとき、F=x^2, xが実数全体を動く時、x^2は0以下の実数を全てとる

よってFはすべての実数値をとる
となってるんですが、>>731

僕的にはx=-5y/2,y=0は
F=0のときに、x=-5y/2,y=0なので、この2つを選んだのかなと思いました。
で、x=-5y/2のときにxを固定しててyの取りうる値の範囲を考えているのかなと。y=0についても同じです。
この場合はグラフをイメージにして考えているのですか?
757132人目の素数さん:2013/10/20(日) 17:15:09.34
ここは家庭教師スレではありません。
758KingMathematician ◆LoZDre77j4i1 :2013/10/20(日) 17:17:14.91
除算の考えでやるならば 540=11*48+12 と書くのが良いか.

Re:>>756 一応 graph でも考えられる.
759132人目の素数さん:2013/10/20(日) 17:18:18.42
ざっくりした質問なんですが
R^n空間上でノルムと不等式を使って評価する証明問題にちょくちょく√nとか出てくるのですが
あれは何を意味するのですか?
760132人目の素数さん:2013/10/20(日) 17:19:00.91
なんでしょうね
761132人目の素数さん:2013/10/20(日) 17:53:35.77
>>756
A,Bが実数のとき、A^2-B^2のとり得る値の範囲について
A=0のときは-B^2となって、0か負の数全体
B=0のときはA^2となって、0か正の数全体
よって、実数全体をとる。
762132人目の素数さん:2013/10/20(日) 18:34:02.90
>>761
ありがとうございます。
A,B,C実数で
ならばA^2-B^2+C^2の取りうる値の範囲はどうすればいいですか?
763132人目の素数さん:2013/10/20(日) 18:38:13.60
次はA^2-B^2+C^2-D^2だな
764132人目の素数さん:2013/10/20(日) 18:41:29.07
無意味に項を増やしてもしょうがあるまい
765KingMathematician ◆LoZDre77j4i1 :2013/10/20(日) 18:45:54.08
{A^2+B^2+C^2+D^2;(A,B,C,D)∈Z^4}={n;n∈Z∧n>=0}. 整数では四項まではとりうる値が変わる.
766132人目の素数さん:2013/10/20(日) 18:52:08.48
>>756
A= { {1, 5/2}, {5/2, 3} }  (行列)
v = (x,y)^t (縦ベクトル)
と置けば、
F=x^2+5xy+3y^2 = v^t A v

まずAの固有値を求める
det(A - λE) = (λ-1)(λ-3) - (5/2)^2 = 0
λ^2 -4λ -13/4 = 0
λ = {4±√(29)} /2 (固有値は正負2値)

よって適当な基底変換( Aが対称行列なので直交変換 )により
F= w^t B w = α x'^2 + β y'^2   (α>0, β<0)
に変形できる。これより F が全実数値を取れるのは明らか。
767132人目の素数さん:2013/10/20(日) 19:07:08.14
>>766
二次形式の話は進みすぎ
768132人目の素数さん:2013/10/20(日) 19:10:36.55
A,B,C実数で
ならばA^2-B^2+C^2の取りうる値の範囲はどうすればいいですか?
>>766
実は2次形式の話でそろそろその話題に入る予定です。
769132人目の素数さん:2013/10/20(日) 19:11:01.32
>>767
習ってない漢字使ったら先生に怒られたんですね
770132人目の素数さん:2013/10/20(日) 19:22:20.04
二次形式程度でグダグダ言うなら隔離スレから出てくるな
771132人目の素数さん:2013/10/20(日) 19:23:07.79
>>733
辺 BC の中点を M とし,C を B のまわりに 60°回転した点を E とする
∠ADB = ∠AEB = 30° (∵ △EBM は三角定規の形)
772悩める中3:2013/10/20(日) 19:35:33.35
771さんありがとうございます。わかりやすく、感動しました。
773132人目の素数さん:2013/10/20(日) 19:55:48.41
2{x+(3y+2z)/2}^2-(3/2)(y+5z/3)^2+43z^2/6
は常に非負の値をとりますか?
また、同時に零でないx,y,zに対して零になりうる2次形式ですか??
774132人目の素数さん:2013/10/20(日) 20:00:47.05
一目で分かるだろが
775132人目の素数さん:2013/10/20(日) 20:04:23.79
>>774
常に非負であるかが分かりません。
776132人目の素数さん:2013/10/20(日) 20:42:37.89
>>773
普通にバラして組み立てなおしたら

()^2 + ()^2 + ()^2 + x^2 + y^2 + z^2

って形が出てくるだろ。
777132人目の素数さん:2013/10/20(日) 20:50:13.54
>>776
展開したら
2x^2+3y^2+5z^2+6xy+4xz+yzです。

上でも教えてもらったんですが、
z=0,x=-(3y+2z)/2=-3y/2とすると
与式=-3y^2/2で0以下の実数
y=-5z/3とすると
与式=2{x+(3y+2z)/2}^2+43z^2/6となり0以上の実数で

与式は全ての実数値をとるじゃダメですか?
778132人目の素数さん:2013/10/20(日) 21:34:37.18
age
779132人目の素数さん:2013/10/20(日) 22:30:42.40
>>773は別すれで取り上げます。ありがとうございます。
780132人目の素数さん:2013/10/20(日) 23:18:09.63
>>970
それは次の話なんですよね、、
z=0,x=-(3y+2z)/2=-3y/2とすると
与式=-3y^2/2で0以下の実数
y=-5z/3とすると
与式=2{x+(3y+2z)/2}^2+43z^2/6となり0以上の実数で

与式は全ての実数値をとるじゃダメですか?
781132人目の素数さん:2013/10/21(月) 00:52:10.10
なんだマルチか
782132人目の素数さん:2013/10/21(月) 00:53:11.40
上はmiss
783132人目の素数さん:2013/10/21(月) 03:13:07.38
マルチだろ
784132人目の素数さん:2013/10/21(月) 16:45:17.31
教えてほしいんですが・・・

赤玉6個、白玉4個が入った箱から1個ずつ取り出し、4回元に戻すことを行った時の次の確率を求めよ。

(1)1回目と3回目に赤玉が出る

(2)2回赤玉が出る

(3)4回目に3個目の赤玉が出る

よろしくお願いします!
785132人目の素数さん:2013/10/21(月) 17:50:38.48
マルチだろ
786132人目の素数さん:2013/10/21(月) 18:13:40.11
3秒前に高校数学の質問スレ回ってきたところにこれだよ
787132人目の素数さん:2013/10/21(月) 20:26:23.34
t∈[0,1] f(t)≧0が連続で∫[0,1]f(t)dt=0なら、f≡0ですか?
788132人目の素数さん:2013/10/21(月) 20:31:12.62
>>787
あるt∈[0,1]についてf(t)=a>0とする
fの連続性から、あるδが存在して、x∈(t-δ,t+δ)でf(x)>a/2とできる
∫[0,1]f(s)ds≧∫[t-δ,t+δ]f(s)ds>δa>0
789132人目の素数さん:2013/10/21(月) 20:49:41.70
高1のユークリッド互除法の逆をたどり次の方程式を満たすx.yの一組を求めよという問題です。
278x+257y=1

模範解答には
1=21-5×4=21-(257-21×12)×4=257×(-4)+21×49←この49がどこから来たのか分かりません

続き
=278×49+257×(-53)
よって278×49+257×(-53)=1より
(x,y)=49,-53
となっています。他の数値の互除法の逆は解けるのですが、同じやり方をしているようでも答えが合いません。どうかお願いします。
790132人目の素数さん:2013/10/21(月) 20:57:17.18
>>789
21+21×12×4=21×49
791132人目の素数さん:2013/10/21(月) 21:01:41.06
確率の問題で


同じ部品を作っている2つの工場x,yがある全部品のうち60%をx 40%をyが作っている。
xの部品の10%が不良品yの部品の5%が不良品であるり全部品の中から1個を取り出すとき次の確率を求めよ

(1)良品を取り出す確率

(2)取り出した製品が良品の場合それがxで作られた製品である確率


わかる人いますか?
792132人目の素数さん:2013/10/21(月) 21:04:32.77
>>789
278x+257y=1
257(x+y)+21x=1
21(12(x+y)+x)+5(x+y)=1
793132人目の素数さん:2013/10/21(月) 21:07:56.61
テスト
794132人目の素数さん:2013/10/21(月) 21:11:32.38
>>789
> 1=21-5×4=21-(257-21×12)×4=257×(-4)+21×49←この49がどこから来たのか分かりません
21が現れている項を21でくくってみな
795132人目の素数さん:2013/10/21(月) 21:11:58.82
知恵遅れにあったんだけど、

三角形ABCにおいて、AB=3k BC=5k CA=7kとし、外接円の半径を7kとする。
また、三角形ABCの外接円の半径を7とする。

この時のkの値を求めよ。

分かる人いますか?
796132人目の素数さん:2013/10/21(月) 21:14:27.43
本当に知恵遅れな問題だな
797132人目の素数さん:2013/10/21(月) 21:24:00.95
>>794
すいません。よくわかりませんでした。よろしければ途中式を教えて下さい…
798132人目の素数さん:2013/10/21(月) 21:36:33.74
>>795
あんた、わからんってこと?
799132人目の素数さん:2013/10/21(月) 21:40:32.01
>>798

見て分からなかったので此方にお邪魔させていただきました。
800132人目の素数さん:2013/10/21(月) 21:41:54.97
>>799
問題文くらいちゃんと写せよ。
801132人目の素数さん:2013/10/21(月) 21:56:37.42
21-(257-21*12)*4
a-(b-a*12)*4
802132人目の素数さん:2013/10/21(月) 22:33:18.78
>>801やっぱり分からないです。

49の出し方が分かればスッキリなのですが…
803132人目の素数さん:2013/10/21(月) 22:50:09.68
>>802
手を動かせばすぐだろ、アホが

21-(257-21×12)×4=21-257×4+21×12×4=21(1+12×4)-257×4=21×49-257×4
804132人目の素数さん:2013/10/21(月) 22:50:18.51
>>795

k = 7/4 くらいだろ
たぶん
805132人目の素数さん:2013/10/21(月) 23:06:16.84
>>803
ありがとうございます!!プラス1をするのを忘れていて、ずっと悩んでいました!!!
本当にありがとうございました!
806132人目の素数さん:2013/10/21(月) 23:11:38.00
Aを奇数全体の集合、Bを偶数全体の集合とするとA∩B=Φですが、これを直接証明、すなわち背理法を用いずに証明できないでしょうか?
807132人目の素数さん:2013/10/21(月) 23:18:20.53
なに言ってんのかさっぱりわかんない
808132人目の素数さん:2013/10/21(月) 23:19:37.41
>>795
ピタゴラス 「どうせ 7/4 くらいだろ。よぉ、知らんけど」
809132人目の素数さん:2013/10/21(月) 23:31:13.22
二項係数の素因数分解の問題で
@ n+1|(2n,n)の証明
A pを素数とするとm+1|(p-1,m),p-m|(p-1,m)の証明
B (m1,m2,m3…mk)=1のとき(m1+m2+…+mk-1)!/m1!m2!…mk!は整数であることの証明
という問題がわかりません
これの範囲が高校生だったか思い出せないのでスレチだったらすみません
810132人目の素数さん:2013/10/22(火) 00:49:05.29
>>795
k=0
811132人目の素数さん:2013/10/22(火) 01:10:55.18
A∩B=A∩(Z-A)=(A∩Z)-A∩A=A-A=Φ
812132人目の素数さん:2013/10/22(火) 03:10:38.74
アスコリ アルゼラの定理の主張と証明が載ってる本教えてください
813132人目の素数さん:2013/10/22(火) 06:35:53.68
数列a_nに対して
(liminfa_n)^2=liminf(a_n)^2
は成り立ちますか?
814132人目の素数さん:2013/10/22(火) 06:42:11.17
a_n = -1+1/n 奇数
   = 0     偶数
815132人目の素数さん:2013/10/22(火) 07:21:31.67
816132人目の素数さん:2013/10/22(火) 09:05:20.55
599=1
615=3
676=4
687=5
706=6
722=7
753=?


この問題がわからない
817132人目の素数さん:2013/10/22(火) 09:07:44.25
818132人目の素数さん:2013/10/22(火) 09:08:28.72
>>816
9かな?そういうの見たことある気がする
819132人目の素数さん:2013/10/22(火) 09:11:16.06
規則性が掴めん
2が抜けてるのと3→4の増え方が大きいのが鍵か
820132人目の素数さん:2013/10/22(火) 09:12:44.58
2だけがないのか
821132人目の素数さん:2013/10/22(火) 09:13:14.23
8以上の数かな
822132人目の素数さん:2013/10/22(火) 09:13:19.83
規則性が掴めないな
お手上げ
823132人目の素数さん:2013/10/22(火) 09:17:49.80
2がないのがポイントかな
824132人目の素数さん:2013/10/22(火) 09:19:40.52
>>816
尊進数だよねこれ
アフリカの数学
825132人目の素数さん:2013/10/22(火) 09:22:10.24
2がなんか特別な意味をもってそう
826132人目の素数さん:2013/10/22(火) 09:22:34.62
ん?俺はアメリカの先住民に伝わる数学と聞いたが
827132人目の素数さん:2013/10/22(火) 09:23:49.15
順当に行けば8っぽいけど分からんわ
828132人目の素数さん:2013/10/22(火) 09:27:05.88
>>824
そんなの聞いたこと無い
ほら吹きは邪魔しないでくれ
829132人目の素数さん:2013/10/22(火) 09:29:07.28
難しいな
2を飛んで3なのは関係あるか?
830132人目の素数さん:2013/10/22(火) 09:38:02.35
599=1 16 2
615=3 51 1
676=4 11 1
687=5 19 1
706=6 16 1
722=7 31 1
753=? ?? ???
次の数との差を書いてみたら
16以外は素数と言うことに気が付いた。
2×8=16で2はもう出てるから???のところには8が入るから
答えは15だ。
831132人目の素数さん:2013/10/22(火) 09:47:48.62
これ結構前の海外の大学の問題だな
答え載ってるページがどこかにあったと思うけど
832132人目の素数さん:2013/10/22(火) 09:48:05.30
黒人唐澤貴洋が考案した尊進数やでこれ
833132人目の素数さん:2013/10/22(火) 09:55:53.89
>>830
51は素数じゃないぞ
834132人目の素数さん:2013/10/22(火) 09:57:48.20
なんかsageない人多いし変な事言う人増えてるね
vipあたりからきて茶化してるのかや?
835132人目の素数さん:2013/10/22(火) 10:03:13.76
>>833
676-615=61だから計算ミスかな?
836132人目の素数さん:2013/10/22(火) 10:08:46.04
なんJのゴミどもが迷惑かけてすいません
837132人目の素数さん:2013/10/22(火) 10:08:48.66
調べてみたらなんJ板にスレ立ってた数学問題でもなんでもない
838132人目の素数さん:2013/10/22(火) 10:29:12.56
偶然問題として成立してしまってるという
839132人目の素数さん:2013/10/22(火) 11:57:20.26
国会議員が、0.3平方キロメートルを、0.3キロ平米(=キロ平方メートル)
と同じだと思って、質問しているのですが、この国は大丈夫なのでしょうか?
840132人目の素数さん:2013/10/22(火) 12:02:38.34
「キロ平米」なんて単位使う機会あるんだ
841132人目の素数さん:2013/10/22(火) 12:05:21.45
ってか、あるのか?「キロ平米」
842132人目の素数さん:2013/10/22(火) 12:35:35.30
キ□平米
 ↑ここに入る文字を答えよ
843132人目の素数さん:2013/10/22(火) 12:36:42.87
尊寸法知らないのによくこの板にいれたもんだな
844132人目の素数さん:2013/10/22(火) 12:40:26.52
>>816
615=2
じゃなかった?
845132人目の素数さん:2013/10/22(火) 12:46:42.34
>>816
599=1
615=2
676=3
687=4
706=5
722=6
753=7
だと思う
846132人目の素数さん:2013/10/22(火) 12:47:43.50
馬鹿ビッパーがいっぱいいるなな、巣に帰れよ
847132人目の素数さん:2013/10/22(火) 12:48:34.67
ななことびまーす
848132人目の素数さん:2013/10/22(火) 13:14:47.36
尊進数も知らん奴がいるとは
849132人目の素数さん:2013/10/22(火) 13:29:59.62
数学板初めてンゴwwwwwwwwwwwwwwww
数学好きは見とけって言われたので来ました(悪いのか?)wwwww
好きな数字は、334ですwwwwwwwww
よろしくニキー!!!!wwwwwwww
850132人目の素数さん:2013/10/22(火) 13:34:59.70
VIPPAAもウェルカムだよー
851132人目の素数さん:2013/10/22(火) 13:35:27.81
>>849
ち〜ん(笑)
(334ネタを出すのは)やってしまいましたなぁこれは教育やろなぁ
852132人目の素数さん:2013/10/22(火) 13:52:41.33
びっぱーvsジェッパー、チーン
853132人目の素数さん:2013/10/22(火) 14:39:01.89
>>834
sageる必要なんて全くないだろう。
854132人目の素数さん:2013/10/22(火) 14:49:41.33
数学得意な人に質問です
中学生向けの数学教えるサイトで見つけたんだけど
24÷(-3)×4=32/3 になってた。
これは合ってるの?
855132人目の素数さん:2013/10/22(火) 14:59:33.11
>>854
この場合は左から順に処理していくから
24÷(-3)=-8
-8×4=-32

https://www.google.co.jp/#q=24%C3%B7(-3)%C3%974
856132人目の素数さん:2013/10/22(火) 15:09:53.77
合ってない
857132人目の素数さん:2013/10/22(火) 15:11:07.87
8/(-3)*4=32/3
858132人目の素数さん:2013/10/22(火) 15:13:26.62
8/3*4=32/3
128/(3*4)=32/3
859132人目の素数さん:2013/10/22(火) 15:15:14.25
>>854 24÷(-3)^2×4=32/3 という問題の写し間違いじゃないのか?
860132人目の素数さん:2013/10/22(火) 15:18:02.08
>>853
>>834にsageる必要なんて書かれてない。
861132人目の素数さん:2013/10/22(火) 15:21:52.63
2log|x|+2c'の計算教えてください

cは不定積分のやつ
862132人目の素数さん:2013/10/22(火) 15:22:34.21
863854:2013/10/22(火) 15:33:39.78
>>859
写し間違いは無いと思うけど、ここに載ってた。
ttp://contest.japias.jp/tqj2002/50027/index.html
864132人目の素数さん:2013/10/22(火) 15:50:36.25
>>863
そのサイトが間違ってるな。
-はどこかに行っちゃってるし、約分するときに分子の24だけを3で割って、分母はそのままだし。

それくらい、間違いだとわかれよ。
865132人目の素数さん:2013/10/22(火) 15:59:14.42
間違い発見を自慢したかっただけだろ
許してやれよ
866854:2013/10/22(火) 16:04:08.98
ときどき、こういう間違いを教えてる算数、数学サイトを見つける。
ということを書きたかっただけなんです。
867132人目の素数さん:2013/10/22(火) 16:25:03.16
>>854なんて短い文章に句点付け損ねたりするんだから
たくさん書けば間違いも入るだろ。
868132人目の素数さん:2013/10/22(火) 16:27:36.46
間違い指摘スレがあったはずだが埋まっちゃったかな
スレ立てると集まるか?
869854:2013/10/22(火) 16:41:22.98
>>867
句読点を付けていないことがあるのは態とです
870132人目の素数さん:2013/10/22(火) 16:43:39.11
>>868
ここらへんのこと?
数学所の誤りを指摘するスレッド
http://uni.2ch.net/test/read.cgi/math/1378799936/
有名書籍の間違っている箇所を指摘するスレ
http://uni.2ch.net/test/read.cgi/math/1345550976/
871132人目の素数さん:2013/10/22(火) 16:48:03.33
>>869
間違いを見つけて自力で訂正させるために間違いを態と入れるんです
872132人目の素数さん:2013/10/22(火) 16:51:51.19
873854:2013/10/22(火) 17:06:20.95
以前に、間違いだらけの小学生向けの算数サイトを見つけていたけど、
数年ぶりに見に行ったら、閉鎖されていた。

>>871
そうくると思ってました。

先生あるいは見本などが間違っていると、数学の苦手な人は、
まさか先生が間違っているとは思わないから、訳がわからなくなって、
ますます数学が苦手になって嫌になってしまう。
874132人目の素数さん:2013/10/22(火) 17:09:31.92
そんなことで嫌になるなら思う存分嫌になればいいだけじゃん
875132人目の素数さん:2013/10/22(火) 17:44:43.12
>>873
馬鹿発見
876132人目の素数さん:2013/10/22(火) 17:56:26.85
誤字熟語
877132人目の素数さん:2013/10/22(火) 18:05:54.94
教えてください。
勝ち負けの勝負を15回します。
その内、13回勝つ確率を教えてください。
878132人目の素数さん:2013/10/22(火) 18:08:35.96
い や で す
879132人目の素数さん:2013/10/22(火) 18:10:06.12
>>873
数学苦手な人ってのは
知能も低いってことだな。
880132人目の素数さん:2013/10/22(火) 18:13:48.24
反復試行
881132人目の素数さん:2013/10/22(火) 18:15:08.02
>>862
なんでこういう時に限ってVIPとなんJって仲良いの?
なんでも実況!VIP(仮)会議スレ
VIPガキ共荒らすなや死ね
ここをvipの植民地にしよう
VIPPER許さないカニ
VIPでパズドラ
VIPから来たンゴ
VIPからきたよ!よろしく
VIPからき☆すた
VIPから出張しにきたよー!!
VIPから来ますたwwwwwwwwwww
VIPからきますた
くっさいくっさいガキの荒し共VIPに帰れ
生き物苦手板をVIPとなんJで分割支配しよう
ここの原住民がVIPに避難してきてるんだが
なんでも実況VIP
VIPPER攻撃止め!
ワロタンポ なんVIP なんv
VIPPERマジ帰れ
大松「VIPとなんJは掛け持ちが多いんやで」
生き物(人間)苦手板はvipperに相応しい
VIPから来ますた
VIPから
882132人目の素数さん:2013/10/22(火) 18:15:48.58
>>862
vipで藤島
vipで藤島★2
VIPから来ましたカにですううううううううう
VIPPERって屑の集まりだな
VIPに帰れクソども
VIPから来た奴とか言うやつちょっとこいや糞
ここをVIPPERの植民地にしたい
VIPから来た人帰ってよ!!!!!!!!!!!!
VIPに帰れ
裏VIPからきますた( ^ω^ )
ガチホモ実況VIPが完成したわけだが…
VIPに帰れ荒らしども
VIPPERだけど
vipperraは巣に帰って
ようこそなんでも実況!VIP(生き物)板へ
ここってvipだよな?
VIPEER休憩所
ひっそりvip
vipから来たが犬とかいう害獣殺したい
VIPからきますた
vipper帰れば
VIPPERいい加減にしろ!!!
VIPに避難所立ってるから移住しよう
【一狩り】なんVIPdeMH4【行こうぜ!】
VIPPERむかつくからその報いをハムちゃんに
住民の避難先はvipに
883877:2013/10/22(火) 18:17:16.14
>>877
自己解決しました。
884132人目の素数さん:2013/10/22(火) 18:34:16.30
これが分かりません
お願いします

座標平面上において面積1の図形の周および内部に
X座標Y座標同士の差が共に整数となるような二点が
少なくとも一組は存在することを示せ
885132人目の素数さん:2013/10/22(火) 18:46:23.57
また後出しポエムか
886132人目の素数さん:2013/10/22(火) 19:04:11.87
ラプラス方程式△φ=0において境界条件が
∂D_1でφ=φ_1(=const.)
∂D_2でφ=φ_2(=const.)
のとき、解が定数しか有り得ずφ_1=φ_2となることを示すことってできますか?
領域としては小球のようなもの(D_1)と大球のようなもの(D_2)に囲まれた場所を考えています
「ようなもの」とつけたのは形が任意のもので示したいからです
887132人目の素数さん:2013/10/22(火) 19:13:50.78
>>886
まず球対称の場合とかでやってみたら
888132人目の素数さん:2013/10/22(火) 19:20:14.28
>>886 ディリクレ問題の一意性使えばいいんじゃね?
889132人目の素数さん:2013/10/22(火) 19:56:13.14
>>885
>>884は確かにそのまんまだとまともな問題の態ではなく
条件後出ししそうだが、ポエムは関係ないだろ
罵倒するなら条件後出しという言葉だけで罵倒しろよ
腐った罵倒は罵倒先よりイライラする
890132人目の素数さん:2013/10/22(火) 20:22:19.38
カルシウム足りてないんでね?
891132人目の素数さん:2013/10/22(火) 20:37:52.81
>>886
>領域としては小球のようなもの(D_1)と大球のようなもの(D_2)に囲まれた場所
ということは、D_1、D_2は各々閉局面なのか?だとしたら∂D_1、∂D_2って何?
892132人目の素数さん:2013/10/22(火) 20:55:33.05
>>888 というと?
>>891 D_1,D_2は小球と大球のようなものでその外表面、内表面を∂D_1、∂D_2としています。
∂D_1、∂D_2は各々小球面と大球面のようなもの、という言い方の方がわかりやすかったですかね
893132人目の素数さん:2013/10/22(火) 21:05:26.43
>>892
∂D_1: 平面x=0、∂D_2: 平面x=1、φ(x,y,z)=x とすると
△φ=0、φ(∂D_1)=0、φ(∂D_2)=1
894132人目の素数さん:2013/10/22(火) 21:09:39.24
電磁的には平面上のポテンシャルが与えられたときの、
空間ポテンシャルを求める問題の様ですが、でしたら
クリーンの方法を使えばよいとおもいます。

ただ接触のない平面には独立にポテンシャルを与えら
れるのではないのでしょうか。たぶん私か題意を判っ
てないと思います。
895132人目の素数さん:2013/10/22(火) 21:17:02.18
>>893
なるほど。コンパクト化されていないのが気になりますが
反例になってそうですね
>>894
言葉たらずですいません。実際の問題は電位が与えられている導体球面Aの中に
それより小さい導体球Bがあって導体球面Aの内部空間及び導体球Bに電荷分布が無い場合の電位を求めるというものです
クリーンの方法は初耳です
896132人目の素数さん:2013/10/22(火) 21:30:53.01
>>884
ピン刺し法で解けるのは知ってるよな?
897132人目の素数さん:2013/10/22(火) 22:07:17.35
数学と物理

どっちが得意な方が知能が高いと思う?

俺は数学は得意だったけど物理はイマイチな偏差値だった
898132人目の素数さん:2013/10/22(火) 22:09:56.08
>>895
板違いっぽいが、問題を正確に書いてみて
899132人目の素数さん:2013/10/22(火) 22:43:04.78
http://i.imgur.com/nVKGISN.jpg

この問題の立式が出来ないのですが誰か教えて頂けないでしょうか。
900132人目の素数さん:2013/10/22(火) 22:56:10.16
>>894 クリーンじゃなくてグリーンだろ w 必要ないが。

>>895 内部の空間に小球を囲む仮想球面∂Sを考え、その球面上の電界(場)Eにつて、
∫∫_∂S E・dS = ∫∫∫divE dV = 0 (内球の電荷). 対称性より E=0. よって
Eを内球から外球まで積分してもΔφ = ∫Eds = 0. よって両者表面の電位は同じ。
901132人目の素数さん:2013/10/22(火) 22:59:18.96
>>899
H = ∫[0,T]dh/dt dt
902132人目の素数さん:2013/10/22(火) 23:06:49.25
T = √ H
903132人目の素数さん:2013/10/22(火) 23:13:08.39
>>901
時間を求めなきゃ
T= ∫dt=∫[0,H]1/(dh/dt) dh
つかdh/dtがhの関数になってないと解けないだろ

>>900 やっちまったw必要ないが。 
904132人目の素数さん:2013/10/22(火) 23:13:48.98
>>897
書いてることのあほっぽさからして、お前は数学得意でも大したことないこと
の一例になる
905132人目の素数さん:2013/10/22(火) 23:15:28.32
>>902 は無視してくれ
906132人目の素数さん:2013/10/22(火) 23:16:21.20
>>904
阪大の2次試験までなら解ける気がする。
907132人目の素数さん:2013/10/22(火) 23:20:01.29
>>901
>>903
ありがとうございます ただこれだと解けないですよね 問題にはこれだけしか書いてなかったのですが条件不足なのでしょうか
908132人目の素数さん:2013/10/22(火) 23:21:12.75
sin^4x/cos^5xの積分ってどうなりますか?お願いします!
909132人目の素数さん:2013/10/22(火) 23:23:34.67
910132人目の素数さん:2013/10/22(火) 23:23:48.63
>>907
たいてい「流出口の流速は深さの平方根に比例する」みたいな条件がつく。
911132人目の素数さん:2013/10/22(火) 23:23:57.20
>>907
水がチョロチョロ出るかジャージャー出るかわからんのに、いつ終わるかわかるはずがない。
912132人目の素数さん:2013/10/22(火) 23:28:42.01
>>909
これって無限につづくってことですかね?
913132人目の素数さん:2013/10/22(火) 23:33:55.98
>>899
これの条件に排出口から出る水の速度は√2ghが抜けていました
ほんとすみませんでした
914132人目の素数さん:2013/10/22(火) 23:34:52.75
>>912
最後 +constant と書いてあるんじゃない?
915132人目の素数さん:2013/10/22(火) 23:36:10.70
>>907
少なくとも水の総量(or水槽の面積)、排出口の面積
そして重力などの排出力の設定がないと解けない。
916132人目の素数さん:2013/10/22(火) 23:38:57.68
>>914
かいてありますね!でもこの答えみただけじゃ、解法がわかりません(笑)ありがとうございました!
917132人目の素数さん:2013/10/22(火) 23:42:10.99
>>916
普通にy=sin(x)とチカン
918132人目の素数さん:2013/10/22(火) 23:48:03.49
ケケケ狢はチカンしたらタイホされました
919132人目の素数さん:2013/10/22(火) 23:48:56.63
ケケケ狢はチカンしたらタイホされました
920132人目の素数さん:2013/10/22(火) 23:53:43.54
>>900
球は正確には球みたいなものなので対称性を使わないで示したいです
921132人目の素数さん:2013/10/23(水) 00:00:41.18
>>917
置換じゃうまく解けないんですけど、とけますかね?
922132人目の素数さん:2013/10/23(水) 00:01:33.88
>>920
球の場合で解ければ、あとはどう変形させても同じことであるのは明らか。
923132人目の素数さん:2013/10/23(水) 00:02:00.42
924132人目の素数さん:2013/10/23(水) 00:10:55.84
>>908
sin^4x/cos^6x なら簡単だが、これはtan(x/2) = t とおいて有理関数にするしかなさそうだ。とても面倒。
925132人目の素数さん:2013/10/23(水) 00:15:19.22
ポエム系か
926既太郎 (2013年度 最強のアスペ):2013/10/23(水) 00:22:19.83
このスレを見てると
自分のアホさに嫌気がさしてくる。

何でおまえら即答できるの? 旧帝大生なの?
927132人目の素数さん:2013/10/23(水) 00:30:35.77
おまいら教えて(´・ω・`)
http://i.imgur.com/tISXEZR.jpg
928132人目の素数さん:2013/10/23(水) 00:33:59.66
はいはい定期ラングレーの問題
929132人目の素数さん:2013/10/23(水) 00:41:52.58
>>926
数学基地と比較してはだめ。
プロ野球の選手なんかとは自分を比較しないだろ?
それと同じ
930132人目の素数さん:2013/10/23(水) 01:40:00.02
上下それぞれに2分の1の確率で1m動く物体Xが有ります。
Xが10回移動した時位置は何処に居る確率が最も高いでしょうか?
Xが10回の移動中最も上昇した時と最も下降した時の位置は何処になる確率が最も高いでしょうか?

またXが最も上昇した時の位置が6mだった時、Xの最も下降した時の位置は何処になる確率が高いでしょうか?
最初の位置を0として±何メートルかで答えなさい。
931132人目の素数さん:2013/10/23(水) 01:51:28.09
いやです、絶対にいやです
932132人目の素数さん:2013/10/23(水) 02:00:11.71
>>931
そう言わずに解いてください
933戦国100:2013/10/23(水) 02:20:22.84
P回あがってQ回下がるとする。
上にx、下にyとすると
(p+q)^n= p^n+n p^(n-1)q+......+ q^n
だから
f(p、q)=(n,p)(1/2)^n

現在地=pーq だから
あとはコンボリュー氏ょんをつかって計算するといい。
いそがしいのでここまで
934132人目の素数さん:2013/10/23(水) 05:37:34.69
>>924
冗談ですよね…?
935132人目の素数さん:2013/10/23(水) 07:23:34.41
>>908
分母分子にcosxを掛ければt=sinxで置換積分できる
その後の計算も面倒だけど。
936132人目の素数さん:2013/10/23(水) 12:11:10.14
937132人目の素数さん:2013/10/23(水) 13:34:51.88
微分方程式の初期値問題を解けという問題で
dx(t)/dt=ax(t)+bu(t),x(t0)=x0
がわかりません
教えてください
938132人目の素数さん:2013/10/23(水) 13:46:44.61
今年の単位は諦めた方がいいぞ、マジで
939132人目の素数さん:2013/10/23(水) 14:35:20.77
>>937
自力で解ける微分方程式の初期値問題を一つ挙げてみ
940132人目の素数さん:2013/10/23(水) 14:40:53.57
>>939
dx/dt=axの問題とかは解けます
941132人目の素数さん:2013/10/23(水) 14:43:52.02
>>908
t=sin(x)とおくと、dt = cos(x) dx, dx = dt/cos(x)
∫sin^4(x)/cos^5(x) dx = ∫t^4/(1-t^2)^3 dt
部分分数に分解が面倒。値代入と微分と係数比較を併用した。
t^4/(1-t^2)^3 = 3/16・1/(1+t) - 5/16・1/(1+t)^2 + 1/8・1/(1+t)^3 + 3/16・1/(1-t) - 5/16・1/(1-t)^2 + 1/8・1/(1-t)^3
あとは容易だ。
∫t^4/(1-t^2)^3 dt = 3/16log(1+t) + 5/16・1/(1+t) - 1/16・1/(1+t)^2 + 3/16log(1-t) - 5/16・1/(1-t) + 1/16・1/(1-t)^2
= 3/16log((1+t)/(1-t) - 5/8・t/(1+t)(1-t) +1/4・t/((1+t)^2(1-t^2))
= 3/16log((1+sin(x))/(1-sin(x))) - 5/8・sin(x)/cos^2(x) + 1/4・sin(x)/cos^4(x)
942132人目の素数さん:2013/10/23(水) 15:57:13.51
>>980
>>941 は置換積分で計算したのだが、部分積分でもできることがわかった。∫sin^10^(x)/cos^11(x)なども計算できる。
三角関数ならこちらのほうが普通かもしれない。
∫sin^4(x)/cos^5(x) dx = ∫sin(x)/cos^5(x)・sin^3(x) dx
=1/4・1/cos^4(x)・sin^3(x) - 3/4∫sin^2(x)/cos^3(x) dx

∫sin^2(x)/cos^3(x) dx = ∫sin(x)/cos^3(x)・sin(x) dx
=1/2・1/cos^2(x)・sin(x) + 1/2∫dx/cos(x)

∫dx/cos(x) = ∫cos(x)/1-sin(x)^2 dx = 1/2∫(1/(1-sin(x) - 1/(1+sin(x))d(sin(x)) = log((1-sin(x))/(1+sin(x))

よって
∫sin^4(x)/cos^5(x) dx = sin^3(x)/(4cos^4(x)) - 3/4(sin(x)/(2cos^2(x)) - 3/16log((1-sin(x))/(1+sin(x))
943132人目の素数さん:2013/10/23(水) 19:45:24.97
>>816
これアフリカ人の唐澤貴洋が開発した尊師数だよね
日本じゃまだ知らない人も多いかも
944132人目の素数さん:2013/10/23(水) 20:35:46.55
座標系が異なるベクトルの内積はどちらかの座標系に変換してからじゃないと計算できませんか?

たとえば
A↑=(a,b,c) (デカルト座標)
B↑=(r,θ,Φ) (極座標)
の内積 A・Bを計算するにはどちらかの座標に統一しなければならないのでしょうか?
945132人目の素数さん:2013/10/23(水) 20:40:30.01
>>944
たりめーだ
946132人目の素数さん:2013/10/23(水) 20:59:36.57
>>945
ありがとうございます
947132人目の素数さん:2013/10/23(水) 22:24:57.72
>>937

移項して exp(-at) を掛けると
 {dx/dt - a・x(t)}exp(-at) = b・u(t)・exp(-at),
 (d/dt){x(t)・exp(-at)} = b・u(t)・exp(-at),
 x(t)・exp(-at) = x0・exp(-a・t0) + b∫[t0,t] u(s)・exp(-as) ds,
 x(t) = x0・exp(a(t-t0)) + b∫[t0,t] u(s)・exp(a(t-s)) ds,
948132人目の素数さん:2013/10/24(木) 00:18:30.73
f(x)を閉区間[0,1]上定義された連続関数で、
@定義域全体でf(x)>0
A∫(0,1)f(x)dx=1

を満たすとする。
このときある関数g(x)が存在して
Bg(0)=g(1)=0
Cf(x)>g(x)>0 (0<x<1)
D∫[0,1] g(x)=0.5
Eg(x)は微分可能

を満たすように作れることを示せ


という問題です、お願いします!
949132人目の素数さん:2013/10/24(木) 00:28:51.83
お願いはラピスペンダントにでもしてください。
950132人目の素数さん:2013/10/24(木) 01:25:51.08
>>949
死ねよ
お前みたいな精神的に向上心の無い者はは馬鹿だ
951132人目の素数さん:2013/10/24(木) 01:32:27.17
出たーSKB
952既太郎 (2013年度 最強のアスペ):2013/10/24(木) 03:02:29.75
>>944
この質問は知能がヤバイ、
俺が5歳の頃でもこんなアホな質問はせんわw

「3メートルの棒と5ヤードの棒をくっつけると長さは 合計何メートルですか?」
を想像すれば分かるだろ。
953KingMathematician ◆LoZDre77j4i1 :2013/10/24(木) 06:04:56.11
Re:>>948 閉区間の連続函数は Riemann 積分可能である.
954132人目の素数さん:2013/10/24(木) 09:18:40.43
>>953
どういうことでしょうか
955132人目の素数さん:2013/10/24(木) 11:07:46.53
R上の部分集合
族B={[a,b);a<b, a,b∈R}によって生成される位相Uを考え、
位相空間(R,U)をXと書くことにする。
f:X→R をf(x)=[x]で定める。fの連続性を判定せよ。


よろしくお願いします。
956132人目の素数さん:2013/10/24(木) 11:16:11.77
閉集合の逆像を考える
957948:2013/10/24(木) 12:49:38.06
>>938の問題ですが
結構考えてはいるんですがなかなか解けません
おそらく
@増大する関数列を考え
A中間値の定理をつかって

たしかに積分が0.5であるような瞬間もあるよね

ということをやるのだと思うのですが・・・
お力を貸してください
958132人目の素数さん:2013/10/24(木) 12:58:39.89
>>957
押せばいいのか?
959132人目の素数さん:2013/10/24(木) 13:21:51.14
>>938の問題て何?
960132人目の素数さん:2013/10/24(木) 13:50:37.74
>>948
閉区間の連続函数は一様連続
任意のε>0に対してδ>0が存在して|x−y|<δなら|f(x)−f(y)|<ε
閉区間を幅δ/2の小区間に分割して、それぞれの小区間でのfの最小値を取る階段関数Fを考えると段差はε以内
その階段関数より2ε小さい階段関数F'を考えるとF〜F'の隙間はε以上
その隙間を通るgを求める
961132人目の素数さん:2013/10/24(木) 14:00:00.22
たたみこみ。
962132人目の素数さん:2013/10/24(木) 16:38:03.49
963132人目の素数さん:2013/10/24(木) 17:09:36.72
わー、そんな面倒くさい問題はパスだ
964132人目の素数さん:2013/10/24(木) 17:50:20.32
965132人目の素数さん:2013/10/24(木) 20:25:09.19
>>960
もう少し詳しくおねがいします 
それだとg(0),g(1)>0となってしまいそうですが・・
966132人目の素数さん:2013/10/25(金) 09:33:57.17
f(x, y) の偏導関数を求めよって問題って

fx(x, y), fy(x, y) 求めればいいの?
967132人目の素数さん:2013/10/25(金) 11:36:13.26
そりゃそうだろ
968132人目の素数さん:2013/10/25(金) 12:46:33.42
赤・青・黄・緑の四色のシャツ・ズボン・帽子(計12点)があるとします。
このうち赤い製品3点は赤い箱に入れられており外からは製品の区別がつかず、箱を開けて確認するのに1分を要します。
また青・黄・緑の製品は色の区別もつかない頑丈な黒い箱に入れられており、中身の確認には2分を要します。

以上の条件でこの中から色の重複無く、シャツ・ズボン・帽子を一点ずつの一組を選びたいと思います。
色と種類が重複していなければ組み合わせは問いません。(例えば赤シャツ・青ズボン・黄帽子でも、緑シャツ・黄ズボン・青帽子でも可)

このとき効率的な作業方法というのは存在しますか?
969132人目の素数さん:2013/10/25(金) 13:05:24.01
日本語をエスパーしてまで考えたくない
970132人目の素数さん:2013/10/25(金) 13:55:45.37
>>968
4人が黒い箱、2人が赤い箱の確認に同時にとりかかる
黒い箱は4つ確認すれば最悪でも色・種類どちらも異なる組み合わせが存在する
残りの1つの種類は赤い箱のものから選ぶ

並列作業ができないなら、色・種類どちらも異なる組み合わせができるまで
黒い箱の確認をすすめる。2〜4箱で可能。
971132人目の素数さん:2013/10/25(金) 14:53:40.82
>>970
作業は一人(並列不可)という条件を書き忘れていました

黒い箱優先で確認していって、色・種類共に異なる組み合わせが出来てから
赤い箱を調べるという手順ですね
ありがとうございます
972132人目の素数さん:2013/10/25(金) 16:17:37.63
X:集合,Y:位相空間
f:X→Yを全単射とします
Xにfによる誘導位相、つまり
U⊂Xが開集合 :⇔ ある開集合V⊂Yが存在し、U=f^(-1)(V)
をいれると、fは位相同型ですか?
973132人目の素数さん:2013/10/25(金) 16:26:06.63
>>972
yes
fの連続性は自明
U⊂Xを開集合とすると、開集合V⊂Yが存在して、U=f^(-1)(V)
gをfの逆写像とすると
g^(-1)(U)=f(U)=f(f(-1)(V))=V
なので逆写像も連続
974132人目の素数さん:2013/10/25(金) 16:26:30.01
定義を確かめるだけ。
975132人目の素数さん:2013/10/26(土) 09:30:55.59
>>957
g(x)を構成しちまえば簡単なような

a,b∈[0,1), t∈(0.1)に対し, 次のようなR上のC^∞関数p(x)を取れる:
p=a: (-∞,0], a<p<1: (0,t/2), p=1: [t/2,1-t/2], b<p<1: (1-t/2,1), p=b: [1,+∞)

[0,1]の有限細分Λ:={I[1],I[2],..,I[n]} (I[i]:閉)を取る
I[i]上のfの最小値をm[i]とし, I[i]上の値をm[i]とする階段関数をs(x)とする
継ぎ目でのsの値の扱いはどっちでもいい

Λ∋I[i]:=[u,v]に対し, p[i](x):=m[i]*(p((x-u)/(v-u)) on I[i] とする
p[i]はI[i]上C^∞, p[i]の端っこでの値は[0,m[i])で自由に取れ,
{p[i]}を(0,1)上 >0, <=s(<=f) かつC^∞につなげられる
さらに, つなげた関数の端っこでの値は0に取れるのでそうしておく
この[0,1]上C^∞つぎはぎ関数のひとつをq[Λ](x)とする

q[Λ]の構成から, k:=∫[0,1](q[Λ])=Σ∫I[i](p[i])>=(1-t)*Σm[i]*|I[i]|
max{|I[i]|}->0 のとき, 右辺->(1-t)*∫[0,1]f=1-t
t<1/2 に取っておけば, Λを十分細かく取ることにより k>1/2
このΛに対し, g(x):=q[Λ](x)/(2k) とすれば条件をみたす
976132人目の素数さん:2013/10/26(土) 10:28:25.54
>>957
g_t(x)=1-((2x-1)^2)^t とすると
g_0(x)=0
0<x<1でlim[t→∞]g_t(x)=1だから
f(x)*g_t(x)について957の方針で行けると思う。
977132人目の素数さん:2013/10/26(土) 10:40:48.84
微分可能性は?
978132人目の素数さん:2013/10/26(土) 11:17:26.88
あー、もとのf(x)が微分可能とは限らないのか。
そうすると区間に区切って構成する方針のほうが良いのか。
979132人目の素数さん:2013/10/26(土) 11:55:29.83
960で終わってんだろう
980132人目の素数さん:2013/10/26(土) 12:52:00.96
端っこのこと忘れてね?
981132人目の素数さん:2013/10/26(土) 12:55:43.10
問題ない
982132人目の素数さん:2013/10/26(土) 13:54:48.77
>>980
元が自作くせーからどうでもいいが、
切り落としてつなげりゃいっちょうあがり
さらにいうと積分の構成法にかかわる話だからあたりまえ体操
983KingMathematician ◆LoZDre77j4i1 :2013/10/26(土) 17:10:53.57
端っことは何か. 端っ子か.
984132人目の素数さん:2013/10/26(土) 17:37:27.27
端っ子ってことはアレか
kingのことか
985KingMathematician ◆LoZDre77j4i1 :2013/10/26(土) 18:04:45.91
Re:>>984 地球面に端は無い.
986132人目の素数さん:2013/10/26(土) 18:16:28.18
平面上に三角形OABがあり、OA=√3、OB=√2、AB=2である。
また、Oから直線ABに引いた垂線と直線ABとの交点をHとする。

(1)内積ベクトルOA・ベクトルOBを求めよ。
(2)ベクトルOHをベクトルOA、ベクトルOBを用いて表せ。またベクトルOHの大きさを求めよ。
(3)Pが線分OH(両端を含む)上を動くとき、
|ベクトルOP|^2+|ベクトルAP|^2+|ベクトルBP|^2の最大値と最小値を求めよ。

(1)は1/2となったんですがよくわかんないです。教えてください
987KingMathematician ◆LoZDre77j4i1 :2013/10/26(土) 19:28:58.20
Re:>>986 内積は直観では分からぬかもしれない. 計算方法が分かればできる.
988132人目の素数さん:2013/10/26(土) 19:29:17.50
>>948 >>957

 G(x;a) = {x(1-x)}^a f(x),
は (3)(4)(6) を満たす。
 ∫[0,1] G(x;0) dx = ∫[0,1] f(x)dx = 1,
0 < x(1-x) < 1/4 より
 ∫[0,1] G(x;1) dx ≦ (1/4)∫[0,1] f(x)dx = 1/4,
これと中間値の定理から、ある 0<a<1 に対して
 ∫[0,1] G(x;a) dx = 1/2,
g(x) = G(x;a) とおくと (5) を満たす。
989132人目の素数さん:2013/10/26(土) 19:34:53.65
>>987
あのあとも解いてみたんですが
(2) ベクトルOH=3/8OA+5/8OB
(3)はOP=KOHとおいて計算してみましたがすごい気持ち悪い数になってしまって・・・
990132人目の素数さん:2013/10/26(土) 19:53:42.23
>>986
 内積は直観では分からぬかもしれない。そこで計算する。
(1)
 AB^2 = (↑AB・↑AB)
    = ((↑OB - ↑OA)・(↑OB - ↑OA))
    = OA^2 + OB^2 - 2(↑OA・↑OB),

 (↑OA・↑OB) = (OA^2 + OB^2 - AB^2)/2

(2)
 ↑OH = (1/8)(3↑OA + 5↑OB),
 OH = (1/4)√23,

なお、
 O (0,0)
 A (OH, -5/4)
 B (OH, 3/4)
991132人目の素数さん:2013/10/26(土) 20:10:57.80
>>988
6を満たさない。
992132人目の素数さん:2013/10/26(土) 20:18:13.17
え?
993132人目の素数さん:2013/10/26(土) 20:19:02.44
失礼。fは連続なだけだった
994KingMathematician ◆LoZDre77j4i1 :2013/10/26(土) 22:33:22.85
Re:>>989 (3) の最大値は 5, 最小値は 37/12.
995132人目の素数さん:2013/10/27(日) 15:22:47.08
>>994
 たしかに気持ち悪いな。
996132人目の素数さん:2013/10/27(日) 18:22:05.29
次スレ立て失敗…
997132人目の素数さん:2013/10/27(日) 18:38:57.61
分からない問題はここに書いてね385
http://uni.2ch.net/test/read.cgi/math/1382866679/
998132人目の素数さん:2013/10/27(日) 22:04:30.34
      ∧_∧
     ( ・ω・) ザック    lヽ,,lヽ
     (つD―○|> ザック (    )
     ( ヽノ 彡.    ゚。°と.、  i
      し(_)         しーJ
 ̄ ̄ ̄ ̄ ̄ ̄\  384  / ̄ ̄ ̄ ̄
999132人目の素数さん:2013/10/27(日) 22:10:01.36
      ∧_∧
     ( ・ω・) ザック    lヽ,,lヽ
     (つD―○|> ザック (    )
     ( ヽノ 彡.    ゚。°と.、  i
      し(_)         しーJ
 ̄ ̄ ̄ ̄ ̄ ̄\  384  / ̄ ̄ ̄ ̄
1000132人目の素数さん:2013/10/27(日) 22:20:50.66
                      へ、      _____
                 /    \   /´   ___`\
              ノ   {     \    . : ´ : : : : : : : : ` ー―一ノ
               /      \     } / : : : : : : : : : : : : : : : : : :く
            {     \    :,   ノ\: : : : : : : : : : : : : : : : : : : : :,
          〃 \   \_,ノ ̄ xく: : : :/ : : : : : : }/ : !: : : : : |
          |:\  \__/{  /:i |: ー{‐\ : : : /=ミ、: |:\ : :|
          |:.:.:.:\__,ノ_人_/.:.:o:しj: :|,x:=ミ、\/んハ Y|: :|x\ハへ、
           ‘:.:.:.:.:.:.:.:.:.:.:.:.:.:.:.:.:.:.:.:.:.:./: 〃んハ   V:り {: :! }:.:0} つ 385
               \:.:.:.:.:.:.:.:.:.:.:.:.:.:.:.;xく : 从 V:り   '    } :‘く:.:ノ´
             _\:.:.:.:.:.:.:.:.:./ヽ:{_\{: \ r    ̄}   人: : : \
      ┌―' \/: : : : : : ̄:, ̄    }ー=ニ\: :\ __ノ イ: \\: : : \
      |: : : 「\;/ ̄二ニ=-′   .ノ   ヽヽ : i ̄ハ\ノ \: ヽ \: : : :,
      ̄     (: : r――‐:,           } | : |  | :,   } : }  } : :|
              ): :)    /⌒>ー┬  ノ/: :八 ,;   } く: :く .ノ: : : |
            く: :く__ | ̄ ̄|:{ : :{ 人_/ : /  ∨  /┌:': :/  \ : く
              | : : | |: : : : ∨: ̄ ̄ : : :/  / /\|__| _/: /
              ̄ └―‐x、;/厂 ̄    / 〉\0}  |: : : :!
                  _,厶=―/     /     }  ̄`> 、|__|
                   // /      ̄     ノ   /  _\
                 i (O)|         /   /  / ̄ ̄}
10011001
このスレッドは1000を超えました。
もう書けないので、新しいスレッドを立ててくださいです。。。